You are on page 1of 177

THNG 12

Ch bin:
TRN NAM DNG
Bin tp vin:
V QUC B CN
TRN QUANG HNG
NGUYN VN HUYN
NGUYN TIN LM
L PHC L
NGUYN TT THU
NG NGUYN C TIN

LI NG CHO EPSILON S 6
Ban Bin tp Epsilon

Cui cng s 6 ca Tp ch Epsilon cng n tay cc bn. S 6 ca nm u tin.


Bt u t ngy 13/2/2015, ri n hn li ln, c vo ngy 13 ca nhng thng chn, Epsilon-tp
ch online ca nhng ngi yu ton li c ra mt hng trm, hng ngn c gi.
V c c s u n vi ni dung ngy cng phong ph v hnh thc ngy cng p hn
nh hin nay l s n lc ca c mt tp th: t nhng ngi vit bi n cc bin tp vin. Tt
c u lm vic trn tinh thn t nguyn v mong mun ng gp cho cng ng.
c bit, d hon ton da trn tinh thn t nguyn, khng c quyn li vt cht, cng nh bt
c rng buc php l no nhng tt c mi ngi u lm vic vi tinh thn trch nhim cao, c
nhng m phi thc trng hon tt bi vit hay hon chnh phn bin tp.
Qua cc s bo, Epsilon dn c thm c nhiu tc gi hn, nhiu cng tc vin hn v nhiu
c gi hn. i ng bin tp cng c b sung v s lng v nng cao v cht lng, va m
bo c cng tc bin tp ng tin , va ch ng to ngun bi di do cho cc s bo.
Chng ta i qua c 1 nm y kh khn nhng cng tht t ho. C nm u tin, c ngha
l s c nm th 2, th 3...
V Epsilon c tip ni, ngun nng lng ln nht i vi chng ti vn l s ng h ca
cc c gi. Nhng s gp , bnh lun, t hng t pha cc c gi s l ng lc cho Epsilon
tip tc c n hnh. c bit, ban bin tp lun ch n cc bi vit t pha c gi. Chng ti
tin rng nhng bi vit ca cc bn chc chn s lm tng thm s phong ph ca tp ch, v ni
dung ti ln phong cch hnh vn.
i nhiu ngi ta s i rt xa.
Thng 12, 2015,
Ban Bin tp Epsilon.

MC LC
Ban Bin tp Epsilon
Li ng cho Epsilon s 6 . . . . . . . . . . . . . . . . . . . . . . . . . . . . . . . . . .

Ng Quang Hng
Tnh duy l ca hm c ti . . . . . . . . . . . . . . . . . . . . . . . . . . . . . . . . .

L Ngc Tu
Xp x Diophantine vi o - nh l Khintchine . . . . . . . . . . . . . . . . . . . .

21

m Thanh Sn
Hnh hc ri lng t . . . . . . . . . . . . . . . . . . . . . . . . . . . . . . . . . . . .

33

I. I. Blekman, A. D. Myshkis, Ya. G. Panovko


Logic ca ton hc ng dng . . . . . . . . . . . . . . . . . . . . . . . . . . . . . . . .

39

Desmond MacHale
Cuc i v s nghip ca George Boole - S khi u cho k nguyn k thut s . . . .

55

Bnh Nguyn
Nhn dng ch mo . . . . . . . . . . . . . . . . . . . . . . . . . . . . . . . . . . . . .

59

ng Nguyn c Tin
Bi ton cn tin . . . . . . . . . . . . . . . . . . . . . . . . . . . . . . . . . . . . . . .

67

Nguyn Tin Dng


Xung quanh bi ton hnh hc trong k thi VMO 2014 . . . . . . . . . . . . . . . . . .

77

Nguyn Ngc Giang


Mi lin h Euclide, Afin v X nh qua mt bi ton trong sch "Cc phng php gii
ton qu cc k thi Olympic" . . . . . . . . . . . . . . . . . . . . . . . . . . . . . . . .

89

Trn Quang Hng


V bi hnh hc thi IMO nm 2009 - Ngy th hai . . . . . . . . . . . . . . . . . . . . . 103
4

Tp ch Epsilon, S 06, 12/2015


Trn Minh Hin
Thut ton tham lam trong xy dng cu hnh t hp . . . . . . . . . . . . . . . . . . . 109
Lu B Thng
nh Bertrand . . . . . . . . . . . . . . . . . . . . . . . . . . . . . . . . . . . . . . 125
Kiu nh Minh
Chui iu ha . . . . . . . . . . . . . . . . . . . . . . . . . . . . . . . . . . . . . . . 133
Yimin Ge
S d ca A:ax C Bx . . . . . . . . . . . . . . . . . . . . . . . . . . . . . . . . . . . . 149
Trn Nam Dng
Bi ton hay li gii p . . . . . . . . . . . . . . . . . . . . . . . . . . . . . . . . . . 153
Trn Nam Dng
Cc vn c in v hin i . . . . . . . . . . . . . . . . . . . . . . . . . . . . . . . 157
Ban Bin tp Epsilon
K thi Ton quc t Formula of Unity - The Third Millennium . . . . . . . . . . . . . . 173

Tp ch Epsilon, S 06, 12/2015

TNH DUY L CA HM C TI
Ng Quang Hng
(i hc Buffalo, M)
Bi vit ny chng minh mt nh l kinh in ca Kinh T hc, gi l nh l
bt kh thi ca Arrow. nh l c nhiu chng minh ngn gn, ch khong na trang.
Nhng chng ta s chn mt con ng tng i di n cng kt lun. ch
n, nh ngi ta thng ni, i khi khng th v bng ng i.

1. nh l bt kh thi ca Arrow
Marquis de Condorcet l mt trit gia, nh ton hc, v nh khoa hc chnh tr ngi Php sng
th k 18. Nm 1785, ng vit bi Essay on the Application of Analysis to the Probability of
Majority Decisions c nh hng su rng n l thuyt chn la x hi, kinh t hc, v n cc
thut ton xp hng qung co trn mng. Condorcet l mt trong nhng ngi u tin mang
(tnh cht ch ca) Ton hc vo nghin cu khoa hc x hi. ng tham gia cch mng Php,
vit vi quyn sch bt h ng h cho tinh thn Khai Sng. ng b bt giam gn mt nm, v
mt trong t. Nhiu kh nng l do t ung thuc c.
ng khm ph ra nghch l Condorcet. i ci nghch l ny nh sau. Gi s nh nc cn
u t vo ngnh giao thng (GT), y t (YT), hoc gio dc (GD). Nh nc lm trng cu dn
. Mi ngi dn b phiu xp hng ca ring mnh v tm quan trng ca ba ngnh ny. V d,
anh A bo ti ngh GD trc, ri n GT, ri n YT. Anh B chn YT > GD > GT, vn vn. Th
kh nng sau y c th xy ra: a s mi ngi xp GD trn YT, a s xp YT trn GT, v a
s xp GT trn GD. l tnh phi l ca chn la x hi. Khi bit ci nghch l Condorcet ri,
chng ta c cc thng k x hi cn thn hn. Obama vi McCain ci nhau, u li thng k ra.
Mt ng bo phi u t ci ny do a s dn chng ng h ci ny hn ci kia, McCain bo ci
kia hn ci n. Chng ta nn ngh ngay n kh nng v l ca chn la x hi. C kh nng c
Obama ln McCain u ng, nhng u ... v l.
n nm 1950, Kenneth Arrow (gii Nobel kinh t 1972) vit mt bi bo rt ni ting v cc
lut bu c [1], trong ng chng minh mt nh l nay ta gi l nh l bt kh thi Arrow1 .
nh l Arrow ni rng hm c ti l lut bu c duy nht c tnh duy l tuyt i. pht
biu nh l ny, ta nh ngha th no l c ti, v th no l duy l.
n gin (nhng khng mt tnh tng qut) ta gi s x hi c 3 chn la A-B-C cn xp
hng bng bu c (GD-YT-GT, hoc anh Ba-anh T-anh Su, hoc bnh m-sa-bia). Mi phiu
bu gm ba mc. mc th nht xp hng A hn B hoc B hn A. mc th hai xp hng
cp B, C; v mc th ba xp hng cp A v C. Nu anh no xp hng vng trn (A hn B, B
hn C, v C hn A) th anh y b chp cheng, khng cho bu. Ni cch khc, ta gi s tt c cc
phiu bu u hp l, ngha l khng phiu no xp hng vng trn.
1

Arrows impossibility theorem

Tp ch Epsilon, S 06, 12/2015


Sau khi c tt c cc phiu bu th x hi s da trn mt lut bu c c xp hng ton x hi
ca b ba A, B, v C, ngha l quyt nh xem x hi thch chn la no hn gia A v B, gia B
v C, v gia C v A. Lut bu c s phi tha mn mt s tin nht nh:
1. Tnh c lp ca cc chn la khng lin quan2 (IIA): vic x hi xp hng A hn B hay
B hn A th c lp vi vic mi ngi xp C cao thp th no.
2. Tnh nht tr (cn gi l hiu sut Pareto): nu mi ngi u thch A hn B th x hi
cng phi chn A hn B.
3. Tnh duy l: x hi khng th xp hng qun quanh theo vng trn (A hn B, B hn C, v
C hn A).
4. Khng c ti: xp hng ca x hi khng th lun ging ht nh xp hng ca mt anh
Tm Tng no m khng m xa g n phn cn li ca x hi.
Arrow chng minh rng khng c lut bu c no tha c bn iu kin trn, nu nh tt c cc
phiu c nhn u hp l. (Bi bo ca Arrow kh l di dng vn t. Vi mi gi thuyt, tin ,
ng li sang trit l v vi kt qu trc .) nh l ca Arrow tht s l mt nh l mang
tnh t hp, v c cc chng minh t hp ngn gn.
Phn m t trn khng c th v mt Ton hc ta chng minh. Mt k nng quan trng
m ngi lm Ton ng dng cn c l kh nng Ton hc ho i tng c nghin cu
trong ngnh ng dng. Bc Ton hc ho vn ny i khi quan trng khng km bc gii
quyt vn .
Mt m hnh Ton hc ca vn chn la x hi ny nh sau. Gi s c n phiu bu. Phiu bu
th i c i din bng mt b ba .xi ; yi ; zi / 2 f 1; 1g3 ; trong
(
xi D
(
yi D
(
zi D

1
1

nu phiu i chn A > B


nu phiu i chn B > A

nu phiu i chn B > C


nu phiu i chn C > B

nu phiu i chn C > A


nu phiu i chn A > C

1
1

(L do ta chn f 1; 1g thay v f0; 1g, ftrue; falseg s tr nn r rng hn di y.) nh ngha


hm NAE W f 1; 1g3 ! f0; 1g nh sau3 : NAE.a; b; c/ D 0 nu v ch nu a D b D c. Khi ,
phiu .xi ; yi ; zi / l phiu hp l nu NAE.xi ; yi ; zi / D 1. Vi n phiu bu th ta c ba vectors
x D .xi /niD1 , y D .yi /niD1 , z D .zi /niD1 2 f 1; 1gn . By gi ta m hnh xem bn tnh cht trn
pht biu v mt ton hc nh th no:
2
3

Independence of irrelevant alternatives


NAE l vit tt ca not all equal.

Tp ch Epsilon, S 06, 12/2015


1. Tnh cht IIA ni rng chn la ca x hi c th c kt bng ba hm s f; g; h W
f 1; 1gn ! f 1; 1g, trong
(
f .x/ D
(
g.y/ D
(
h.z/ D

1
1

nu x hi chn A > B
nu x hi chn B > A

nu x hi chn B > C
nu x hi chn C > B

nu x hi chn C > A
nu x hi chn A > C

1
1

2. Tnh nht tr ni l vi mi x 2 f 1; 1g, th f .x; : : : ; x/ D x, g.x; : : : ; x/ D x v


h.x; : : : ; x/ D x.
3. Tnh duy l ni l khng tn ti b phiu hp l .x; y; z/ m li cho ra chn la x hi
khng hp l f .x/ D g.y/ D h.z/.
4. Tnh khng c ti m hnh ho nh sau. Vi i 2 n, gi Dicti l hm c ti4 th i , tr
v phiu bu ca xi , ngha l Dicti .x/ D xi . Tnh khng c ti ni rng f; g; h Dicti ,
vi mi i 2 n.
T tnh nht tr v tnh duy l c th suy ra rng f  g  h. Ta lp lun nh sau. Xt b ba
x, y D x, v z D .f .x/;    ; f .x//. Do tnh nht tr ta c h.z/ D f .x/. Nh vy cho chn
la x hi c tnh duy l th g.y/ f .x/; ngha l g. x/ D f .x/ vi mi x. Tng t ta c
g. x/ D h.x/ vi mi x. Do f  h. Lp lun i xng dn n f  g  h.
Tm li, chn la ca ton x hi c th c m t bng mt hm s f W f 1; 1gn ! f 1; 1g.
V ta cn tm mt hm f sao cho
 Nu .x; y; z/ l b phiu hp l th NAE.f .x/; f .y/; f .z// D 1. (y l tnh duy l cha
chn la x hi.)
 f Dicti vi mi i 2 n. (Hm chn la x hi khng nn l hm c ti.)
nh l Arrow ni rng khng c hm f no tho hai tnh cht trn cng mt lc. (Lu rng
tt c cc hm c ti u l cc hm duy l!) Chng ta s chng minh nh l Arrow bng phn
tch Fourier ca cc hm nh phn. Chng minh ny l pht kin tuyt vi ca Gil Kalai [2]. Vic
nghin cu cc hm nh phn (cn gi l hm Bool5 ) l mt ti quan trng trong l thuyt
my tnh. N quan trng mt cch hin nhin v my tnh x l cc bt nh phn f0; 1g. Nhng
c th hn, mn gii tch cc hm nh phn c ng dng rt c th trong l tuyt tnh ton hin
i. Phng php gii tch nghin cu cc hm nh phn l ta tm cch vit chng thnh t
hp tuyn tnh ca cc hm n gin hn. lm c iu ny ta cn bin i Fourier ri rc
(DFT). m t DFT mt cch tng qut ta cn l thuyt biu din nhm. Do ta bt u vi
l thuyt biu din.
4
5

Dictator
Boolean functions

Tp ch Epsilon, S 06, 12/2015

2. S lc l tuyt biu din nhm


L thuyt biu din nhm cho php ta nghin cu cc nhm (trong i s tru tng) dng i s
tuyn tnh. (i s tuyn tnh vn tu!) Bng cch ny, mt s vn , c tnh ca cc nhm
tru tng c th c gii quyt v tm hiu dng cc cng c ca i s tuyn tnh. T gc
nhn t hp, quyn Nhm i xng ca Bruce Sagan rt th v [3].
Trc ht ta nh ngha biu din ma trn6 ca mt nhm. Mt biu din ma trn n chiu ca
mt nhm G l mt php ng cu7  W G ! GLn .F/ trong F l mt trng i s, v d nh
trng s phc, cn GLn .F/ l nhm tuyn tnh tng qut8 bc n trn trng F. Tng qut hn,
ta khng nht thit phi biu din nhm bng cc ma trn. Gi V l mt khng gian vector c s
chiu hu hn. Gi GL.V / l nhm cc bin i tuyn tnh trn V (ngha l GL.V / l tp hp
cc nh x tuyn tnh kh nghch). Mt php biu din ca nhm G trn khng gian V l php
ng cu
 W G ! GL.V /:
Ni cch khc, mt biu din l mt lut gn: ta gn cho mi phn t g ca nhm G mt nh x
.g/ 2 GL.V / sao cho php gn ny tng thch vi cc hot ng ca nhm G. Nu c mt
b vector c s ca V th ta c th d dng chuyn  thnh mt php biu din ma trn. (Trong
trng hp , mi phn t g 2 G s c tng ng mt ma trn kh nghch .g/.)
V d 1. Xt nhm G D Zn . nh x  W G ! GLm .R/ gn mi phn t k 2 f0; : : : ; n
ma trn .k/ kh nghch m  m, sao cho, vi mi j; k 2 Zn , ta c

1g mt

.j C k/ D .j /.k/:


(y l nh ngha ca php ng cu.) Do , .0/ phi l ma trn n v. V, vi mi k ta c
.k/ D .1/k . Ngha l, sau khi chn ma trn kh nghch .1/ sao cho .1/n D .0/ D Im
(nu c) th phn cn li ca  l hon ton xc nh.
Trong phn cn li ca bi ny, n gin vn ta ch xt V l mt khng gian tuyn tnh m
chiu trn trng s phc (hiu l V D Cm ). Nh hu ht cc i tng tru tng khc trong
ton hc, ta tm cch chia mt php biu din nhm thnh cc thnh phn nh hn, cho n khi
ti gin. T , ta c th nghin cu mt cu trc ln bng cc cu trc ti gin, vi hy vng l
nhiu cu hi d tr li hn.
Mt khng gian con W ca V c gi l G-bt bin nu cc phn t ga G tng ng vi cc
nh x t W vo W . C th hn, nu vi mi w 2 W v g 2 G ta c .g/w 2 W th W c
gi l G-bt bin. Tt nhin, nu W l G-bt bin th nh x thu hp ca  trn W cng l mt
biu din ca G.
E Nu ngoi hai khng gian tm thng ny ra, V
Hai khng gian bt bin tm thng l V v f0g.
khng cn khng gian con G-bt bin no khc, th  c gi l mt biu din ti gin ca G.
Nu V l tng trc tip9 ca hai khng gian con G-bt bin W1 v W2 , k hiu l V D W1 W2 ,
th php biu din  trn V l tng trc tip ca 1 v 2 , vit l  D 1 2 , trong 1 v 2
6

Matrix representation
Homomorphism
8
General linear group
9
Direct sum
7

10

Tp ch Epsilon, S 06, 12/2015


l cc thu hp ca  trn W1 v W2 , theo th t. Nu  l php biu din ti gin  khng phi l
E
tng trc tip ca cc php biu din khc, ngoi tr ci tng tm thng V fOg.
V d 2. Xt nhm G D Z2n , v nh x  W G ! GL2 .C/ nh ngha nh sau:


1 0
.0/ D
D I2
0 1
  i=n

e
0
.1/ D
0
e  i=n
.k/ D .1/k ; k 2 Z2n :
(Lu rng i l s phc v .1/2n D .0/ nh .) Trong v d ny th V D C2 . Xt
 

 

a
0
W1 D
W a2C
W2 D
W b2C :
0
b
 
a
D thy W1 l mt khng gian G-bt bin, ti v vi mi k 2 Z2n v mi w D
2 W1 , ta c
0
 k i=n
    k i=n 
e
0
a
ae
.k/w D
D
2 W1 :
k i=n
0
0
0
e
Tng t, W2 cng bt bin. Ngoi ra C2 D W1 W2 v
     
a
a
0
D
C
b
0
b
 
a
vi mi vector
2 C2 . V th,  khng phi l biu din ti gin. Hai thu hp 1 v 2 ca 
b
nh ngha nh sau:




1 0
0 0
1 .0/ D
2 .0/ D
0 0
0 1
  i k=n 


0
0
e
0
1 .k/ D
2 .k/ D
0
0
0
e  i k=n
l cc biu bin ti gin c s chiu bng 1.
n y ta c kin thc pht biu mt nh l cc k n gin v quan trng ca l thuyt
biu din: nh l Maschke. nh l ny tng t nh nh l phn tch ra tha s nguyn t,
hoc nh l c bn ca i s rng cc a thc u l tch ca n thc tuyn tnh. Heinrich
Maschke (18531908) l mt nh Ton hc ngi c. ng tng theo hc cc ngi khng l
Weierstrass, Kummer v Kronecker. Tt nghip nm 1880, khng tm c v tr c, ng di
c sang M, nhn c mt v tr trong khoa Ton mi m ca i Hc Chicago, ni mt nh
Ton hc lng danh ca Vit Nam hin nay ang lm vic; anh cng l mt chuyn gia v l
thuyt biu din.
nh l 2.1 (nh l Maschke). Bt k php biu din (phc) no trn mt nhm hu hn G u
l tng ca cc php biu din ti gin.
11

Tp ch Epsilon, S 06, 12/2015


Chng minh. Ta ch cn chng minh rng, nu  cha ti gin th  D 1 2 v 1 ; 2 c s
chiu nh hn. Gi s W  V l mt khng gian con G-bt bin khng tm thng. Ta chng
minh rng tn ti W ? sao cho W W ? D V v W ? cng G-bt bin. Gi f; g l mt dng
Hermit10 tu trn khng gian V , d chng minh rng dng song tuyn sau y sau y
hv; wi D

1 X
f.g/v; .g/wg
jGj g2G

l G-bt bin: hv; wi D h.g/v; .g/wi, 8g 2 G; v; w 2 V . T , khng gian trc giao W ?


ca W nh ngha theo dng song tuyn ny11 cng l G-bt bin.
Nh vy, ta c th nghin cu cc php biu din dng cc php biu din n gin hn mt
cht. Tuy nhin, mt php biu din vn l mt i tng rt phc tp m t (n l mt php
ng cu tha mn mt s tnh cht i s, hoc cng c th xem n l mt ma trn nu ta chn
trc mt h c s trn khng gian V ). Thm ch, c bao nhiu php biu din (khng ng
cu12 vi nhau) ta cng khng bit. C th c v hn cc php biu din khng? Lm th no
phn loi chng?
phn loi cc php biu din, c mt cch loi b a s thng tin v php biu din, ch
gi li mt vi con s! Cc con s ny cha rt nhiu thng tin v php biu din, v ta c th
dng chng phn loi cc php biu din. Kt qu ny l mt trong nhng nh l p nht
trong i s.
Cc con s k diu ny c cha trong mt hm gi l hm c trng13 ca php biu din.
Hm c trng  ca php biu din  trn nhm G l mt hm  W G ! C nh ngha nh sau
.g/ D trace..g//:
(Nh rng .g/ l mt ton t tuyn tnh kh nghch trn khng gian phc V , ci vt14 trace..g//
ca .g/ l tng cc tr c trng ca n. Hm c trng  cng l mt vector m cc ta
c nh ch s bi cc thnh vin ca nhm G.
V d 3. Li xt nhm G D Zn v mt biu din  W G ! GLm .C/. Gi  l hm c trng ca
biu din ny, th
.0/ D trace..0// D trace.Im / D m:
V vi mi k 2 Zn ta c
k

.k/ D trace..k// D trace..1/ / D

m
X

ki

i D1

trong i l cc tr c trng ca .1/. Xt vector c trng v 2 Cm tng ng vi i , th


.1/v D i v. Do , .1/n v D ni v. M .1/n D Im . Do , i l mt trong cc cn bc n ca
1.
10

Hermitian form
W ? WD fv j hv; wi D 0; 8w 2 W g
12
Isomorphic
13
Character
14
Trace
11

12

Tp ch Epsilon, S 06, 12/2015


Tng qut hn, nh ngha s chiu ca mt hm c trng l s chiu ca khng gian V ca
php biu din. nh ngha mt tch v hng Hermit gia cc hm c trng nh sau:
h; 0 i D

1 X
.g/0 .g/:
jGj g2G

(2.1)

(a l lin hp phc ca a.) Hm c trng ca php biu din cha cc k nhiu thng tin v
php biu din. Sau y l vi kt lun quan trng:
 .1/ chnh l s chiu ca php biu din. (Lu rng 1 l phn t n v ca nhm G.
Nu G D Zn th 1 l s nguyn 0.)
 .g/ D .hgh 1 /, vi mi phn t g; h 2 G, ngha l hm c trng c gi tr nh nhau
trn mi lp lin hp15 ca nhm.
 .g 1 / D .g/
 Hm c trng ca  0 l tng  C 0 ca cc hm c trng thnh phn ; 0 .
 Gi N D jGj, v 1 ; 2 ; : : : l cc i din ca cc lp ng cu ca cc php biu din
ti gin trn G, v gi i l hm c trng ca i . Ta c:
Cc vectors i vung gc vi nhau v c chiu di n v. Gi c l tng s cc lp
lin hp ca nhm. Gi C l khng gian vector ca cc hm f W G ! C sao cho
f c gi tr nh nhau trn mi lp lin hp ca G. Ta c, cc hm c trng i to
thnh mt h c s trc chun ca C. (Ta s dng tnh cht ny ni thm v bin
i Fourier ri rc trn cc nhm Abel trong mc ti.)
Tng s cc lp ng cu ca cc php biu din ti gin bng vi tng s cc lp
lin hp ca nhm G. Gi r l tng s ny.
Gi di l s chiu ca i , ta c di chia ht cho N , v
N D d12 C    C dr2 :
 Mt hm c trng bt k ca nhm u c th biu din (theo mt cch duy nht) thnh
t hp tuyn tnh ca cc hm c trng ti gin.
 Hai php biu din c hm c trng ging nhau th ng cu vi nhau
 Mt hm c trng  l ti gin nu v ch nu n c chiu di n v (ngha l h; i D 1)
 Nu G l mt nhm Abel th cc biu din ti gin ca n u c s chiu bng 1.
C mt h qu tuyt p ca l thuyt biu din nhm khi G l nhm cc hon v ca n phn t.
Gi f  l s cc standard Young tableaux dng . Ta c:
X
.f  /2 D n
`n

nh l ny cng c th chng minh bng gii thut Robinson-Schensted.


15

Conjugacy class

13

Tp ch Epsilon, S 06, 12/2015

3. Bin i Fourier ri rc
Ti hc v bin i Fourier ri rc (DFT) ln u tin vo khong nm 1993. Hc xong thy
rt hoang mang, theo kiu: nu ly vector ny, tnh ton th ny, th ra cc h s th kia, nhng
khng hiu tng nm sau cc cng thc . Sau khm ph ra l DFT chng qua l mt php
thay i c s trong khng gian tuyn tnh. T thy mi th r rng, d hiu hn ra.

3.1. Biu din nhm Abel hu hn


Cc biu din ti gin ca mt nhm Abel bt k u l cc biu din vi s chiu bng mt.
Nu nhm c n phn t th c n hm c trng trc giao. Nhm tun ton l mt nhm Abel.
Nhm tun hon Zn c ng n hm c trng ti gin a ; a 2 Zn . Mi hm c trng a l mt
vector trong trng vector phc Cn , nh ngha l a .b/ D !nab , trong !n D e 2 i=n l cn
nguyn thu. Cc hm c trng ny l mt h trc chun theo tch Hermit (2.1):
ha ; b i D

1 X
a .c/b .c/ D ab :
n c2Z
n

(ab D 1 nu a D b v 0 nu a b.)
nh l c bn ca cc nhm Abel hu hn ni rng cc nhm Abel hu hn G u c th vit
di dng tng trc tip ca cc nhm tun hon: G Zm1    Zmk . Cc biu din ti gin
ca nhm G l tng s ca cc biu din ti gin ca cc nhm tun hon Zmi . Cc hm c
trng ti gin ca nhm G l tch tng s ca cc hm c trng ti gin ca cc nhm tun hon
Zmi . Vi mi phn t a D .a1 ;    ; ak / 2 Zm1    Zmk , ta c mt hm c trng ti gin
a ca nhm G nh ngha nh sau: vi mt ta " b D .b1 ;    ; bk / 2 Zm1    Zmk th
a .b/ D

k
Y

a i bi
a 1 b1
ak bk
!m
D !m
   !m
:
1
k
i

i D1

Mt trng hp c bit ca nhm Abel hu hn rt quan trng trong bi ny v trong khoa


hc My Tnh ni chung l l G D Zn2 D f0; 1gn . Mi phn t ca G l mt nh ca khi lp
phng n-chiu, l mt php gn s tht16 vo n bin nh phn, hoc l mt tp con S  n
trong S l tp cc ta bng 1 ca phn t. y, nhm G c N D 2n phn t, v v th N
hm c trng. Do !2 D 1, vi mi cp a; b 2 Zn2 ta c a .b/ D . 1/ab .
Thay v dng a; b nh s cc hm c trng v cc ta ca chng, ta c th dng cc tp
con A; B ca n nh ch s, trong A D fi j ai D 1g, v B D fi j bi D 1g. Theo cch
ny, b cc hm c trng c th nh ngha
bng A .B/ D . 1/jA\Bj . Cn nu chng ta dng
P
b 2 Zn2 lm tham s th A .b/ D . 1/ i2A bi . Bn nn lm quen vi vic chuyn qua li gia
cc tp con v cc vectors ca Zn2 .
L do chnh chng ta quan tm n cc hm c trng ti gin ca nhm Zn2 l nh sau. Ta cn
nghin cu cc hm nh phn gm n bin nh phn kiu nh f W f0; 1gn ! f0; 1g. Mi hm loi
ny c th xem l mt vector trong khng gian f0; 1gN . D nhin, chng cng l cc vectors trong
khng gian RN v CN . Nh phn tch trn, cc hm c trng ti gin l mt c s trc
16

Truth assignment

14

Tp ch Epsilon, S 06, 12/2015


chun ca khng gian CN . V th, mt hm nh phn n bin bt k, nu vit thnh mt vector
trong khng gian CN , u l t hp tuyn tnh ca cc hm c trng ti gin.
Thay v lm vic trn khng gian vector CN , mt cch tng ng chng ta cng c th lm
vic trn khng gian (tuyn tnh) ca hm s f W f0; 1gn ! C. Hm f W f0; 1gn ! C bt k
u c th biu din di dng
X
X
P
f .y/ D
fOS S .y/ D
fOS . 1/ i 2S yi :
S n

S n

ci m yi trn s m th hi kh chu. Chng ta i bin. t xi D . 1/yi . Ngha l nu


yi D 0 (FALSE) th xi D 1, cn yi D 1 (TRUE) th xi D 1. Th ta c pht biu sau y:
B 1. Mi hm f W f 1; 1gn ! C u c th vit di dng
X
f .x/ D
fOS S .x/;
S n

trong (lm dng k hiu mt cht) S W f 1; 1gn ! f 1; 1g l mt hm n thc17


Y
S .x/ D S .x1 ;    ; xn / D
xi :
i 2S

m hm S by gi gi l h c s n thc ca cc hm f W f 1; 1gn ! C. Nh rng ci h


c s n thc ny l mt h c s trc chun ca khng gian cc hm f W f 1; 1gn ! C. Trong
, tch v hng" ca hai hm f; g bt k c nh ngha l
i
h
1 X
hf; gi D n
f .x/g.x/ D E f .x/g.x/ ;
x
2
n
x2f 1;1g

trong tr k vng v phi tnh trn phn b u ca cc vectors x 2 f 1; 1gn . Chng ta s


thy rng hiu tch v hng ca hai hm l tr k vng ca tch nh trn rt hu dng v sau.

3.2. Bin i Fourier ri rc


tng chnh ca bin i Fourier ri rc ch l mt pht biu c bn ca i s tuyn tnh: cc
vector trong mt khng gian vector u l t hp tuyn tnh ca mt h c s bt k ca khng
gian . (Xem thm bi ca Terry Tao gii thiu v bin i Fourier ni chung, v quyn sch
tuyt vi ca anh V H Vn v Terry Tao c cc ng dng ca gii tch ng iu trong ton t
hp [4]).
Trong ng cnh ca chng ta, mi hm f W f 1; 1gn ! R u l t hp tuyn tnh ca cc hm
n thc:
X
f .x/ D
fOS S .x/;
S n

T hp ny l duy nht. Cc h s fOS D hf; S i gi l cc h s Fourier ca f . Chng l cc s


thc v f v S l cc vectors thc. T gi tr i chng ta c thm lm vic lun trn khng
gian RN thay v CN v khng cn ci lin hp khi tnh tch v hng ca hai vectors na. H c
s n thc S cng c gi l h c s Fourier.
Hai ng thc c bn nht ca bin i Fourier l
17

Monomial

15

Tp ch Epsilon, S 06, 12/2015


 ng thc Plancherel
E f .x/g.x/ D hf; gi D
x

fOS gO S D N hfO; gi
O

Sn

 v trng hp c bit l ng thc Paserval


X
 2 
fOS2 :
E f .x/ D hf; f i D
x

S n

D chng minh hai ng thc trn t nh ngha v tnh trc chun ca cc S . (i khi, m
bo tnh i xng ngi ta nh ngha tch v hng nh trn nhng chia cho cn bc hai ca N .)

4. Lut bu c v bin i Fourier cho cc hm nh phn


4.1. Lut bu c ni chung
Trong trng hp hm nh phn f W f 1; 1gn ! f 1; 1g th f 2 .x/ D 1 vi mi x, v th ng
thc Parseval ni rng
X
fO2 D 1:
S

S n

Mt hm nh phn nh mt lut bu c. C n phiu bu xi cho hai ng c vin 1 v 1. Hm


f tr v ngi thng c. Sau y l mt s hm (lut) bu c hay thy trn thc t:
 Majn l hm bu a s, ch nh ngha vi n l, tr v 1 nu a s cc phiu l 1, v tr
v 1 nu a s cc phiu l 1.
 Dicti l hm c ti ( nh ngha), tr v phiu bu ca xi , ngha l Dicti .x/ D xi .
 Const1 v Const 1 l cc hm hng s (hay hm ng c, dn bu"), lun tr v gi tr
ng c 1 hoc 1.
Ta cng c th nh ngha mt s hm khc nh hm chn l, hm electoral college (nh trong
lut bu c ca M), vn vn. Xem bi ny ca Ryan ODonnell thm mt s v d.
Vi mt lut bu c nht nh, chng ta mun bit nhiu thuc tnh ca n.
 N c thin v khng? Thin v y c hiu nh sau, nu ta ly mt b n phiu bu
ngu nhin th xc sut m kt qu l 1 hoc 1 khc nhau c no. Mt lut bu l cng
bng nu hai xc sut ny bng nhau. Do , ta nh ngha s thin v ca hm f bng
P f .x/ D 1
x

P f .x/ D
x

1 D E f .x/ :
x

n y th ta thy phn tch Fourier c li th no. Do hm ; D 1, thin v ca f l


E f .x/ D E f .x/; .x/ D fO; :
x

thin v ca f chnh bng vi h s Fourier th nht! D thy rng cc hm ng


c/dn bu c thin v l 1. Cc hm c ti v hm a s c thin v bng 0.
16

Tp ch Epsilon, S 06, 12/2015


 nh hng ca mt phiu no ra sao? Nu Tm Tng i phiu t 1 sang 1 th kt
qu b i th no? Vi b phiu x, gi xi l b phiu m ta i phiu xi li. Th tm nh
hng (Influence) ca phiu th i trn kt qu c nh ngha l
Infi .f / WD Pf .x/ f .xi /:
x

Trong l thuyt chn la x hi th tm nh hng ny cn c gi l ch s sc mnh


Banzhaf hoc ch s Banzhaf-Penrose index. Ch s ny c mt t nh hng trong mt
vi phin ta v bu c.
Bi tp 1. Chng minh rng Infi .f / D

fOS2 :

i 2S

D thy rng tm nh hng ca cc hm ng c l 0, tm nh hng ca hm c ti l


0 cho tt c tr anh c ti c nh hng bng 1. Tm nh hng ca hm q
a s th mt
cng hn mt cht. Dng xp x Stirling ta cng tnh c n bng khong

2
.
n

 nh hng ca nhiu ra sao? Khi ghi li c triu phiu bu th xc sut m mt phiu b


ghi sai khng b qua c. Gi xc sut ny l  chng hn. Gi s ta ly mt b phiu
bu x hon ton ngu nhin. Gi y l b phiu t c bng cch lt mi phiu xi vi xc
sut . D thy, vi mi i ,
Exi yi D 1 2:
x

Do cp .x; y/ c gi l .1
c nh ngha l
Stab1

2 .f

/D

x;y

2/-correlated. n nh nhiu ca f ti .1

f .x/f .y/ D Ef .x/ D f .y/

Ef .x/ f .y/:

.1 2/ cor

2/

Ngc li:
Ef .x/ D f .y/ D
x

1 1
C Stab1
2 2

2 .f

/:

Bi tp 2. Chng minh rng:


Stab1

2 .f

/D

.1

2/jSj fOS2 :

S n

n nh nhiu ca cc hm ng c l 1, ca hm c ti th i l 1 2. n nh
nhiu ca hm a s l th v nht. C th chng minh c iu sau y:
lim Stab1

n!1

2 .Majn /

D1

2
arccos.1


2/:

p
Nu ta dng xp x arccos.1p 2/  2  (kh tt khi  nh) th ta thy rng ci nhiu 
dn n xc sut khong 2  l kt qu bu c b thay i.
17

Tp ch Epsilon, S 06, 12/2015

4.2. Chng minh nh l Arrow


Gi s tn ti hm f sao cho, vi bt k b phiu hp l no .x; y; z/ no, ci chn la x hi
.f .x/; f .y/; f .z// cng duy l. Ta s chng minh rng f phi l hm c ti.
Vi mi c nhn i, chn b ba .xi ; yi ; zi / ngu nhin t mt trong 6 b ba hp l .1; 1; 1/,
.1; 1; 1/, .1; 1; 1/, . 1; 1; 1/, . 1; 1; 1/, v . 1; 1; 1/. Ta s c mt b ba vectors
.x; y; z/ hp l. Xc sut m .f .x/; f .y/; f .z// l duy l phi bng 1.
Khai trin Fourier ca hm NAE l
NAE.a; b; c/ D

3
4

1
ab
4

1
bc
4

1
ca:
4

Nh vy, xc sut m .f .x/; f .y/; f .z// l duy l s bng


E NAE.f .x/; f .y/; f .z/ D

x;y;z

3
4

1
E f .x/f .y/
4

1
E f .y/f .z/
4

1
E f .z/f .x/ :
4

Do x; y; z c vai tr nh nhau, ta kt lun


E NAE.f .x/; f .y/; f .z/ D

x;y;z

3
4

3
E f .x/f .y/ :
4

Nh rng tr k vng c tnh t cch ly cc b ba duy l x; y; z nh m t trn. T d


thy rng x; y l mt cp . 1=3/-correlated. Do
E f .x/f .y/ D Stab

1=3 .f

/D

. 1=3/jS j fOS2 :

S n

P
P
P
cho gn, ta nh ngha Wk .f / D jSjDkj fOS2 . Nh rng S fOS2 D 1, do k Wk .f / D 1.
Do , xc sut m .f .x/; f .y/; f .z// l duy l bng
3
4

3X
3
. 1=3/k Wk .f / D
4
4
kD0

3
1
W0 .f / C W1 .f /
4
4

1
W2 .f / C   
36

Xc sut ny ch c th bng 1 nu W1 .f / D 1 v Wk .f / D 0 vi mi k 1. Nhng


W1 .f / D 1 nu v ch nu f D Dicti hoc f D Dicti vi i no . Nhng f phi tha tnh
nht tr, do f D Dicti . V l tnh duy l ca s c ti.
Chng minh nh l Arrow bng phng php ny khng ch cho vui. Chng minh c ca
Arrow khng cho chng ta bit xc sut s ph l ca chn la x hi l bao nhiu. Phn tch
Fourier cho chng ta bit, nu ta dng hm a s, khi n tin n v cng th xc sut c chn la
x hi duy l l


3 3
2
1
arccos. 1=3/  :912:
4 4

Con s ny gi l s Guilbaud. Chng ta khng nhng bit nghch l Condorcet c th xy ra m
cn bit c xc sut ca n (gi s ai cng chn phiu bu hp l ngu nhin, khng suy ngh).
18

Tp ch Epsilon, S 06, 12/2015

Ti liu tham kho


[1] ARROW, K. J. A Difficulty in the Concept of Social Welfare. Journal of Political Economy
58 (1950), 328.
[2] KALAI, G. A Fourier-theoretic perspective on the Condorcet paradox and Arrows theorem.
Adv. in Appl. Math. 29, 3 (2002), 412426.
[3] SAGAN, B. E. The symmetric group, second ed., vol. 203 of Graduate Texts in Mathematics.
Springer-Verlag, New York, 2001. Representations, combinatorial algorithms, and symmetric
functions.
[4] TAO, T., AND VU, V. H. Additive combinatorics, vol. 105 of Cambridge Studies in Advanced Mathematics. Cambridge University Press, Cambridge, 2010. Paperback edition [of
MR2289012].

19

Tp ch Epsilon, S 06, 12/2015

20

XP X DIOPHANTINE VI O
NH L KHINTCHINE
L Ngc Tu
(i hc Brandeis, Massachusetts, M)

1. Gii thiu
Trong phn 1 [5], chng ta c c cu tr li cho cu hi v kh nng xp x s thc bi s
hu t qua nh l Dirichlet:
p
nh l 1.1 (Dirichlet 1948). Vi mi s v t x 2 R X Q, tn ti v s s hu t 2 Q sao cho:
q

x p < 1 ;
(1.1)

q q2
v m rng kt qu ny ra khng gian vc t Rn trong phn 2 [6]. Mt khc, nh l Dirichlet
c chng minh l ti u qua s tn ti ca cc s/vc t xp x km. Ni mt cch khc, vi
p
mi s v t x, ta c th tm c v s nghim hu t cho bt ng thc (1.1). Tuy nhin,
q
nh l Dirichlet xt chung tt c cc s v t, nu nh xt ring bit tng s v t x th hm xp
1
x 2 c th khng phi l ti u. Chng hn nh cc s Liouville L c nh ngha nh sau:
q
mt s v t x c gi l mt s Liouville x 2 L nu nh vi mi n  1, tn ti mt s hu
p
t 2 Q sao cho:
q

x p < 1 :
(1.2)

q qn
Vo nm 1844, nh ton hc Joseph Liouville chng minh rng tp L khng rng, v l v d
u tin v s siu vit (transcendental numbers). Lu rng nu nh x 2 L l mt s Liouville
p
th vi mi n  1, bt ng thc (1.2) c v s nghim 2 Q.
q
Trong phn ny, chng ta s a thm yu t o vo vn v kh nng xp x s thc bi s
hu t. Ni mt cch c th hn, nu nh ta thay Vi mi s" trong nh l Dirichlet bng Vi
1
hu ht mi s (theo o Lebesgue)" th ta c th thay hm s xp x 2 bng hm s no?
q
Cu hi ny c A. Y. Khintchine tr li hon ton vo nm 1924 [1] v m rng ra cho
khng gian vc t Rn vo nm 1926 [2]. Kt qu ny sau y c A. V. Groshev chng minh
cho khng gian ma trn Mm;n .R/ vo nm 1938.
gii thiu kt qu ca Khintchine, chng ta cn mt s k hiu sau: Hm s c gi l
mt hm xp x nu nh W 1; 1/ ! .0; 1/ l mt hm khng tng. Ta gi s thc x 2 R l
21

Tp ch Epsilon, S 06, 12/2015


mt s

-xp x c ( -approximable) nu nh tn ti v s s hu t

cho:

Tp cc s

p
<
q

p
2 Q vi q > 0 sao
q

.q/
:
q

(1.3)

-xp x c c k hiu l WA. /.

nh l Dirichlet trn R c th c vit li theo k hiu mi ny nh sau:


Nu nh

.q/ D

1
th WA. / D R:
q

nh l Khintchine cho tp s thc R c pht biu nh sau:


nh l 1.2 (Khintchine 1924). Gi s nh l mt hm xp x sao cho q .q/ l mt hm
khng tng, v k hiu .E/ l o Lebesgue ca tp E.
(i) Nu nh chui

1
X

.n/ hi t th .WA. // D 0.

nD1

(ii) Nu nh chui

1
X

.n/ phn k th .R X WA. // D 0.

nD1

Ni mt cch khc, nu nh chui

1
X

hi t th vi hu ht tt c cc s thc, bt ng thc (1.3)

nD1

c v s nghim hu t; cn nu nh chui ny phn k, vi hu ht tt c cc s thc, bt ng


thc (1.3) ch c hu hn nghim hu t.
Lu 1.3. Cc kt qu dng nh nh l 1.2 trong l thuyt xp x Diophantine thng c
gi l cc nh lut 0-1 .
Lu 1.4. Mt s h qu trc tip th v ca nh l Khintchine nh sau:
(i) Tp cc s xp x km BA c o Lebesgue bng 0.
(ii) Vi hu ht mi s thc x, bt phng trnh:

p
1
x
<

2
q
q log q
c v s nghim hu t.
(iii) Vi  > 0 bt k v vi hu ht mi s thc x, bt phng trnh:

p
1
x
<

q
q 2 .log q/1C
ch c hu hn nghim hu t.
Trong phn cn li ca bi ny, chng ta s gii thiu tm tt v o Lebesgue, v cng c
chnh chng minh nh l 1.2: B Borel-Cantelli trong l thuyt xc sut v lin phn s.
22

Tp ch Epsilon, S 06, 12/2015

2. Phn hi t ca nh l Khintchine
C rt nhiu ti liu tham kho cho phn ny. y chng ti ch gii thiu mt s nh ngha
v tnh cht c bn dn n B Borel-Cantelli.

2.1. o Lebesgue
o Lebesgue trn khng gian Rn l m rng ca khi nim di (n D 1), din tch (n D 2),
v th tch (n  3). Trn R, cc on thng .a; b/ l thc o c bn o di ca mt tp
hp, v o on .a; b/ c nh ngha bi: ..a; b// WD b a. o (ngoi) ca mt tp
E  R bt k c xy dng bng cch s dng mt s m c cc on thng ph ln tp
E vi tng di cng nh cng tt:
(1
)
1
X
[
.E/ WD inf
.Ii / W E 
Ii vi Ii l cc on thng :
i D1

iD1

Mt tp con E  R c gi l mt tp o c nu nh vi mi A  R:
.A/ D .A \ E/ C .A \ .R X E//:
Tp cc tp o c to thnh mt -i s tha mn cc tnh cht sau:
(1) Tp rng ; v R l cc tp o c.
( 2) Nu nh E l mt tp o c th phn b R X E cng l mt tp o c.
( 3) Nu nh A1 ; A2 ; ::: l cc tp o c th

1
[

Ai cng l mt tp o c.

i D1

o Lebesgue trn cc tp o c tha mn cc tnh cht sau:


(M0) .;/ D 0.
(M1) Nu nh A  B l 2 tp o c th .A/  .B/.
(M2) Nu nh A1 ; A2 ; ::: l mt dy cc tp o c ri nhau tng cp th:
!
1
1
[
X

Ai D
.Ai /:
i D1

i D1

Lu 2.1. p dng Tin chn (Axiom of Choice), ta c th xy dng c tp con ca R


khng o c chng hn nh tp Vitaly hoc tp Bernstein. Tp khng o c dn n mt s
nghch l nh nghch l Banach-Tarski. Mt khc, nu nh ta b Tin chn, Solovay [4] chng
minh rng tn ti mt m hnh ca tp s thc m trong , mi tp con u l tp o c.
Lu 2.2. Ta ni mt tnh cht P tha mn vi hu ht mi s x, nu nh tp fx 2 R W
x khng tha mn tnh cht P g c o Lebesgue bng 0.
Bi tp 2.3. Chng minh rng .a; b/ D b

a.
23

Tp ch Epsilon, S 06, 12/2015


Bi tp 2.4. Chng minh rng nu E  R l mt tp n c th .E/ D 0.
Bi tp 2.5. Tm o Lebesgue ca tp Cantor:
)
(
1
X
ai
; ai 2 f0; 2g :
C WD x 2 0; 1 W x D
3i
i D1
Bi tp 2.6. Tm o Lebesgue ca tp Liouville L.

2.2. Tp limsup v B Borel-Cantelli


Gi X l mt khng gian bt k. Mt h B cc tp con ca X tha mn cc tnh cht tng
t nh (1)(3) c gi l mt  -i s ca cc tp o c trn X . Mt hm khng m
 W B ! R0 tha mn cc tnh cht nh M(0)M(2) c gi l mt o trn X . B 3
.X; B; / c gi l mt khng gian o.
.E/
cho 0 .X/ D 1.
.X/
Khng gian o .X; B; / vi .X/ D 1 c gi l mt khng gian xc sut. Trong l thuyt
xc sut, cc tp o c E 2 B tng ng vi cc s kin, v o .E/ ca E tng ng vi
xc sut s kin E xy ra.
Nu nh 0 < .X/ < 1, ta c th thay o  bng o 0 .E/ D

Vi mt dy cc tp con E1 ; E2 ; ::: ca X , ta nh ngha tp limsup ca dy ny nh sau:


lim sup En WD
n!1

1 [
1
\


Ek D x 2 X W c v s is sao cho x 2 Ei :

nD1 kDn

Ni cch khc, nu nh E D lim sup En l s kin "c v s s kin En xy ra". B


n!1

BorelCantelli c th c pht biu nh sau:


B 2.7 (B Borel-Cantelli). Cho .X; B; / l mt khng gian xc sut, v E1 ; E2 ; :::; 2 B
1
X
l cc s kin. Nu nh chui
.En / hi t, th:
nD1



 lim sup En D 0:
n!1

Bi tp 2.8. Chng minh B 2.7.


Bi tp 2.9. Tm phn v d cho mnh o ca B 2.7.
Tr li vi nh l Khintchine, p dng c B Borel-Cantelli, ta cn c mt khng gian
xc sut, v biu din tp c s -xp x c WA. / di dng mt tp limsup.
Bi tp 2.10. Chng minh rng x 2 WA. / khi v ch khi x C 1 2 WA. /.
24

Tp ch Epsilon, S 06, 12/2015


p dng bi tp trn, chng minh nh l Khintchine, ta ch cn tp trung vo cc s nm
trong on 0; 1/. Gii hn o Lebesgue trn on 0; 1/ s cho ta mt khng gian xc sut.
t


En D x 2 R W x

p
<
n


.n/
;
n

ta c th biu din tp WA. / di dng mt tp limsup nh sau:


WA. / D lim sup En :
n!1

Bi tp 2.11. Tm o Lebesgue ca En \ 0; 1 (tp ny ch bao gm hu hn cc on thng).


Bi tp 2.12. Chng minh rng nu nh chui
t.

1
X

.n/ hi t th chui

nD1

1
X

.En \ 0; 1// hi

nD1

p dng Bi tp 2.10 v B Borel-Cantelli, ta c c phn hi t ca nh l Khintchine.

3. Phn phn k ca nh l Khintchine


chng minh phn phn k ca nh l 1.2, chng ta s s dng cng c tt nht cho xp x
m ta c c trn tp s thc: lin phn s.

3.1. Mt s iu c bn ca Lin phn s


Nhc li trong phn 1 [5], chng ta gi mt lin phn s hu hn c di .n C 1/ l mt biu
thc c dng:
1
ao I a1 ; :::; an WD a0 C
1
a1 C
1
:::
C
an
vi mt dy s thc hu hn a0 2 R; a1 ; :::; an 2 R X f0g.
Khi a0 2 Z; a1 ; :::; an 2 N, ta gi biu thc trn l mt lin phn s n hu hn. Vi mi dy
a0 2 Z; a1 ; a2 ; ::: 2 N, dy cc lin phn s n hu hn a0 I a1 ; :::; an l mt dy hi t khi
n ! 1, v ta k hiu gii hn ny l:
a0 I a1 ; ::: WD lim a0 I a1 ; :::; an :
n!1

Mi s v t x 2 R X Q c th c biu din duy nht di dng mt lin phn s n v hn:


x D a0 I a1 ; ::::
Vi cch biu din nh trn, phn s hi t th n ca x l:
pn
pn .x/
D a0 I a1 ; :::; an ;
D
qn
qn .x/
25

Tp ch Epsilon, S 06, 12/2015


vi

pn
dng ti gin.
qn

t xnC1 D anC1 I anC2 ; :::, chng ta c c cng thc sau:


B 3.1. Vi mi n  0:
xD

xnC1 pn C pn
xnC1 qn C qn

Tuy trng c v phc tp, nhng biu din s thc bng lin phn s c tnh cht sau ging vi
biu din cc s trong h thp phn:
B 3.2. Gi s nh x D a0 I a1 ; ::: v y D b0 I b1 ; :::.
pn
pn C pn 1
<y<
khi v ch khi b0 D a0 ; :::; bn D an :
qn
qn C qn 1
Hay ni mt cch khc, tp cc s c biu din lin phn s c phn u c nh
J.a0 ; a1 ; :::; an / WD fy D a0 I a1 ; :::; an ; r W r 2 R; r  1g
l mt on thng:



pn pn C pn 1
;
J.a0 ; a1 ; :::; an / D
;
qn qn C qn 1
v J.a0 ; :::; an / \ J.b0 ; :::; bn / ; khi v ch khi b0 D a0 ; :::; bn D an .
Bi tp 3.3. Chng minh B 3.2.
B 3.4. Vi k hiu nh trn, vi mi a0 2 Z; a1 ; :::; an ; k 2 N:
.J.a0 ; :::; an ; k//
2
1
<
<
:
3k 2
.J.a0 ; :::; an //
k2
Chng minh. p dng B 3.1 v 3.2, v ng thc:
pn qn

pn 1 qn D . 1/n ;

ta c c:

pn
.J.a0 ; :::; an // D
qn

pn C pn 1
qn C qn 1
1
D
qn .qn C qn 1 /
1
:
D 
qn 1
2
qn 1 C
qn

pn .k C 1/ C pn 1
qn .k C 1/ C qn 1
1
1
D
.qn k C qn 1 /.qn .k C 1/ C qn 1 /
1


:
D
qn 1
1
qn 1
2
2
qn k 1 C
1C C
kqn
k
kqn

pn k C pn
.J.a0 ; :::; an ; k// D
qn k C qn

Lu rng

qn 1
qn 1

< 1.
kqk
qk
26

Tp ch Epsilon, S 06, 12/2015

3.2. nh lut 0-1 ca Lin phn s


Vn dng mt s tnh cht c bn ca lin phn s trn, chng ta s chng minh mt nh lut
0-1 ca Lin phn s. nh lut ny l mt trong 2 b quan trng dng chng minh phn
phn k ca nh l Khintchine.
nh l 3.5. Gi s nh f .n/ l mt hm s dng bt k, t:
Fn D fx D a0 I a1 ; ::: W an  f .n/g:
1
X



1
(i) Nu nh chui
hi t th  lim sup Fn D 0.
f .n/
n!1
nD1
1
X



1
phn k th  R X lim sup Fn D 0.
(ii) Nu nh chui
f
.n/
n!1
nD1
Chng minh. Chng ta s chng minh phn phn k, cn phn hi t dnh lm bi tp dnh
cho c gi. Lu rng trong c 2 trng hp, chng ta c th gii hn cc s x 2 0; 1/.
Xt mt dy a1 ; :::; amCn bt k sao cho vi 1  i  n:
amCi  f .m C i/:
Ta c c:
0
[
@

1
X

J.0; a1 ; :::; amCn ; k/A D

kf .mCnC1/

.J.0; a1 ; :::; amCn /

(B 3.2)

kf .mCnC1/

1
> .J.0; a1 ; :::; amCn /
3

X
kf .mCnC1/
Z 1

1
k2

(B 3.9)

1
dx
> .J.0; a1 ; :::; amCn /
2
3
f .mCnC1/C1 x
1
D
.J.0; a1 ; :::; amCn /:
3.f .m C n C 1/ C 1/
Mt khc,
J.0; a1 ; :::; amCn / D

1
[

J.0; a1 ; :::; amCn ; k/:

kD1

T ta suy ra:
0
1

[
@
J.0; a1 ; :::; amCn ; k/A < 1
k<f .mCnC1/


1
.J.0; a1 ; :::; amCn /:
3.f .m C n C 1/ C 1/

t
Em;n D

J.0; a1 ; :::; amCn /;


/2NmCn

.a1 ;:::;amCn
amCi <f .mCi /;1i n

27

Tp ch Epsilon, S 06, 12/2015


bt ng thc trn cho ta:

.Em;nC1 / < 1


1
.Em;n /:
3.f .m C n C 1/ C 1/

Ln lt p dng bt ng thc ny, ta c c:



n 
Y
1
.Em;n / < Em;1
1
3.f .m C i/ C 1/
i D2
n
X

< Em;1 exp

i D2

Bi tp 3.6. Chng minh rng chui

1
3.f .m C i/ C 1/

!
.1 C x < e x /

1
X

1
phn k (! 1) dn n chui
f .n/
nD1

n
X
i D2

1
3.f .m C i/ C 1/

phn k (! 1) vi mi m.
V vy, v phi hi t v 0 khi n tin n C1, v vi mi m; n, ta c c:
.Em;n / D 0:
Lu rng 0; 1/ X

1
[

Fn D

nDmC1

1
[

Em;n ,

nD1

!


1 [
1
\
 0; 1/ X lim sup Fm D  0; 1/ X
Fn
m!1

D
D

1
[

mD1 nDm
1
[

0; 1/ X

mD0
1 [
1
[

!!
Fn

nDmC1

!
Em;n

mD0 nD1

1 X
1
X

.Em;n / D 0:

mD0 nD1

Bi tp 3.7. Chng minh rng:


0
1
[
@
J.a0 ; :::; an ; k/A <
kf .nC1/

4
.J.a0 ; :::; an //:
f .n C 1/

Bi tp 3.8. p dng bi tp 3.7 chng minh rng


.FnC1 \ 0; 1// <
V t suy ra phn hi t ca nh l 3.5.
28

4
:
f .n C 1/

Tp ch Epsilon, S 06, 12/2015

3.3. Chng minh phn phn k ca nh l Khintchine


B 3.9. Tn ti mt hng s tuyt i C > 0 sao cho vi hu ht mi s x D a0 I a1 ; :::, tn
ti N > 0 sao cho vi mi n  N ,
qn < e C n :
Chng minh. t
[

En .g/ D

J.0; a1 ; :::; an /:

a1 a2 :::an g

pn
.J.0; a1 ; :::; an // D
qn

pn C pn
qn C qn

1
1
1
D q .q C q / < q 2 < .a a :::a /2
1
n n
n 1
1 2
n
n
1

ta c c cn trn cho o ca En .g/:


1
:
.a1 a2 :::an /2

.En .g// <

a1 a2 :::an g

Tch bn tay phi c th b chn bi tch phn nh sau:



n
n 
Y
Y
1
1
1
D
1
C
ai ai .ai C 1/
a2
i D1 i
i D1
n
Y

1
ai .ai C 1/
i D1
Z
n
ai C1
Y
dxi
D 2n
xi2
i D1 ai
Z a1 C1 Z a2 C1
Z
n
D2
:::
 2n

an C1

dx1 dx2 : : : dxn


x12 x22 : : : xn2

a1

a2

an

Z Z

dx1 dx2 : : : dxn


x12 x22 : : : xn2

Gi In .g/ l tch phn:


:::
trn min:
x1 ; :::; xn  1;

x1 x2 : : : xn  g;

ta c c:
.En .g// < 2n In .g/:
Bi tp 3.10. Chng minh rng khi g  1, In .g/ D 1.
Bi tp 3.11. Chng minh rng khi g > 1,
n 1
1 X .ln g/i
:
In .g/ D
g i D0 i

29

Tp ch Epsilon, S 06, 12/2015


t g D e An , vi mt hng s A > 1 bt k, ta c c:
 En e

An



<e

n 1
X
.An/i

n.ln 2 A/

i D0

< e n.ln 2

.An/n
n
n
n.An/
A/
p
nn e n n

A/

< Be n.ln 2
p
< B 0 ne

(cng thc Stirling)

n.A ln A ln 2 1/

Vi A ln,
A
1
X

 En e An

ln A

ln 2

1 > 0;



hi t. Theo B Borel-Cantelli, vi hu ht mi s x trong



on 0; 1/, x ch nm trong hu hn cc tp En e An . Hay ni mt cch khc, vi hu ht mi
s trong khong 0; 1/, vi mi n ln,
v khi , chui

nD1

a1 a2 :::an < e An :
Khi ,
q n D an q n

C qn

< 2an qn

< ::: < 2n an an 1 :::a1 < 2n e An D e C n

vi C D A C ln 2.
1
X

Chng minh nh l Khintchine. Gi s nh chui

.n/ phn k, t:

nD1

f .x/ D e C x


eC x ;

vi C l hng s c c t B 3.9. Tch phn:


b

Z
a

1
f .x/dx D
C

Cb

.u/du
Ca

vi 0 < a < b tin n v cng khi b ! 1, v v hm f .x/ khng tng theo gi thuyt, ta c
1
X
c chui
f .n/ phn k.
nD1

p dng nh l 3.5, vi hu ht mi s x D a0 I a1 ; :::, tn ti v s ch s n sao cho:


anC1 

1
:
f .n/

T suy ra:

pn
1
1
f .n/




qn
qn qnC1
anC1 qn2
qn2
30

Tp ch Epsilon, S 06, 12/2015


Theo B 3.9, vi hu ht mi s x D a0 I a1 ; ::: v vi mi n ln,
qn < e C n () n >

ln qn
:
C

V vy, ta c c, hu ht mi x D a0 I a1 ; :::, tn ti v s ch s n sao cho:




ln qn

.qn /
C
x pn  f .n/ 
D
;

2
2
qn
qn
qn
qn
tc l x 2 WA. /.

Ti liu tham kho


[1] A. Y. Khintchine, Einige Satze u ber Kettenbruche, mit Anwendungen auf die Theorie der
Diophantischen Approximationen, Math. Ann. 92 (1924), pp. 115125.
[2] A. Y. Khintchine, Zur metrischen Theorie der Diophantischen Approximationen, Math.
Zeitschrift 24 (1926), pp. 706713.
[3] A. Y. Khintchine, Continued Fractions (1935).
[4] R. M. Solovay, A model of set-theory in which every set of reals is Lebesgue measurable,
Ann. of Math. 92 (1970), pp. 156.
[5] L Ngc Tu, Xp x Diophantine trn R v Lin phn s, Epsilon 4, (2015).
[6] L Ngc Tu, Xp x Diophantine trn Rn - Quy tc Dirichlet v Hnh hc ca s, Epsilon
5, (2015).

31

Tp ch Epsilon, S 06, 12/2015

32

HNH HC RI LNG T
m Thanh Sn

c s ng ca gio s m Thanh Sn, trong s ny Epsilon trn trng


gi n c gi bn dch ca gio s t bi Geometra y entrelazamiento cuntico
ca Juan Maldacena, Investigacin y Ciencia, s 11=2015. Bn gc c gi c th
xem ti trang nh ca gio s y.

1. Gii thiu
Vo u th k XX c hai cuc cch mng trong vt l: C hc lng t v thuyt tng i
rng. C hc lng t cho ta bit cc nh lut chi phi th gii vi m, cn thuyt tng i
rng, c Einstein xy dng nm 1915, l mt l thuyt v khng gian v thi gian. Theo thuyt
tng i rng, khng-thi gian c cong v khng phi tnh, m l ng.
Ti nay cc tin on ca c hai l thuyt u c thc nghim xc nhn. Tuy nhin hai l
thuyt thng c p dng vo nhng hin tng rt khc nhau. Ta thng dng c hc lng
t m t cc vt rt nh (nh nguyn t hay photon), v dng thuyt tng i rng nghin
cu s thay i ca khng thi gian gn cc vt nng (v d cc ngi sao hay cc thin h).
nghin cu cc h vt l va nng va nh, nh v tr ngay sau v n ln, ta cn mt cch miu
t lng t cho khng thi gian. iu ny, mt trm nm sau ngy Einstein xy dng c thuyt
ca mnh. vn cn l mt thch thc ln cho vt l c bn.
Hai nm trc, c khch l bi mt cuc tranh lun v cc tnh cht ca l en, nh vt l
Leonard Susskind ca i hc Stanford v tc gi xut ra mt mi lin h gia hai hin
tng c v nghch l trong c hc lng t v thuyt tng i rng: Hin tng ri lng t
(quantum entanglement) v l giun (wormholes). Ri lng t l mt dng tng quan lng t
c th tn ti gia nhng h vt l cch xa nhau. L giun l nhng ng tt xut hin trong mt
s nghim ca phng trnh Einstein v ni nhng vng rt xa nhau ca khng gian.
Di y chng ta s thy hai hin tng ny c lin quan vi nhau. S tng ng ny tm
thi ch c c th chng minh cht ch trong mt vi trng hp c th, nhng c l ng trong
trng hp tng qut. tng ca chng ti v mi lin h gia hnh hc v ri lng t c th
l mt nguyn tc m tt c l thuyt lng t ca khng-thi gian, hay hp dn lng t, phi
tun theo. Nguyn tc ny c nhng h qu quan trng. Thm ch, mt cch no , c th chnh
khng-thi gian cng xut hin ra t s ri lng t ca nhng thnh phn vi m c bn nht ca
th gii.
Mt iu th v l c hai khi nim, ri lng t v l giun, u xut phi t hai bi bo m bn
thn Einstein vit vo nm 1935: Hai cng trnh dng nh lin quan n hai hin tng rt khc
nhau, v sau ny Einstein cng khng bao gi ngh l chng c th c lin h g vi nhau. Thc
s m ni, ri lng t l mt hin tng lm cho Einstein ht sc bc bi. Di y chng ta s
xem li hai bi bo v gii thch mi lin h ca chng t quan im hin i.
33

Tp ch Epsilon, S 06, 12/2015

2. L en v l giun
Mt tin on ng kinh ngc ca l thuyt Einstein l l en. L en c nhnh thnh khi ta
gom mt khi lng vt cht ln vo mt vng nh ca khng gian. Vt cht ny khng cn l g
c bit, v d t khng kh ta cng c th to ra mt l en. Ch c iu l cn rt nhiu khng
kh: Ta cn phi lm y mt hnh cu c kch thc bng kch thc ca h mt tri. Nu ta lm
c nh vy th khi kh ny s suy sp di sc nng ca mnh v nn li cho ti khi tr thnh
l en.
Tt c cc l en u c bao bc bi mt mt cu gi tng gi l chn tri s kin. Ta gi
mt cu ny l gi tng v mt nh du hnh v tr ri t do vo l en s khng thy g ch
ny. Tuy nhin, mt khi vt qua mt cu ny, ngi s khng quay tr li c. Ngi s
i vo mt vng m khng gian suy sp vo mt k d, ch m hnh hc co li hon ton. Ti
gn im k d nh du hnh v tr s cht bp di lc hp dn.
Bn ngoi vng cha vt cht, l en c m t bng mt nghim ca phng trnh Einstein
m nh vt l Karl Schwarzschild tm ra nm 1916: Mc tiu ban u ca Schwarzchild l tm
trng hp dn ca mt cht im. Trn thc t, nghim ca ng ta khng c vt cht: N m
t mt trng hp dn thun tu vi i xng cu, khng hn khng km. Tuy trng c v n
gin, cc tnh cht ca khng-thi gian ny kh kh din gii. Ch n nhng nm 196o ngi ta
mi hiu c tng i cu trc ton cc ca nghim ny.
Nm 1935; mt trong hai bi bo ni n trn, Einstein v Nathan Rosen, mt cng tc
vin ca Einstein Vin Nghin cu Cao cp Princeton, tm ra mt kha cnh rt th v ca
nghim Schwarzschild. H tm ra rng nghim ny cha hai khng gian c lp ni vi nh bng
mt ci ng. Ti mt thi im nht nh, ta c th hng dung ra hnh hc ca nghim nh sau:
xa vng trung tm, khng gian l phng (khng c cong ng k), nhng khi vo gn trung
tm, hnh hc b mo i v ni vo mt khng gian th hai, mt khng gian cng tim cn phng.

S kt ni hnh hc m Einstein v Rosen tm ra c gi l cu Einstein-Rosen .ER/; hay l


giun. Einstein v Rosen phn tch cu trc hnh hc ca mt siu din ti mt thi im c nh
(ni cch khc, mt khng gian cong ba chiu) nhiu nm trc khi ta hiu c cu trc ton
cc ca nghim Schwarzschild. Mc tiu ca Einstein v Rosen l tm mt miu t hnh hc cho
mt ht c bn khng c k d. Ngy nay chng ta bit rng din gii ca h l sai lm.
Chic cu nguyn thu ca ER ni hai khng gian c lp. Tuy nhin, ta c th c nhng nghim
ging nh vy nhng hai vng c ni cng thuc v mt khng gian. Ch cn thay i mt
cht, nghim Schwarzchild c th c din gii nh l mt nghim cha hai l en rt xa
34

Tp ch Epsilon, S 06, 12/2015


nhau nhng bn trong li ni vi nhau. Ta tng tng l c mt l en ti ni ta ang sng
v mt l en mt ngn h khc. Mt ngi quan st m ta s gi l Romeo ng cch chn
tri s kin ca l en th nht 1 mt, trong lc Juliet ng cch ng cch chn tri s kin
ca l en th hai cng 1 mt. Nu rut ca hai l en c ni vi nhau bng mt chic cu
ER; khong cch gia Romeo v Juliet qua l giun s l 2 mt, bt k hai vng khng gian xung
quanh hai l en xa nhau n mc no.

Nhng kiu hnh hc ny c v c vn . Ta nh li l mt trong nhng nguyn l c bn ca


thuyt tng i hp l ta khng th gi tn hiu i nhanh hn tc nh sng. Th nhng c
v l l giun cho php vi phm nguyn l ny v ta c th gi tn nhiu qua n. Tuy nhin, nm
1963; Robert W. Fuller i hc Columbia v John A. Wheeler i hc Princeton chng
minh rng khng th dng cu ER gi bt k loi tn hiu no. thy iu ny ta phi xem
xt tnh cht ng ca hnh hc trong thi gian ng mt vai tr quan trng. L giun ca
chng ta m t hnh hc ca khng gian ti mt thi im c nh. Nhng hnh hc ny tin ho
theo thi gian. Fuller v Wheeler chng minh c rng chic cu ER gin ra di ca cu
tr thnh v cng trc khi ngi quan st kp vt n. iu ny c th s lm cc nhn vt
trong cc phim khoa hc vin tng tht vng, v h vn hay dng l giun du hnh trong v
tr vi tc ln hn tc nh sng.
Trong trng hp hai l en ni vi nhau bn trong bng mt l giun, chn tri ca hai l en
chm vo nhau ti mt khonh khc, nhng sau ri nhau ra qu nhanh cho ai c th kp
vt cu sang bn kia. Nh th nu Romeo mun gi mt thng ip nhanh hn tc nh sng
cho Juliet, anh ta s khng th lm c. Romeo c th phng mt tn la mang thng ip vo
l en bn anh ta v tn la s ri vo bn trong l en. Thuy nhin, khi bn trong, hai chn
tri chy khi nhau vi tc rt nhanh, v khng gian suy sp trc khi thng ip c th ti
chn tri ca Juliet.
Tuy nhin, Romeo v Juliet vn c c hi gp nhau. H c th nhy, ngi no vo nhng l en
ca ngi y, v gp nhau bn trong l en. Tuy nhin c mt vn : Mt khi vo trong,
h khng th ra ngoi na. y l mt trng hp ca s cun ht cht ngi. Ci l ca hnh
hc ny l n m t hai l en c cng chung mt rut. V th m Romeo v Juliet c th gp
nhau trong l en.
35

Tp ch Epsilon, S 06, 12/2015


Ta phi nhn mnh l nhng l giun ca chng ta rt khc nhng l giun thng gp trong cc
phim khoa hc vin tng. Nhng l giun trong phim (nhng l giun ta c th i qua c) i
hi mt loi vt cht c nng lng m, c l khng tng thch vi cc nh lut vt l ta bit.
V vy nhiu nh vt l tin rng loi l giun trong cc phim khoa hc vin tng khng th tn
ti trong t nhin. Cc l en ta xt y cn c mt kha cnh khc ng nhc n. Cc l en
c to ra do vt cht suy sp ch tng ng vi mt phn ca hnh hc Schwarzchild, bi v s
c mt ca vt cht lm nghim thay i. Cc l en loi ny c nghin cu rt r; trong
trng hp ny khng c l giun no ht. Loi l en c to ra qua cc qu trnh vt l thin
vn, v d nh khi mt ngi sao suy sp, l loi khng c l giun ni vi mt vng khc ca
khng gian hay ni chng vi nhau, khc vi nghim y ca Schwarzschild. Tuy vy chng
ta vn mun hiu r hn din gii vt l ca khng-thi gian Schwarzschild. D sao, y cng l
mt trong nhng nghim n gin nht ca phng trnh Einstein.

3. Tng quan lng t


ng ngc nhin l s gii thch cho nghim Schwarzschild li lin quan n bi bo th hai ca
Einstein ta nhc ti trn. Cng trnh ny ngy nay rt ni ting v c nh hng. Bi ny
c vit cng nm, vi ng tc gi l Rosen v Boris Podolsky, cng l nh nghin cu ca
Vin Nghin cu Cao cp. Cc tc gi (ngy nay c bit n bi tn vit tt EPR) ch ra l c
hc lng t cho php mt loi tng quan (correlation) rt l gia cc h vt l xa nhau, mt
mi tng quan m sau ny c gi l ri lng t.
S tng quan gia cc vt xa nhau c th xy ra ngay trong vt l c in. Gi s bn i ra khi
nh m ch mang theo mt chic gng tay v bn qun chic kia nh. Trc khi nhn vo ti,
bn khng bit mnh mang chic no i. Tuy nhin, mt khi bn nhn vo ti v thy mnh mang
chic gng phi, bn bit ngay chic nh l gng tri.
Tuy nhin, ri lng t lin quan n tng quan gia cc i lng lng t, nhng i lng
c th phi tun th nguyn l bt nh ca Heisenberg. Nguyn l ny ni rng c nhng cp
bin s m ta khng th bit hon ton chnh xc cng mt lc. Th d ni ting nht l v tr v
vn tc ca mt ht: Nu ta o chnh xc v tr th vn tc tr thnh khng xc nh v ngc li.
Trong bi bo ca mnh, EPR hi ci g s xy ra nu ta c hai h xa nhau v trong mi h ta
quyt nh o mt cp bin chu nguyn l bt nh.
Trong v d m EPR phn tch, ta xt hai ht c cng khi lng v ch chuyn ng trong mt
chiu. Ta gi hai ht l R v J v ta chun b hai ht sao cho trng tm ca chng c to
xc nh, tc l Xcm D xR C xJ D 0: Ngoi ra ta c th lm cho vn tc tng i gia hai
ht vrel D vR vl ; c mt gi tr chnh xc, v d vrel D v0 : Trc khi tip tc, ta phi lm r
mt iu. Ta va t gi tr chnh xc cho c v tr v vn tc. iu ny c vi phm nguyn l bt
nh ca Heisenberg khng? Ta nh li l nguyn l bt nh ni n v tr ca mt h v vt tc
lin quan n v tr . Tuy nhin, nu c hai h, khng g cm ta bit v tr ca h th nht v
vn tc ca h th hai. Trong trng hp ang xt, chng ta khng xc nh v tr v vn tc ca
trng tm, m l v tr ca trng tm v vn tc tng i ca hai ht. Do y l hai i lng
c lp, khng c vn g khi ta xt trng thi ban u nh EPR mun.
By gi ta gp mt iu rt bt ng. Gi s hai ht ca chng ta i ra khi nhau rt xa v hai
ngi quan st xa nhau, Romeo v Juliet, quyt nh o v tr ca chng. Do cch nhng ht
36

Tp ch Epsilon, S 06, 12/2015


ny c chun b, nu Juliet nhn c gi tr xJ th Romeo s tm c ht ca mnh v tr
xR D xJ : Mt khc, nu h o vn tc v Juliet nhn c kt qu xJ th Romeo chc chn s
tm c gi tr vR D v0 C vJ : Tt nhin Romeo v Juliet mun chn o i lng no cng
c. Nhng nu Juliet o v tr v Romeo o vn tc, kt qu ca h s hon ton ngu nhin v
khng c tng quan g ht.
Ci l l nu Juliet quyt nh o v tr ca ht ca mnh th ht ca Romeo s c mt v tr hon
ton xc nh mt khi ta bit kt qu o ca Juliet. Cng vy nu hai ngi o vn tc. Ta c th
ngh l khi Juliet o v tr, ht ca Romeo bit ngay lp tc l phi chim mt v tr xc nh.
Mi nhn th iu ny c v l mt s truyn thng tin tc th: Lp i lp li th nghim nhiu
lm, Juliet c th gi cho Romeo mt thng ip gm s 0 v 1 bng cch chn o v tr hay tc
ca ht. Tuy vy, Romeo khng th c c thng ip nu khng bit kt qu o ca Juliet.
Do ta khng th dng tng quan do ri lng t gi tn hiu nhanh hn nh sng.
Ri lng t c th l mt tnh cht b mt nht ca cc h lng t, tuy nhin qua nhiu nm
thng c kim chng bng thc nghim. Trong hai moi nm tr li y, cc tng quan
lng t a n nhng ng dng thc t v nhng tin b ln trong cc ngnh nh mt m
v thng tin lng t.

4. ER = EPR
Ta quay li cc l en. Nm 1974 Stephen Hawking chng minh rng cc hiu ng lng t lm
cho cc l en pht ra bc x ging nh mt vt nng. iu ny chng t cc l en c nhit .
Nhit ny cng cao nu vt cng nh. Tht s ra mt l en c th trng. C th, mt l en
vi kch thc bng mt vi khun, vi bc x c bc sng ging nh bc sng ca nh sng
nhn thy, s c mu trng do bc x Hawking. L en ny khng pht ra nhiu nh sng, nhng
n gn n ta s thy mt im sng chi. Nhng mt l en kch thc ny c mt khi lng
khng l, do ta khng th s dng n nh mt ngun nng lng.
i vi nhng l en c to ra mt cch t nhin t s suy sp ca mt ngi sao, bc x
Hawking yu n mc trn thc t khng th quan st c. Nhng vt th ny qu to v qu
lnh c th cm nhn c hiu ng ny. Tuy nhin, vic cc l en c nhit c nhng h
qu quan trng.
Chng ta bit t th k XIX l nhit c c l do chuyn ng ca cc phn t vi m trong
h. V d, trong cht kh, nhit ny sinh do chuyn ng ca cc phn t kh. V th ta c th
ch i l mt l en c cha nhng thnh phn vi m c kh nng to ra v s cc cu hnh,
hay l trng thi vi m. Chng ta cng tin rng, t nht nhn t bn ngoi, cc l en cng phi
hnh x nh nhng h lng t bnh thng, chu tt c cc nh lut c hc v nhit ng hc.
Xem xt t bn trong, khng c g cm ta xt cc trng thi ri lng t ca cc l en. Ta tng
tng hai l en cch xa nhau, mi l en c mt s ln cc trng thi vi m. Ta c th ngh ra
mt cu hnh trong mi trng thi vi m ca l en th nht c tng quan vi trng thi vi
m tng ng ca l den th hai. C th l nu ta quan st c l en th nht mt trng thi
vi m nht nh, th l en th hai cng ng trng thi ny.
iu hay l, xut pht t mt s xem xt nht nh lin quan n l thuyt dy v l thuyt trng
lng t, ta c th chng minh rng hai l en vi cc trng thi vi m ri vi nhau nh m t
37

Tp ch Epsilon, S 06, 12/2015


trn tc l trng thi EPR s cho ta mt khng thi gian trong c mt chic cu ER ni
hai l en vi nhau. Ni cch khc l ri lng t gy ra mt lin kt hnh hc gia hai l en.
Chng ti gi ci ny l s tng ng gia ER v EPR; hy ER D EPR; v n lin h hai
bi bo ca Einstein v cng s vit nm 1935: T quan im ca EPR; cc quan st gn chn
tri ca hai l en c tng quan vi nhau v ri lng t. T quan im ca ER; cc o c
c tng quan vi nhau do khong cch gia hai h qua l giun l rt nh. xc lp s tng
ng, iu quan trng l ta khng th gi thng tin qua l giun, cng nh ta khng th gi
thng tin dng ri lng t.
Ta c th ngh ti mt tng lai xa khi hai gia nh th ch mun gi Romeo v Juliet xa nhau.
H gi Romeo n Tinh vn Tin n v gi Juliet di Ngn h. Tuy nhin h cho php hai
ngi trao i thng ip v cc cp h lng t ri vi nhau. Vic ny s i hi rtt nhiu thi
gian, nhng chng ta ang mt tng lai m tui th cao hn hin nay rt nhiu. Vi s kin
nhn, Romeo v Juliet c th lm ra hai l en ri vi nhau. Nhng l en ny nhn t ngoi th
hon ton bnh thng, do hai gia nh khng nghi ng g. Tuy nhin, sau khi lm ra hai l
en, Romeo v Juliet c th nhy v bn trong v gp nhau trong ln cui cng trc khi cht
im k d.

5. Mt nguyn l ph qut?
Nhng tng dn ta ti y c nhiu nh nghin cu pht trin qua nhiu nm, bt u t
mt nghin cu ca Werner Israel ca i hc Alberta. Cng trnh ca ti v Susskind c
khch l bi mt nghch l c Ahmed Almheiri, Donald Marolf, Joseph Polchinski v James
Sully, tt c lc u i hc California Santa Barbara, pht biu nm 2012: Ngc li
vi nhng g mi ngi ngh trc , nhng nh nghin cu ny a ra kin rng hin tng
ri lng t buc ta phi thay th chn tri s kin ca mt l en (mt mt cu rt m, theo l
thuyt ca Einstein), bng mt ro chn nng lng cao khng th vt qua. T quan im ca
mi lin h ER D EPR ta c th gii quyt c nghch l ny.
S tng ng ER D EPR gi l bt c khi no c ri lng t ta cng c mi lin kt hnh
hc. iu ny phi ng k c trong trng hp n gin nht khi ta c hai ht ri vi nhau. Tuy
nhin trong trng hp ny mi lin kt hnh hc phi c cu trc rt nh v rt lng t, rt
khng ging khi nim hnh hc bnh thng ca chng ta. D chng ta cn cha bit m t
nhng hnh hc vi m nh th no, tng ca chng ti l mi lin h ER D EPR cho ta
mt nguyn l m tt c cc l thuyt hp dn lng t phi tun theo. L thuyt lng t hp
dn c nghin cu nhiu nht l l thuyt dy. Trong l thuyt dy, mi lin h ER D EPR
c th c chng minh mt cch cht ch trong mt s trng hp m s ri lng t c mt
dng nht nh, tuy nhin hin nay khng c s ng thun l mi lin h ny c tho mn
trong tt c cc trng hp.
Chng ta thy l s ri lng t c th, ng ngha en, ko hai h xa nhau li gn nhau. Ta
cng bit l hai vng gn nhau ca khng gian c ri vi nhau. Mt cch t nhin, ta c th ngh
l khng-thi gian, mt cu trc lin tc, c th bt ngun t ri lng t, mt tnh cht rt lng
t. tng ny ang l tiu im ca nhiu nh nghin cu, nhng cn cha c tng hp li
thnh mt pht biu chnh xc.

38

LOGIC CA TON HC NG DNG


I. I. Blekman, A. D. Myshkis, Ya. G. Panovko

Bi vit ny c chng ti trch t "Ton hc ng dng", nh xut bn Khoa


hc K thut H Ni, 1985. y l bn dch Vit ng ca Trn Tt Thng t bn gc
ting Nga ca ba tc gi I. I. Blekman, A. D. Myshkis, v Ya. G. Panovko (c
gi c th i chiu thm vi ta ting Anh ca sch l Mechanics and Applied
Mathematics: Logics and Special Features of the Applications of Mathematics").

Phng hng ng dng v l thuyt trong pht trin ton


hc
Hai ngun ton hc c bn
Phng hng ng dng v l thuyt: V tr hin nay ca ton hc ng dng s tr nn r rng
hn nu nh theo di s b con ng pht trin ca bn thn ton hc. R rng l ng lc pht
trin ca ton hc c hai ngun c bn tn ti mt cch khch quan. Mt l ngun bn ngoi do
vic cn thit phi dng cc phng tin ton hc gii nhng bi ton nm ngoi phm vi
ton hc, cc bi ton ca cc khoa hc khc, c k thut, kinh t, ... chnh y l ngun u tin
v mt lch s. Ngun th hai l ngun bn trong do vic cn thit phi h thng ha cc s kin
ton hc khm ph c, gii thch cc mi lin h gia chng vi nhau, hp nht chng li
bng cc quan nim khi qut l lun, pht trin l lun theo cc quy lut bn trong n, chnh
ngun ny thi im dn ti ch tch ton hc thnh mt khoa hc. 1
Tt nhin i khi cng kh phn gii cc ngun ny. V d nhng ngun thc y ny sinh do
vic p dng cc phng php ca mt lnh vc ton hc v mt lnh vc khc 2 i khi li rt
ging cc ngun thc y thu nhn c khi p dng cc phng tin ton hc ngoi phm vi
ca ton hc. Mt khc, vic h thng ha c th l do c nhng nhu cu thc tin trc tip.
V vy c th l mo him nu xc nh qu chi tit v ranh gii gia hai ngun . Tuy nhin
nhng c im ca cc ngun v nh hng ca chng trong i b phn cc trng hp vn
d dng nhn thy c. Hai phng hng pht trin ca ton hc ng vi hai ngun c
gi l phng hng ng dng v phng hng l thuyt (thun ty).
Xin nhn mnh l y mun ni v nhng nh hng chim u th trong vic xy dng v pht
trin ca cc phng php ton hc, ca cc khi nim v nhng khng nh. Cn i vi bt k
1

Trong li gii thiu cho mt cun sch ph bin ni ting ca R.Courant v G.Robbins [28] ni: R rng l
s vn ng i ln trong lnh vc ton hc l do s xut hin nhng nhu cu m mc nhiu hay t u c mang
mt tnh cht thc tin. Nhng mt khi xut hin th s vn ng t phi c nhng khun kh ni ti ca n v
vt ra ngoi phm vi ca tnh hu ch trc tip. Chnh v vy s bin i hon ton mt khoa hc ng dng thnh
khoa hc l thuyt thy trong lch s c i v ngy nay cng khng phi mc km hn: Ngi ta tha
nhn s ng gp ca cc k s v cc nh vt l vo ton hc hin i.
2
V d p dng gii tch v ton hc

39

Tp ch Epsilon, S 06, 12/2015


bn cht ton hc no c xy dng th vn n thuc phng hng no - l thuyt hay
ng dng - thng l v ngha. Nn quy phng php Bubnov - Galeckin hoc khi nim hm
delta ca Dirac, hoc cng thc ca Taylor thuc v phng hng no? Ch c th tr li c
nhng cu hi nu mun ni v lch s pht sinh cc khi nim hoc v nhng hon cnh
c th trong bt gp chng. ng l mt s chng nh i s ng iu chng hn th
hin nay hon ton thuc v phng hng l thuyt v mt t vn nh phng php chn xc
sut xc nh kh nng thc tin ca s kin, khi nim v tnh hi t thc tin hay v hn thc
tin th hin thi hon ton thuc v phng hng ng dng ( y khng phi ngu nhin m
c cc t hin nay, hin thi).

Giai on u ca pht trin ton hc


nhng giai on pht trin sm ca ton hc th hai phng hng ny c th thy c c
bit r nt. Bi v lc y cc phng hng ny tc ng ln nhau tng i yu nn thm ch
cn c th ni c l hai ngnh ton hc hu nh tch bit nhau - ton hc ng dng v ton
hc l thuyt (thun ty).
V d ton hc c Ai Cp cng nhin l ton hc ng dng, n lin h trc tip vi vic o c
rung t, tnh ton th tch cc bnh, tnh m thc tin, tnh thi gian (ni ring l d on
nht thc v nguyt thc), ... Ton hc Mehico c i v mt s cc dn tc khc cng c
tnh cht y. Ton hc thun ty c l pht sinh ln u tin C Hy Lp lin h vi khoa hc
ngy bin v tch hng khi ton hc ng dng. 3 Chnh khoa hc C Hy Lp ra phng
php suy din trong vic xy dng l thuyt. Theo phng php ny th mi khng nh trong
lnh vc ny hay khc u c th bng nhng phng php khc ca logic hnh thc m suy ra t
mt s nhng khng nh khng chng minh c gi l tin (chng hn xem [30]). K t
phng php trnh by ny c gi l mt trong nhng nt tiu biu quan trng nht ca ton
hc (nu khng phi l mt nt duy nht). Tnh cht ch ca phng php suy din gy c
mt s n tng mnh m cho nhng th h sau n ni c nhng (ni thm l khng
thnh) gn hnh thi suy din cht ch cho cc lnh vc tri thc khc. y thm ch c c
trit hc (o c hc ca B.Spinoza).
Xin lu mt tnh tit ng ch l t khoa hc c Hy Lp tip cn khi nim v hn,
tnh tit v sau c xa b v ri li c ti sinh mc cao hn ch cc cng trnh
v logic ton hc ca th k XX. Khoa hc c Hy Lp khng tha nhn tnh v hn thc ti v
khng th tm thy mt pht biu ton hc no thi iu m hin nay c gi l mt tp
hp v hn hoc mt qu trnh v hn. Mt v d tiu biu mnh m hin nay c pht biu
l: Tp hp cc s nguyn t l v hn th Euclide pht biu i l: Nu cho mt tp hp
no (hiu ngm l hu hn) cc s nguyn t th cn t nht mt s nguyn t na . y c
3

V im ny F.Klein [29, tr. 146 147] vit r Nu bt u t nhng ngi Hy Lp c thi ta s thy c s
phn gii r rt gia ton hc thun ty v ton hc ng dng t thi Paton v Aristotel. Thuc ton hc thun ty
trc ht c php dng hnh Euclide quen thuc. Thuc ton hc ng dng c bit c cc php ton v s gi l
logictic . l s chung nht). y ton hc ng dng kh b thnh kin coi thng v trong nhiu trng hp
vn cn n tn ngy nay, nhng i b phn ch thy nhng ngi t mnh khng bit tnh ton. Tnh hnh ny ca
logictic c th phn no do n c pht trin trong mi lin h vi lng gic v vi nhng nhu cu o c thc
tin rung t m thi xa l do nhng ngi lm nhng ngh khng c cao qu lm thc hin. Tt nhin n li
c phc hi mt phn no l do nu thiu n th khng th lm c khoa hc khc, tuy c ging nh trc a
nhng li ngc vi n ch lun lun c coi l mt trong nhng khoa hc cao qu nht: l thin vn hc.

40

Tp ch Epsilon, S 06, 12/2015


th thy c s tng t trc tip vi khi nim tnh tip tc c khng hn ch l khi nim m
mt trong nhng phng hng hin i ca logic ton c tha nhn l thay th khi nim
tnh v hn thc ti.
Mi ngi u bit rng khng tha nhn tnh v hn thc ti s ko theo nhng kh khn nht
nh v mt logic (v im ny ta nh li nhng ngy bin ca Zenon) m ngi Hy Lp ni
chung thy r khi h nhn xt rng khng gian v thi gian c th phn chia c v hn
nhng v mt hin thc th khng th phn chia c v hn nh vy. Di y s cn thy rng
tha nhn v hn thc ti cn ko theo nhng kh khn khng nhng ln hn v mt logic m
cn c nhng kh khn thc tin na.
Nhng biu hin cao nht v tnh cht ch trong ton hc c Hy Lp l l thuyt cc t l v
phng php vt cn ca Evdocs tng t nh l thuyt tng t nh l thuyt hin i v s
thc v phng php chuyn qua gii hn nhng khc ch l ngi Hy Lp khng thy ni
n nhng tp hp v hn cc qu trnh v hn. 4
Tuy nhin cng vi nhng kit tc ny v tnh cht ch, trong logic ca ton hc Hy Lp cn c
c nhng l hng m theo quan im hin nay th qu r rng. V d nhng nh ngha ban u
v cc khi nim im, ng thng, ... thc cht khng phi l cc nh ngha (im l ci
khng c b phn, ... ) v v sau khng c nhc ti na. Cc tin ch bao hm cc mi
tng quan gia cc i lng nhng khng phi tt c cc mi quan h c dng n. Hon
ton khng c nhng nh ngha v cc tin lin quan ti khi nim th t cc im trn mt
ng thng hoc trn mt ng trn, tc l khi nim ny dng nh thuc v s cc t (ging
nh cho trc, ngi ta cho , ...) c ngm hiu khi xy dng l thuyt. Ngoi ra iu ng
ch l nhng ngi Hy Lp tnh ton chiu di, din tch, th tch ca cc ng thng, cc
hnh, cc vt th khc nhau v i khi kh phc tp nhng vn v bn thn s tn ti o y
th li khng c t ra, ...
Trong khi ngi Hy Lp (Archimede ni ring) dng c cc php chng minh da trn
nhng tng t mt cch my mc, song nhng php chng minh y b coi l khng cht ch,
ch c tnh cht chun b, v nhng khng nh thu nhn c nht thit v sau phi c chng
minh mt cch cht ch (chng ti s cn quay li khi nim tnh cht ch trong ton hc).
C l s tch bit r nt gia ton hc thun ty v ton hc ng dng cng l nt tiu biu cho
cc nc o Hi thi Trung c. y l thuyt v thc tin gii cc phng trnh i s cng
nh gii tch t hp ngy cng n nhp su vo ton hc thun ty, ni ring, nhng pht minh
ton hc ln hi nh cc h s nh thc, cc cng thc gii phng trnh bc 3 v bc 4 th
hon ton thuc v ton hc thun ty.

Khng loi tr mt vic l do chnh pht hin c rt sm nhng kh khn lin quan ti nhng i lng
v c nn khin ngi Hy Lp khng pht trin nghin cu cc php ton v s m trong nhng thi i trc
t c nhng thnh tu ng k Phng ng. Thay vo h i tm phng hng trong con ng ri
ren ca hnh hc tin thun ty. Th l bt u mt trong nhng n lc m mn k l trong lch s khoa hc v c
th y nhng kh nng sng chi b nh nhng. Hu nh trong hai thin nin k nh hng ca truyn thng
hnh hc Hy Lp km hm s tin ha tt yu ca quan nim v s v v php tnh bng ch m sau ny tr
thnh nn tng ca khoa cc hc chnh xc [28, Li gii thiu]

41

Tp ch Epsilon, S 06, 12/2015

Phc hng khoa hc


Vi giai on u ca thi k Phc hng khoa hc, tnh hnh thay i v c bn, t nhng
cng trnh ca G.Galilei, I.Kepler v ca cc nh khoa hc khc. i vi h, ton hc l phng
tin t duy ton hc tr thnh mt trong nhng v kh chnh ca khoa hc t nhin. p lc
mnh m ca khoa hc t nhin c tc dng rt tt i vi s pht trin ca ton hc. Trong
nhng th k XVI - XVIII c hai phng hng ng dng v l thuyt lin tc tc ng ln
nhau. Mt khung cnh in hnh l s ra i v pht trin ca khi nim ton hc phc thuc vo
nhng bi ton ca khoa hc t nhin hay ca hnh hc (lc nhng ng dng vo hnh hc
t c phn bit vi nhng ng dng vo c hc hay quang hc), v sau khi khi nim
c xy dng th chng bao lu n c mt cuc sng c lp v tip tc c pht trin theo
nhng quy lut ni ti ca ton hc. Mt s nhng kt qu ca s pht trin thun ty li
c p dng vo khoa hc t nhin v iu ny li lm xut hin nhng khi nim v bi ton
ton hc na. V d r rng v mi ngi u bit v s pht trin y l vic hnh thnh cc php
tnh vi phn v tch phn.
Trong thi hong kim ny ca pht trin ton hc mt cch hi ha th s tch bit v hn na,
s i lp gia ton hc thun ty v ton hc ng dng tr nn mt ht c ngha. C c
iu cng cn l do cc nh khoa hc ln thi k nh N.Newton, L.Euler, J.Lagrange v
nhng ngi khc khng ch l cc nh ton hc m cn l nh vt l, cc nh c hc na. Trong
cc cng trnh ca mi nh khoa hc pht trin c phng hng l thuyt ln phng
hng ng dng ca ton hc.

Thi k thng tr ca phng hng l thuyt tp hp


Thi k qu sang giai on tip sau ko di nhiu thp nin v v vy ch c th quy c
cho rng thi gian bt u thi k l th k XIX. N lin h vi hng lot cc cng trnh xut
sc nht v l thuyt tp hp (G.Cantor) v l thuyt hm (K.Weierstrass), v xy dng cc cu
trc i s tr tng u tin v phn tch cc tin hnh hc. Cc cng trnh ni ting v tin
b su sc thi k bin mt b phn ng k ca ton hc thnh mt khoa hc thng nht
c nhng yu cu thng nht v cc nh ngha, cc khng nh v cc php chng minh v c
nhng tiu chun thng nht v tnh cht ch.
Nguyn nhn ch yu ca giai on qu ny l do cui thi k trc trong ton hc tch ly
nhiu s kin, ra i hng lot cc l thuyt khng thng nht vi nhau v khng c nhng c
s lp lun vng chc l iu c th pht trin nhng l thuyt mt cch tin tng. S pht
trin tip sau ca cc b mn gii tch trong t pht ra i phng php da trn vic p
dng v hn thc ti - cc chui v hn, nhng i lng v cng b - i hi phi xc nh
r rng nhng khi nim v hm s v v vic chuyn qua gii hn, iu tt nhin ko theo
s ra i ca l thuyt s thc v l thuyt cc tp hp s, ... Quan im l thuyt tp hp cho
php pht biu c r nt khi nim nhm, mt khi nim quan trng nht trong i s hc v
dn ti mt kt cu logic ca hnh hc tha mn nhng ngi ng thi. 5
5

Nh P.S.Aleksandrov vit trong li ni u ca cun sch [31]: L thuyt tp hp v nhng ng dng gn


gi nht ca n khng nhng ch hnh thnh mt i tng nghin cu mi ca ton hc, ngha ca l thuyt tp
hp dng nh v cng to ln v n a ra mt phng php tng hp mi bao trm nhanh chng ton b ton
hc. Tt nhin y l ni v ton hc thun ty.

42

Tp ch Epsilon, S 06, 12/2015


Nh vy bc chuyn tip trong ton hc sang quan im l thuyt tp hp da trn c s, nh
hin nay vn ni mt cch khoan dung ca l thuyt tp hp ngy th 6 l s cn thit v v vy
l tin b. Trong khi vic sp xp th t cc c s ca khoa hc, nhng kh nng mi v to
ln khm ph c qua vic , ni ring nhng kh nng kh mnh m ca s tng tc gia
cc b mn khoa hc khc nhau 7 thc s nng cao vai tr ca phng hng l thuyt trong
ton hc m trong giai on ny (ko di mi ti chin tranh th gii th hai) chim u th v
quyt nh phong cch ca ton b ton hc ni chung.
Cn i vi phng hng ng dng th n tip tc c pht trin trc ht trong mi lin h
vi s pht trin ca vt l hc v c hc thin th song c l y khng c mt bc ngot
no c. Thi k ny m ra nhng con ng ng dng mi, chng hn nh i s vector v
gii tch, i s tenx v gii tch v v sau cn c c php tnh ton t, l thuyt hm suy rng,
... nhng bn thn tnh cht ca nhng ng dng th trong mt thi gian no v nguyn tc
vn nh c. B my ton hc c in kt hp vi nhng quan nim su xa v mt vt l dn
ti ch thc hin c nhiu khm ph ni ting, nh ngi ta vn vit trong cc sch ph bin
rng u nhn ca ngi bt nhng v d loi ny c mi ngi bit n rng ri l s tin
on v cc sng in t ca C.Maxwell, vic khm ph ra cc hnh tin Neptun v Pluton, s
tin on ca P.Dirac v Positron, ... Trn c s ny ra i mt trong nhng lnh vc quan
trng nht ca khoa hc ngy nay l vt l l thuyt.
Nhng thnh tu ca phng hng l thuyt, vic xy dng mt mc cht ch thng nht cho
ton b ton hc dn ti phng hng gii cc bi ton ton hc ra i trong nhng ng
dng cng mc cht ch ca phng hng l thuyt. Biu hin r nht ca khuynh hng ny
thy D.Holbert v A.M.Liapunov m chng ti s ni chi tit sau. Trong mt s cc trng
hp, khuynh hng ny t ra c th thc hin c v ngoi ra iu dn ti tnh i
ngu khi gii mt bi ton ng dng ni chung bi v vic t bi ton v biu th php gii
c tin hnh mc cht ch vt l (nhng c gng trong vic tin ha tng chng ring l
ca vt l trn c s l thuyt tp hp dng nh khng thnh cng cho nn y khng th
trnh khi mc cht ch vt l) nhng php gii ton hc li c thc hin mc cht ch ton
hc (s phn bit ny chnh l c trng cho thi k ny bi v thi k hong kim th cc
mc cht ch rt gn nhau nu khng phi trng nhau). Trong nhng trng hp phc tp hn
cng nh khi cc nh vt l gii nhng bi ton ton hc ng dng th trong php gii thng c
nhng lun c v vt l, song cc nh ton hc th coi php gii nh vy l khng c y
v tm cch thay n bng php gii hon ton t mc cht ch Weierstrass. Th l c mt s
phn i na v ngh nghip gia nhng yu cu v mc cht ch ca php gii mt bi ton
ng dng ca cc nh ton hc v cc nh ng dng.
Gp phn vo vic phn i ny cn c th c mt cc php tnh ton m mi ngi u bit l
hu nh khng bao gi thc hin c y mc cht ch Weierstrass. Song rt ra khi
nhng ng dng truyn thng ca Euler v ca nhng nhc trng khc trong thi k hong
kim cc nh ton hc ca phng hng l thuyt tp hp ngng vic tnh ton v hot ng
ny c t ra cc nh thin vn hc, cc nh pho binh, ... cng nh cho mt nhm khng ln
6

N i lp vi mt quan im hin i hn da hon ton vo cc phng php ca logic ton.


V d ch da trn c s l thuyt tp hp m a ra cch lp lun theo tin cho l thuyt xc sut mc
cc lnh vc khc ca ton hc thun ty. Nhng thnh t ny dn ti mt s cc quan im cc oan, v d
F.Doob [32, tr.7] vit: L thuyt xc sut ch l mt ngnh ca l thuyt o v khc ch n c bit ch ti
mt s cc khi nim chuyn mn ca l thuyt y v lnh vc ng dng c bit ca n. C l trn c s cng
c th ni c rng vn hc l mt ngnh ngn ng hc khc ch n c bit ch ti, ...
7

43

Tp ch Epsilon, S 06, 12/2015


cc chuyn gia tnh ton l nhng ngi c coi l ng mt ch no gia cc nh khoa
hc v cc k s. Nhng thnh tu m i b phn cc nh ton hc t c trong lnh vc ny
khng c coi trng v trong mi trng hp chng b coi l hon ton khng so snh c
vi nhng thnh tu ng ngc nhin ca phng hng mi.
Hon ton mi gn y, trong nhng bi ging ca mt trong nhng nh i s hc ni ting
c nhn mnh mt cch h thng n s khc nhau ch yu gia khoa hc v tnh ton 8 .
Xin lu l v sau, khi ton hc tnh ton tr thnh mt ri th li din ra s phn tng tip
sau: Theo cch din t hm hnh ca R.S.Guter [1, tr.13] th nhng ngi lm vic trong lnh
vc ton hc tnh ton chia thnh nhng ngi chng minh tnh hi t ca cc qu trnh tnh
ton v s tn ti ca nghim v nhng ngi p dng cc qu trnh tnh ton v tm li gii.
Chnh nhng ngi sau trc tip phc v hu ch cho cc khoa hc ng dng.
Cn i vi mc cht ch t c da trn c s l thuyt tp hp ngy th v trong gii tch
ton th da trn l thuyt gii hn m t bng ci gi l ngn ng -  9 th dng nh n thc
tin thu ht c tt c cc nh ton hc. ng l cng c nhng ngi khng hi lng
(v d L.Brauer, G.Weyl, ...) nhng trong hot ng thc tin th h cng s dng mc cht ch
y. ch ny ch khc ni chn tri cng c thy nhng mu thun trong l thuyt tp hp
nhng i b phn cc nh ton hc ch thy ch l chuyn bun ci khng ng kh chu
bi v n ch ng n nhng con ngo p khng ai cn n nh tp hp ca tt c cc tp hp,
hoc tp hp ca tt c cc tp hp khng cha chnh n, ... Cuc tho lun v tin chn (tin
Zermelo) kt thc vi tha thun rng trnh c n, nu c th, th trong trng hp
ngc li phi ch ra r rng vic p dng n, tt nhin nu ngh cn thn th s tha thun qu
l c ngy th v dng nh nhng ch dn nh vy c lm thay i mt ci g . Trong nhng
nm gn y trong nhng cng trnh v ton hc thun ty, tin chn c c s dng qu
rng ri, thm ch nhng mnh c chng minh bng tin c ng dng mt cch
c h thng, iu khin ngi ta c n tng kh r rng l c s ty tin i vi tin .
Nhng thnh tu ca nhng phng hng l thuyt tp hp, ngn ng v phng php ca n
tr thnh quen thuc i vi mt vi th h cc nh ton hc v to ra mt quan nim v
mc cht ch t c dng nh tuyt i. Nhiu nh ton hc (v nht l nhng ngi khng
phi l ton hc) hin nay vn c thi quen tin tng vo tuyt i v ch a vo ton hc nhng
mnh c chng minh cht ch tuyt i, nhng bi ton c t ra mt cch cht ch
tuyt i, ...
8

Cun sch hu ch ca C.Lanczos [33] bt u bng nhng li nh sau: Trong nhiu nm tc gi nghin
cu cc lnh vc ca gii tch ton l nhng lnh vc trc ht c s quan tm ca cc k s v cc nh vt l hc.
Vic lnh vc ny ca ton hc lm vic trong sut th k XIX khng c ch nh nhng cng trnh c in ca
gii tch c l l kt qu ca mt quan nim sai lm v mt lch s. Cho mi ti thi Gauss v Legendre, cc phng
php lm vic ca gii tch mi thu ht c s ch ca cc nh ton hc ni ting. Pht minh v l thuyt gii
hn lm thay i tnh hnh. K t thi gian vic a ra mt qu trnh xp x v hn c th xc lp c tnh
ng n ca cc kt qu gii tch xc nh c lp vi vic qu trnh p dng c c thc hin trong bi ton
hay khng, c coi l tha ng. Kt qu l dn dn phn cch ton hc thun ty v ton hc ng dng v
nh vy l ta c cc nh gii tch thun ty pht trin t tng ca h trong th gii trong th gii kt cu thun ty
l thuyt v cc nh gii tch s l nhng ngi chuyn cc qu trnh gii tch thnh cc php tnh ton trn my.
9
Cch ni i vi mt  > 0 bt k tm c s n, vi bt k  > 0, tm c tr thnh qu quen thuc
n ni ch  bin thnh du hiu dng nh ch mc cht ch Weierstrass.

44

Tp ch Epsilon, S 06, 12/2015

Quan im v hin i
Trong li gii thiu c ni v nt c th ton hc trong giai on hin nay, mt giai on
c lin h vi s m rng mt cch mnh m ngoi vi v khi lng ca nhng ng dng ton
hc v mt khc vi s xut hin v pht trin k thut tnh ton in t l ci lm tng nhiu
ln tnh hiu lc ca cc ng dng . R rng l ngay c bn trong ton hc trong nhng nm
gn y cng c nhng thnh tu xut sc, v sau chng ti s ni ti mt s nhng thnh tu
. Song nu xt s pht trin ca khoa hc chng hn nh trong 30 nm li y theo lp trng
ca nn vn minh ni chung th nhng thnh tu bn ngoi ca ton hc s thy l ln hn
nhiu. Ni cch khc, hin nay chnh s pht trin ca phng hng ng dng trong ton hc
(xt trong tt c nhng biu hin ca n) tr nn tri hn. R rng l trong thp nin ti y,
u th ny s c bo m v thm ch s cn c y mnh na.
iu cho php ni c rng sau nhng giai on pht trin ca ton hc m c th quy
c gi c l giai on tin hy Lp, giai on Hy Lp, phc hng v giai on l thuyt tp
hp, ta bc sang mt giai on mi v cht ca vic ton hc ho tng qut. C th quy
c nh du giai on u ca thi k ny l t nhng nm 40 ca th k XX (thi gian xy
dng nhng my tnh in t u tin). Bc chuyn mnh v cht trong ton hc v ang ko
theo nhng bin i v cht trong nhiu lnh vc ca khoa hc t nhin, k thut, ca cc khoa
hc v i sng x hi.
Nu dng cc thut ng trong lch s pht trin x hi th c th gi giai on hin nay l giai
on lng quyn (trong thi k qu sang giai on dn ch).
Mt mt ton hc ca phng php l thuyt tp hp vn tip tc pht trin. S kt hp, pht
trin v hnh thnh nhng quan nim mi ca i s hc, ca tp i cng v tp i s, ca
gii tch c in v gii tch hm, ca l thuyt hm v hnh hc cho php xy dng c
nhng chng mi, chn mui ca ton hc, cho php tin hnh nhng khi qut suy rng v tm
ra nhng phng php mi cho cc bi ton c v gii c nhiu nhng bi ton , cho
php nu ln cc lp nhng bi ton mi ng ch . Mt s kin ni bt l vic xy dng gio
trnh C s ton hc gm nhiu tp ca N. Bourbaki, mc ch ca gio trnh ny l xy dng
mt c s tng qut, mt ngun t liu bng mt ngn ng thng nht c th tip tc pht
trin c ton hc theo l thuyt tp hp. 10 N. Bourbaki cn cha cp n nhiu chng
ca ton hc (nhng tht v i!) v rt c th c nhng chng y s khng trnh c s
Bourbaki ho. Hin nay khuynh hng ny ca ton hc thun tu ang vang ln mt thanh
iu lt t cc thanh iu khc.
Tuy nhin trong nhng thp nin gn y ngi ta cn thy c nhng mt yu ca khuynh hng
ny. 11 iu biu th theo hai tuyn logic ton v nhng ng dng. Tuy nhin , nh thy,
chng li c lin h vi nhau. Trc ht, tnh cht ch tuyt i ca l thuyt tp hp ngy th
v nhng l thuyt ly n lm c s dng nh khng hon ton l tuyt i v iu ny c th
thy c t nhng lun c chung nht v n khng th khng cn n cc nh logic hc. Khng
phi mi ngi u ngh rng l thuyt tp hp ngy th khng c mt h thng hp l cc tin
10

Chi tit xem [34].


W.Spohn ni v s nguy him i vi ton hc hin i v xu hng nghing v pha cc chng tru tng
nht v kt lun rng : Hy cho ton hc quay v hng pht trin chnh ca n n c th tr thnh ca ci
chung ca mi ngi v chim mt v tr thch ng vi n vi tinh cch mt lc lng thng tr trong x hi ca
chng ta.
11

45

Tp ch Epsilon, S 06, 12/2015


lm c s v n ch da vo s d thy v nhng ng ngn cm tu tin, v d c php
coi tp hp tt c cc s t nhin l c lc lng nhng cm khng c coi tp hp tt c cc
s th t (siu hn) hay tp hp tt c cc bn s l c lc lng. Vic phn tch ca cc nh
logic v h thng cc tin ca l thuyt tp hp, mt h thng a ra mt th t no ,
chng minh rng mt s cc lp trng quan trng nht t n c gii quyt (v d tin
chn) m thc ra c th hoc chp nhn hoc khng v iu c th dn ti s cng tn ti
bnh ng cc mn ton hc l thuyt tng hp khc nhau, khng tng ng vi nhau ta
nh cc mn hnh hc khc nhau vy. S c lp ca gi thit continum vi cc tin khc ca
l thuyt tp hp c pht hin trn cc cng trnh ca K. Gidel v P. Cohen (1960) cond c
hiu lc hn : n c ngha l khi xy dng mt l thuyt nh vy th hoc c th chp nhn gi
thit ny nh l mt tin b sung, hoc chp nhn mt khng nh tri ngc nh mt tin ,
nhng t suy ra rng vn gi thit continum thc ra c ng hay khng li l v ngha !
(xem vn ny [39]). Ni ring, pht kin ni ting ny chng minh rng khi nim tn ti
m l thuyt tp hp ngy th bm chc vo thc ra khng phi t n c gii quyt.
Song theo quan im ang c pht trin th iu cn quan trng hn l mt yu t ca ton hc
l thuyt tp hp i vi nhng ng dng. Chng ta s khng than phin rng tt c nhng khng
nh ca n u thuc ci ngu nhin khi mt bi ton ng dng c chuyn hon ton sang
ngn ng ton hc. iu ti hn l trong nhiu trng hp, iu khng nh v nghim ch c
tnh cht thun tu v tn ti, tc l chng minh c nh l v s tn ti ca nghim ch trong
nh l khng h ni g n vic nghim c th tm thy mt cch chnh xc hay xp x.
Mc du i khi kt cu ca nghim c th suy ra c t php chng minh nh l nhng iu
ny thng gy tht bi, cho nn, ni vui mt cht th ngi ng dng ging nh vo hon
cnh ca mt ngi n ng ng k kt hn vi mt ngi ph n m anh ta cha h c
trng thy v cng khng bit nng u na. Theo cch ni rt ng ca H. Weyl th Ton hc
l mt kho tng khng l trong bn v tin giy. [40, tr. 106]. Trong nhiu trng hp, kt cu
c coi l kt qu thc hin mt qu trnh v hn ny hay khc m khng phn tch php xp x
hu hn ca n. Nu nh cng thc xp x dng gii c km theo s nh gi v sai s th, tr
nhng trng hp qu gin n hoc nhng v d c la chn c bit ging dy, s nh
gi y thng khng p dng c hu hiu trong nhng v d c th hin thc (y l cha ni
rng trong i b phn cc nh gi li c tnh cht gn ng v bao gm c nhng nhn t hng
s cha bit). Nhng tnh ton hng s, nht l trn my tnh in t c phn ng gp ca n
bi v thc hin s nh gi tuyt i cht ch v nh hng ca vic lm trn s trong nhng
php tnh cng knh t ra li rt kh khn v v vy s nh gi ny thc t cha h bao gi c
lm c.
Nhng tiu chun cng ngc v mc cht ch ca ton hc l thuyt hin i dn ti ch
lm tr tr khng ng mong mun i vi vic nghin cu cc khi nim ton hc m thot u
khng tho mn nhng tiu chun , v d nh hm denta, entropi, ...
Tt c nhng iu kin nhn xt trn y dn ti mt tnh hnh l trong i b phn nhng nghin
cu ng dng th nhng lp lun ton hc tuyt nhin khng mc suy din thun tu m ton
hc thun tu i hi, chng khng th v khng cn phi mc nh chng ti s ni chi tit
di y v vn ny. C th ni rng nhng kt cu suy din khng theo kp nhp iu ca
cuc sng hin nay 12 v trong qu trnh tip cn n chn l th chng thng c mt hiu sut
12

Nu nh ngay t u khoa hc c pht trin bi nhng b c cht ch v tinh vi c mt s cc nh ton

46

Tp ch Epsilon, S 06, 12/2015


qu thp n ni nhng nh ng dng t pht tm ra mt bin php hu hiu hn nhiu cho
cc lp lun ton hc 13 m chng ti s ni chi tit di. Nhng bin php lp lun mc cht
ch vt l, ng dng i vi cc nh ng dng thuc cc phng hng khc nhau l kh
ging nhau. Nhng bin php y cng c trng cho phong cch lp lun ca ton hc ng dng,
n cng ph bin trong cc nh ng dng nh l phong cch suy din ph bin trong cc nh
ton hc thun tu. chnh l iu cp ti phn trn khi ni v s lng quyn. 14
Cn phi c bit lu rng cc phng hng ng dng nh hng ng k n ton b ton
hc hin i ni chung k c phng hng l thuyt ca n. Ni ring chnh iu ny gii
thch s c bit ch n vn thut ton ho cng nh vn ti u ho c trng cho ton
hc hin i v c vt tch biu hin lnh vc a dng ca n.
kt lun chng ti xin a ra mt c trng ni bt ca pht trin ton hc bt u t thi k
phc hng trong li ni u ca cun sch R. Courout v G. Robin. [28] Sau thi k tch lu lc
lng mt cch chm chp vo th k 17 cng vi s ra i ca hnh hc gii tch v php tnh
vi tch phn m ra mt thi k cch mng nh v bo trong pht trin ca ton hc v vt
l. Trong nhng th k XVII v XVIII, t tng ca ngi Hy Lp v s kt tinh theo tin v
s suy din mt cch c h thng m dn v mt nh hng ca n d rng hnh hc c i
vn tip tc c nhng bng hoa n r. T tng tuyt vi v logic xut pht t nhng nh ngha
r nt v nhng tin hin nhin khng mu thun g nhau ngng nhp cng vo nhng
ngi khai ph mi nhng tri thc ton hc. Ham m cuc say sa chn chnh vi nhng thng
on trc quan, chuyn dch nhng kt lun bt kh xm phm vi nhng li khng nh na
huyn b v v ngha, tin tng m qung vo lc lng siu nhn ca cc th tc hnh thc, h
khm ph c mt th gii ton hc mi v cng phong ph. Nhng ri t mt, trng thi
nhp nh ca t tng b ham m bi nhng thnh tu lm chng mt nhng ch cho tinh
thn nhn ni v ph phn. Trong th k XIX, thc v s cn thit phi cng c khoa hc, nht
l trong mi lin h vi nhng yu cu ca nn gio dc cao ng ph bin rng sau cch mng
Php dn n vic xt li nhng c s ca mn ton hc mi, ni ring, s ch hng v
cc php tnh vi tch phn v tch phn nhm lm sng t khi nim gii hn ngm hiu trong
gii tch. Nh vy th k XIX khng nhng ch l mt thi i ca cc thnh tu mi m cn l
thi i chng kin s hon li tnh chnh xc v cht ch ca cc php chng minh l tng
hc hin nay m ti rt ngng m th chnh xc s khng cho php c y mnh ln L.J.Mandeishtam ni
nh vy. [42, tr. 51] Ph ho thm l li pht biu ca Kapys.P.L [40, tr. 30] : T duy logic sc bn, thuc tnh ca
cc nh ton hc, khi tin ho nhng c s mi th lp tc gy nn phin toi v n lm te lit c tng tng.
13
V thm ch c mt ngn ng ring, trong cc thut ng S hi t thc tin, cht lng ca phng
php tnh, cht lng ca m hnh ton hc, ...
14
V.V.Novozhilov [44] vit : Hin nay ngy cng thy r khuynh hng phn cng lao ng: C nhng nh bc
hc ang hon thin ton hc theo hng logic pht trin ni ti ca n, c cc nh khoa hc khc li nghin cu
cc phng php ton hc v c p dng mt cch mau chng. Nhng i biu ca hai phng hng ny r rng
khc nhau khng nhng ch nhm cc vn ca h m cn t cht ca t duy.
Nh ton hc l thuyt tm mi cch lp lun mt cch cht ch nht cho mi mt hnh ng v lun lun b
trn trong khun kh khoa hc ca mnh trong khi cc nh ton hc ng dng li dng cc phng php m tnh
cht ch v sai cn cha c nh gi. Kh nng trnh sai st ca anh ta l da vo kinh nghim, so snh cc
kt qu tnh ton vi thc nghim. Anh ta cn lm nh vy v i b phn cc bi ton hin nay trong vt l v k
thut u khng c gii quyt mt cch cht ch. V nhng bi ton hc a dng ny sinh trong cng tc thc
tin ca ngi k s hin nay, cng nh v c im ca cc phng php ton hc tng ng c c ni n
trong tuyn tp [45], ni ring xin lu nhng bi bo c tnh cht tng hp ca N.F.Garasjuta. Ju.A.Mitropolski v
A.V.Skorokhod. V mi tng quan gia ton hc v k thut, xem thm [46]. V mi tng quan gia ton hc v
vt l hin nay, xem thm [47-49]. V trin vng pht trin ca ton hc, xem thm [50].

47

Tp ch Epsilon, S 06, 12/2015


c in. V mt ny th hnh nh Hy Lp qu l c tri hn ln. Li mt ln na con lc li a
v pha tnh tuyt vi logic v tnh tru tng. C l hin nay chng ta cha bc ra khi thi
k d rng c php hy vng l s gin on ng tic gia ton hc thun tu v nhng
ng dng sinh ng ca n s c thay th bi mt k nguyn hp nht cht ch hn. Phn d
tr c ca cc lc lng bn trong v cng vi s cc k gin lc khc t c trn c s
nhng quan nim r rng cho php ngy nay nghin cu c l thuyt ton hc m khng tch
ri khi nhng ng dng. Xc lp mt ln na mi lin h hu c gia tri thc thun tu v tri
thc ng dng, s cn bng lnh mnh gia tnh tng qut tru tng v tnh c th sinh ng
l iu chng ti thy nhim v ca ton hc trong mt tng lai khng xa. 15

Trong ton hc gm c nhng g?


Ton hc ng dng l ci g? v ni chung n c tn ti hay khng? Nhng vn ny n
lc cn phi tho lun mt cch trit . ng ch l thut ng ton hc ng dng hin nay
tr thnh mt thut ng rt mt (nht l nhng ngi khng phi chuyn gia).
C l quan im ph bin nht i vi khi nim ton hc ng dung trong hng ng cc nh
ton hc l quan im cho rng ni chung khng c ton hc ng dng. Ngoi ra, cc nh ton
hc khc nhau li cho nhng t ny mt ni dung hon ton khc tu theo cc nh ton hc y
c gia nhp vo bn thn mn ton hc hay khng. 16
C nhng ngi cho rng ch nhng kt cu thun tu suy din mi c gi l ton hc. Tt c
nhng g nm ngoi kt cu , khng c quan h vi ton hc hoc nhng b mn ca ton hc
th cng khng c gi l ton hc. 17 Hin nay quan im ny t c pht biu m song mt
cch khng chnh thc n vn cn kh ph bin, bn cnh nhng vic khc, quan im ny t
ra thun tin cho nhiu ngi dy ton vi nhng ngi khng phi cc nh ton hc.
Thc t quan im ny thu hp mt cch v l v ng k ranh gii ca Khoa hc Ton hc v
i v trc ht mang li ci bt li cho chnh mn Ton hc (v tt nhin cho chnh s nghip
o to cc nh bc hc tr). V vn ny M. Las v S. Ulam vit : Nhng c gng tic
rng li kh ph bin nhm tch ri ton hc thun tu khi ton b phn hot ng khoa
hc cn li v nhm cht n li nu thnh mt loi nc ct c bit ch lm ngho nn c
ton hc ln cc mn khoa hc khc. [13, tr. 234].
F. Klein cng pht biu mt t tng nh vy : Nhng quan nim logic thun tu phi to
ra, nh ngi ta vn ni, mt b xng rn chc ca c th ton hc, m bo cho n n nh
15

A.N.Kolmorokov c ni rng c m m ng theo ui t lu l thanh ton phn cch gia nhng phng
php cht ch ca cc nh ton hc thun tu v nhng bin php khng cht ch ca nhng lp lun ton hc
ca cc nh ton hc ng dng, cc nh vt l v cc k thut (dn theo [51]).
16
Khi lit k cc quan im cc phn 6 v 7 ni ring dng nhng kin ni ming ca M.A.Krasnoselski.
17
y l mt cu ni cc oan nht ca lp trng ny Ton hc l s xy dng mt l tr thun tu v v vy n
khng cn cc mi lin h vi nhng lnh vc hot ng khc ca con ngi (L.Mordell [52, tr. 28]). Xin dn thm
li pht biu ca J.Diendonne v vn ny (dn theo [53, tr. 18]) : ... v nguyn tc, trong c s ton hc hin
i khng c mt mc ch hu dng no v l mt b mn tr lc m li ch thc tin quy v s khn, nh ton
hc trong cc cng trnh nghin cu ca mnh khng h vng vn t tng v mc hu ch ca cc kt qu t
c trong tng lai (hn na iu cng khng th thy trc c) m ch theo ui c mun hiu c hin
tng ton hc, mt hin tng t kt thc chnh n, ton hc khng hn g s xa x m mt nn vn minh c
th t cho php mnh. Rt tic rng iu li c ni ra mt ngi, mt trong nhng ngi lnh o nhm
Bourbaki v c nh hng ng k n b mt ca ton b ton hc hin i.

48

Tp ch Epsilon, S 06, 12/2015


v ng tin cy. Nhng bn thn cuc sng ca ton hc, nhng quy np quan trng nht v tnh
tch cc ca n li l ch yu lin h vi nhng ng dng ca n, tc l vi mi tng quan gia
nhng khch th tru tng vi ton b cc lnh vc khc. Loi tr nhng ng dng khi ton
hc chng khc g i tm mt thc th sng ch t mt hi ct, khng bp tht, khng thn kinh,
khng mch mu. [29, tr.46].
Cui cng chng ti xin dn li ca A. Poincar: Vt l hc khng nhng ch cho chng ta (cc
nh ton hc tc gi) ci l gii quyt vn , n cn gip chng ta tm thy cc phng
tin gii quyt vn na. iu ny xy ra theo hai con ng. Mt l n cho ta linh cm ca
php gii, hai l gi cho ta tin trnh ca cc lp lun. [54, tr.108].
Thc cht y biu th mt quan im th hai m chng ti cho rng d dng chp nhn
c hn, quan im cho rng gia nhp vo lnh vc hot ng ca ton hc cn c c cc
phng php gii thc tin i vi cc bi ton bt ngun t bn ngoi ton hc.
Song chng ta cn phn khi hn vi mt quan im th ba, rng nht, cho rng ton hc khng
nhng ch bao hm cc lnh vc suy din m cn bao hm ton b nhng thc cht ton hc
cc khch th ton hc, cc phng php v t tng gp trong ton hc l thuyt cng nh trong
cc ng dng : tc l kt cu cc m hnh ton hc, thc nghim ton hc, nhng lp lun quy
np hay nhng lp lun hp l khc c tnh cht ton hc, ...
Trong cun sch rt hay ca G. Polya [56, tr.309] c ni rng : Gii hn ca ton hc l min
nhng lp lun chng minh thuc bt k khoa hc no t mc pht trin l nhng khi nim
thuc khoa hc y c th biu th c di dng logic ton tru tng. Chng ti mun thm
vo y l trong khi nim chng minh khng nn gn cho n mt ni dung gio iu, hp hi.
Tt nhin nhng mn sinh ca quan nim ny, mt quan nim m chng ti cho rng tin b v
chn mui hn c i vi ton hc (v iu cng kh c bn i vi cc nh ton hc) phi
khc t tnh thng nht l thuyt tp hp ca ton hc m ch coi n nh mt ht nhn no
ca ton hc m thi.

Cc quan im v ton hc ng dng


Trc ht chng ti ly lm bun ru m lu rng theo kin ca mt s nh ton hc th
nghin cu nhng ng dng ni chung l mt iu h thn.
V vn ny F. Klein vit : Rt tic rng ... vn cn gp nhng gio vin i hc khng
che du nhng li l qu ngo mn i vi nhng ngi lm v ng dng. Cn phi u tranh
quyt lit nht vi ci tnh kiu cng th hin trong nhng quan im . Mi thnh tu xc
ng, cho d n thuc lnh vc l thuyt hay ng dng, u phi c nh gi cao nh nhau v
n to cho mi ngi kh nng nghin cu nhng vt m mnh cm thy gn gi hn c. Khi
mi ngi s biu hin a dng hn s ln nhng ti nng mnh c : v nhng thin ti v i
nh Aschimede, Newton, Gauss l nhng ngi lun lun chim lnh c l thuyt ln thc tin
nh nhau. [57, tr.314].
Xin dn thm li ca R. Courant : Thc ra gia ton hc thun tu v ton hc ng dng
khng th vch ra mt ranh gii r rt c. V vy trong ton hc khng th phn ra mt lp
ngi thy t ti cao thin v ci v p hon thin ca ton hc v ch ch n thin hng
49

Tp ch Epsilon, S 06, 12/2015


ca mnh, v nhng ngi phc v cho h. S phn ng cp trong trng hp tt nht
cng ch l mt triu chng ca nhng b c hp hi. [12, tr.27]. N.Bailey cng ni v s bt
li ca s ua i y trong cun sch hu ch ca ng [58, tr.138] khi ni v nhng ng dng ca
ton hc trong vo sinh vt v y hc.
V.V. Novozbilov vit : Tic rng nh l thuyt cho n nay vn thng coi nh ng dng nh
mt nh ton hc loi hai, mt nh khoa hc khng c kh nng lm vic mt cch tht cht ch
v ly ci ring lm tn tht ci chung. D dng pht hin cc nh ng dng nhng sai st
trong lp lun xt v tnh cht ch, nh l thuyt thng t ra th vi nhng thnh tu ca h l vic h bit cch gii nhng bi ton thc ti vi mt s chnh xc dng cho nhng mc
ch thc tin m bn thn nh l thuyt khng th gii c bng cc phng php cht ch.
[44]
Qua nhng cu trch dn trn ta thy kh ni bt kha cnh tm l ca vn . Nhng c lp vi
iu tng cn phi nhn mnh rng hin nay ngi ta ngy cng tha nhn s tn ti khch
quan ca ton hc ng dng. Song ng sau s tha nhn cn nhiu quan im khc nhau.
V d mt s ngi cho rng ton hc ng dng l mt b phn ton hc thng dng hng
ngy vi ngha xu ca t , tn ti di dng mt s cc bin php, cc n thuc v quy
tc thiu cht ch v khng hon thin v mt logic (c th l do trnh ton hc cn thp ca
cc chuyn gia trong lnh vc ny). Nhng thiu st ca ton hc ng dng c th khc phc
c v kt qu s l ci ton hc khuyt s c nng ln mc ton hc tiu chun. 18
Chng ti cho rng quan im ngy th nhng ph bin ny, nu nh n khng phi mt biu
hin ca s ua i th cng l da trn mt c s khng nhn thc c tnh hnh ng n ca
vn . Thc ra theo quan im ny th lm th no c th gii thch c mt s vic l cc nh
vt l, cc k s nghin cu l thuyt v cc chuyn gia khc trong r rng c khng t nhng
ngi kh thng minh khi p dng ton hc c tnh n trnh ngn ng suy din cht ch? V
mc du cc hc vin h c hc ngn ng mt cch c h thng song h vn mun
(liu c hi chng?) c hc li khi chuyn sang ngn ng ca ton hc ng dng v xy dng
li ton b phong cch t duy suy din thun tu i hi nht thit phi c. Chng ti ngh rng
s xy dng li hon ton l t nhin v iu gii thch duy nht cho n l: S xy dng li
l cn thit. Di y chng ti s c gng chng minh rng s vng mt yu cu nht thit phi
c v s hon thin logic hnh thc trong nhng ng dng ca ton hc l iu khng th trnh
c, v khng phi l mt du hiu ca s thua km m l mt ngun sc mnh c bit ca
ton hc ng dng.
Mt quan nim khc ng nht ton hc ng dng vi ton hc tnh ton v ton hc my
tnh. y l mt quan im hp hi v c xu hng phin din.
By gi chng ti xin cp n mt quan im c pht biu trong mt bi bo ca chng
ti [2]. Xut pht im ca chng ti l cho rng php gii ton hc nhng bi ton ng dng c
18

i khi quan im ny th hin s phn i nhng cng trnh, v tuyt nhin, khng phi l him, trong s
nng ni v ton hc dn ti nhng sai lm trc tip (m di y chng ti s ni mt cch chi tit) hoc cn ti
hn, nhng cng trnh trong s km ci v ton hc c n b bng cch dn ra cc ngha thc dng
ca cc kt qu. Tic rng quan im ny i khi cng c thc nhng nh ng dng khi h dng mt tng th
no khng cn c gi tr y . iu ny li dn n nhng ng tc c v khoa hc rt l lng m thng cn
l tro phng na.

50

Tp ch Epsilon, S 06, 12/2015


nhng nt c th ca n. y, trc ht v mt nguyn tc th khng th tin hnh php chng
minh mc cc cng trnh nghin cu ton hc thun tu t ra bi v m hnh ton hc ca mt
khch th thc ti ch c th m t nhng nt c bn theo ngha ny hay khc ca khch th y
ch khng khi no c nh v khng th c nh m t n mt cch y . Mt khc trong
vic gii nhng bi ton thc tin th trc ht c nhng yu cu m trong nghin cu ton hc
thun tu ch c coi l nhng yu cu th cp: Bi v mt bi ton ng dng phi c gii
khng nhng ng m cn phi kp thi, tit kim sc lc, nghim phi chp nhn c i vi
cc phng tin tnh ton hin c v phi thun tin trong s dng thc tin v phi c chnh
xc thch hp vi bi ton, ... 19
Chng ti quy nh gi vic thc hin tt nht tt c nhng yu cu m i khi cn mu thun
nhau na l tnh ti u ca nghim (i vi nhng ng dng) d phi lu trc rng trong giai
on pht trin hin nay ca khoa hc th kh m ch ra c mt chc nng duy nht v mc
ch. Xut pht t iu ngh mt nh ngha l: Ton hc ng dng l khoa hc v cc
phng php gii ti u, m v thc tin l chp nhn c, nhng bi ton hc ny sinh t bn
ngoi ton hc. Nh vy, ton hc ng dng l ton hc b gin tip bi thc tin, mt b phn
khoa hc hp thnh ta nh sinh ho hay nhit k thut. S pht trin ca b mn ny c xc
nh bi s m rng nhm nhng ng dng cng nh bi s thay i ni dung c th ca khi
nim tnh ti u ca php gii bi ton, ni ring, ni dung ny hon ton thay i do nh hng
ca cc phng tin tnh ton hin nay. Tt nhin nu chng ta tm thy nghim ti u th iu
khng c ngha l ta loi b nhng nghim ch p ng gn ng yu cu ca tnh ti u. Phn
ln cc nghim thc ti m chng ta dng th cng l nhng nghim m trong mt thi gian no
, mt mc no tho mn yu cu .
V vn ny ta c th nh n mt cu cch ngn ni ting : Ton hc thun tu lm ci c
th khi cn cn ton ng dng lm ci cn khi c th . 20 Cu cch ngn truyn i mt xu
hng ni chung l ng, d rng t cn c dng y theo nhng ngha khc nhau. Ch
n ngha th hai, di y chng ti s c gng chng minh rng ton hc ng dng lm ci
cn khi cn lm.
Cng ng ch n mt quan im c L.V. Ovsjannikov pht biu bng li: Ton hc ng
dng l khoa hc v cc m hnh ton hc, chi tit hn, c th ni rng, l khoa hc v cc kt
cu, nghin cu, din t v ti u ho cc m hnh ton hc. nh ngha ny nhm vo i tng
ca khoa hc v theo chng ti th khng mu thun g vi nh ngha trn l nh ngha nng
v tnh cht chc nng hn. Nh vy, nu mun so snh tng t - ni chung cng kh xa gia
ton hc v ngn ng th ton hc thun tu v ton hc ng dng c th s lm ngi ta ln
lt nh n vn phm v ng ngha.
Bn v vn ton hc ng dng c to thnh mt khoa hc c lp khng l vic lm c cht t
kinh nghim v do tnh nhiu ngha ca cch ni khoa hc c lp th ng n hn c th
19

Vi tinh thn ny, I.Babuska, E.Vitasek v M.Phager vit trong mt cun sch rt hay [59, tr.9] : ... ngy
nay, mt bi ton ch c coi l gii c khi c mt phng php hiu lc cho mt kt qu cn thit vi mt
chnh xc d dng trong mt khong thi gian nht nh. Trong cun sch ca N.S.Bakhvalov [60, tr.14] c vit
vi mt quan nim su sc v nhng tnh hung ng dng thc ti c ni rng: Tm thy mt nghim tho mn
bi ton m kp thi th tt hn l c c nghim y ca bi ton vo lc n mt ngha.
20
V im ny xin dn li ca J. Dixon: Nu nhng phng php y sc mnh ca ton hc khng cho php
thu c kt qu i khi xy ra th cn phi tip tc tm ti. Nn nh rng khi phn tch v k thut th cn c
c mt kt qu bng s bng bt k phng php no. [20, tr. 78].

51

Tp ch Epsilon, S 06, 12/2015


khng nn ni v mt khoa hc m l v mt kha cnh ca ton hc ra i trong nhng ng
dng ca n, v nu c th, th nn ni v kt qu ca php chiu ton hc mt cch c o
ln nn vn minh, iu quan trng l vi php chiu th ton hc c nhng nt mi v cht v
php chiu y, nhng nt y cng s nh ngha cho ton hc ng dng.
Do chng ti s dng cc t ton hc ng dng coi nh mt thut ng lm vic c xc
nh bi quan im cui cng nu trn v dnh vn v tnh c lp ca s tn ti ton hc
ng dng vi tnh cch mt khoa hc cho cc nh trit hc. 21 phn bit vi iu , khi
ni v ton hc thun tu, chng ti trc ht s quan nim rng l ton hc chnh thng t
Weierstrass n Bourbaki da trn c s l thuyt tp hp ngy th. 22
Chng ti ch yu ch n nhng vn c th hn : l nhng nt c trng ny sinh trong
ng dng ca ton hc, c im ca phng php lp lun ca ton hc ng dng v ni ring
l nhng lp lun no c tha nhn l c chng minh trong ton hc ng dng , ...
Vic tho lun nhng vn ny c th l hu ch, thm ch kh si si ni trong c cc cng
trnh hon ton c tnh cht c th.
kt lun, chng ti a ra nhng li ni sng sa R. Courant ni v s khc nhau trong
phng php tip cn cc vn ca ton hc thun tu v ton hc ng dng v cng dng lm
phn gii thiu c o cho phn trnh by tip sau ca chng ti.
Cng mt vn ton hc c th c gii quyt khc nhau, ngi theo quan nim ton hc
cht ch (v khuynh hng ny i khi cn thy mi ngi nghin v t duy khoa hc) th i
hi mt s hon thin khng nhn nhng. Anh ta khng cho php c nhng l hng trong logic
ca t duy v trong cch gii cc bi ton c t ra, v kt qu t c theo anh ta phi l
mt nh cao ca mt mt xch lin tc nhng lp lun ton thin. V nu nh i phng ca
quan im ny m gp nhng kh khn dng nh khng khc phc ni th anh ta s mau chng
tm cch pht biu li bi ton hoc thm ch t n khc i nhng cng loi vi bi ton c,
trong c th khc phc c nhng kh khn (ci c th khi cn tc gi). Cn c mt con
ng vng khc na, xc nh li xem ci g c coi l nghim ca bi ton. Trong thc t,
cch lm ny i khi l mt bc s b kh c chp nhn i ti nghim chn chnh ca bi
ton ban u.
Trong cc cng trnh nghin cu c tnh cht ng dng th mi th u khc. Trc ht khng
th d dng lm thay i hoc lng trnh bi ton c t ra. y i hi mt ci khc l
a ra mt cu tr li ng n v ng tin cy theo quan im chung ca ngi ta. Trong trng
hp cn thit, nh ton hc c th c nhn nhng: Anh ta phi sn sng a nhng d on vo
21

V im ny cn c mt kh khn ph na l nhng khi nim nghin cu ng dng, chng ng dng, ...


ch l tng i, iu i khi dn n s hiu lm khc nhau. V d dn n ch c nhng ngi khng h l nh
vy nhng cng gi nhau l nhng nh ng dng (v tng ng l nhng nh l thuyt). C nhiu nhng cng trnh
nghin cu cc cun sch c th gi c l ng dng (nu chng c xem xt theo nhng lp trng tru tng
hn) cng nh gi c l ton hc thun tu (gi s theo lp trng ca mt k s). Tt nhin tnh tng i ca
khi nim ng dng cng c trong vt l, c hc v cc b mn khoa hc khc. V khi nim v trin vng pht
trin ca ton hc ng dng xem thm [61-72].
22
Cn nhn mnh rng vic tch ra ton hc thun tu v ton hc ng dng l khng c tnh cht tuyt i v
thc cht l nhng kha cnh khc nhau ca mt khoa hc vn gi nguyn cc nt quan trng v s thng nht
(trc ht ch yu i tng nghin cu l cc cu trc, nhng cng khng phi cng ch c vy). Trong mi mt
kha cnh ny li ny sinh nhng t tng su sc tc ng ln nhau mt cch tch cc (v vy, ni ton hc thun
tu l khng t m nn ni ton hc l thuyt). Song s tng tc d cn lu mi ti u c.

52

Tp ch Epsilon, S 06, 12/2015


xch cc lp lun cng nh cho php mt sai s nht nh trong nhng gi tr bng s. Nhng
ngay c nhng bi ton ch yu theo phng hng thc tin, v d bi ton v cc dng c sng
va chm, cng c th i hi mt cng trnh nghin cu ton hc c bn xc nh xem bi
ton c t ra c ng hay khng. Trong cc cng trnh nghin cu ng dng c th i hi
c nhng php chng minh nhng nh l ton hc thun tu v s tn ti bi v s tin tng l
c nghim c th m bo cho tin cy ca m hnh ton hc c s dng, (thc ra iu ny
c phc tp hn i cht v v sau chng ti s ni li vn ny tc gi). V cui cng, ch
ng trong ton hc ng dng l cc php xp x v thiu chng th khng th chuyn c cc
qu trnh vt l thc ti thnh cc m hnh ton hc.
Vic quay li vi hin thc c bin i thnh cc m hnh ton hc tru tng v s nh
gi nhng s tng ng t c y i hi phi c nhng thi quen trc quan hon thin
qua kinh nghim. Thng cn phi bin i nh th no i vi bi ton hc lc u t ra rt
phc tp c th gii c bng cc phng php hin i. iu ny phn no gii thch c
tnh cht ri ro v tr c v s tho mn c cc nh ton hc lm vic vi cc k s v cc nh
khoa hc t nhin gii cc bi ton hin thc c khp ni, ti con ngi tm cch nhn
thc thin nhin v iu khin n.

53

Tp ch Epsilon, S 06, 12/2015

54

CUC I V S NGHIP CA GEORGE BOOLE


S KHI U CHO K NGUYN K THUT S
Desmond MacHale
(Dch gi: Hong Cao Phong, H Ni)

i hc Cork hin ang k nim 200 nm ngy sinh nh ton hc v logic hc George Boole. T
thng 10 nm 2015; i hc Cork ln k hoch cho mt lot cc s kin trong nm dnh cho
George Boole 1 . Sc nh hng trong nhng cng trnh ca Boole ang ngy cng tr nn mnh
lit trong vi thp nin tr li y, t khoa hc my tnh cho n cng ngh k thut s.
Cun tiu s mi ca Boole vit bi Desmond MacHale va c xut bn bi Nh xut bn
i hc Cork. y l bn chnh sa ca phin bn nm 1985 xut bn bi Nh xut bn Boole.
Li ta ca phin bn trc c vit bi nh ton vt l hc ni ting ngi Ai-len - John L.
Synge. Ln ny, cun sch c mt li ta mi c vit bi Ian Stewart ca i hc Warwick.
n bn u tin ny c bin tp bi Ivor Grattan-Guinness 2 .
y l mt cun tiu s rt ton din, ng gp v cng to ln cho s hiu bit ca chng ta v
mt nh ton hc v logic hc c tm nh hng ln ca th k XIX. N khc ha mt cch y
bc chn dung ca Boole, khng ch l mt con ngi ca khoa hc m cn l mt nh ci
cch x hi, nh t tng tn gio v mt ngi n ng ca gia nh. ng cng l mt trit gia
tinh ty. Cun sch l kt qu ca nhng nghin cu v tm hiu k lng, cng phu, k li nhng
cu chuyn v cuc sng v s nghip ca Boole mt cch c h thng vi s cun ht bn b.

1. Thu thiu thi


George l con c trong mt gia nh c 4 con, ng l mt thanh nin nht nht nhng c bit
thng minh. T thi nin thiu, ng c th s dng thnh tho ting Latin v ting Hy Lp,
ngoi ra ng cn t hc ting Php, ting c v ting . iu ny gip ng sau ny c th d
1
2

http://www.georgeboole.com
Vol. 14, Sept. 1985, http://www.maths.tcd.ie/pub/ims/nl14

55

Tp ch Epsilon, S 06, 12/2015


dng tip cn vi s pht trin ca nn ton hc trn ton chu lc, cho php ng tin xa hn hu
ht nhng ng nghip ca mnh. ng hc qua lp tiu hc, nhng do qu ngho nn khng
th tip tc bc trung hc; chnh v vy, ng phi t hc nhng kin thc ton cao cp bng cch
nghin cu cc tc phm ca nhng nh ton hc hng u thi gian .
Boole bt u nghin cu ton hc mt cch thc s nghim tc khi ng khong 16 tui. Sau khi
thnh tho gii tch, ng nghin cu sang cc tc phm ca Newton, Lagrange, Laplace, Jacobi
v Poisson. Nh l mt ngi t hc, Boole to ra mt phng php c lp khi tip cn
nhng vn nghin cu. Cm hng ban u ca ng vi ton hc n t ng dng ca n nhm
gii quyt nhng vn khoa hc khc, nhng dn dn, ng b quyn r bi v p v s th v
ca ton hc thun ty. ng cng b mt s cng trnh v phng trnh vi phn, tch phn,
logic, xc sut, hnh hc v i s tuyn tnh.
Nm 1841, cun sch L thuyt tng qut v php bin i tuyn tnh ca Boole t nn mng
cho s pht trin ca mt ngnh ton hc mi - l thuyt bt bin. Cayley v Sylvester l nhng
ngi pht trin l thuyt bt bin ln mt tm cao mi, nhng c hai u tha nhn Boole
khi ngun cm hng cho nhng n lc ca h.
Ti liu V mt phng php tng qut ca gii tch ca ng c xut bn vo nm 1844
trong tuyn tp cc cng trnh trit hc ca Hi Hong gia. Cng nh vy, ng ginh c
Huy chng vng ca Hi, gii thng u tin nh vy trong lnh vc ton hc. Bi bo gii
thiu mt phng php tng qut ha cch gii quyt mt lp ln nhng phng trnh vi phn v
sai phn vi h s thay i.

2. i hc Cork Queen
Thng 10 nm 1846; Boole ng tuyn cho v tr gio s ton hc ti mt trong ba trng i hc
Queen: Belfast, Galway v Cork. n xin xt tuyn ca Boole lm hi ng ht sc sng st
bi ng tuyn b: Ti khng phi l thnh vin ca bt k trng i hc no v cha tng hc
ti mt trng i hc no. Tuy vy, n ca ng nhn c s ng h rt mnh m t cc nh
ton hc hng u. Sau thi gian tr hon, ng c giao ph mt v tr trong i hc Cork v
lm vic vo thng 10 nm 1849 vi mc lng khi im 250 bng Anh mt nm.
Cc i hc Queen u l nhng c s a gio phi, v do vy gy nn nhng cuc tranh ci t
u khi b h thng Cng gio m t nh l nhng trng i hc v thn. Trong mt thi gian,
Boole c gng trnh tham gia trc tip vo cc cuc xung t tn gio, tuy nhin ng khng th
khng b nh hng. Mc d bn cht ha nh, ng cng dnh vo mt lot cc cuc tranh
lun gay gt trng i hc. C l nhiu thng tin chi tit c cung cp bi MacHale hn l
thc s cn thit, t nhng l th di n cc t bo trch dn.
Trong nhng nm u Cork, Boole rt c n v dng nh khng my hnh phc. Tuy nhin,
mi th hon ton thay i vo nm 1855 khi ng kt hn vi Mary Everest 23 tui lc tui
40: Mary l chu gi ca v gio s ngi Hy Lp ti i hc Queen v ng thi l chu gi ca
George Everest, nh tng trc a hc ca n , cng l ngi c t tn cho ngn ni
cao nht th gii. Mt cch ngn gn, cuc hn nhn rt hnh phc v h c nm ngi con
gi, c nm u ti gii hn ngi theo mt cch no .
56

Tp ch Epsilon, S 06, 12/2015

3. Cc nh lut ca t duy
Nm 1833; khi mi 18 tui, Boole c mt tng thong qua rng tt c nhng quan h logic
u c th biu din di dng cc cng thc. Chnh tng ny sau tr thnh cng hin
to ln ca ng i vi ngnh khoa hc: Gii thch cho qu trnh suy ngh ca con ngi bng
nhng ngn ng ton hc chnh xc. Nhng n lc bin logic thnh mt b mn khoa hc
khi ngun t Aristotle, v Leibniz t nhng bc u tin trn con ng th hin mi quan
h logic di dng mc d ng cha tm ra mt k hiu ph hp. Nm 1847; Boole vit cun
sch Phn tch v Logic hc, c miu t vi ta l Php tnh cho cc lp lun hp l.
Cun sch ny nh du cho s khi u ca logic hnh thc.
Tc phm ln nht ca Boole, Cc nh lut ca t tng, c vit trong sut thi gian
ng i hc Queen. Mt trong nhng nhn xt su sc nht ca ng chnh l ton hc khng
h gii hn bi con s hay s lng, m mang mt ngha ln hn v cch biu din vn vt
di dng thc ph hp vi mt s quy tc nht nh. Mc tiu ca ng l chuyn ha cc mnh
logic thnh dng cng thc v t , nhng kt lun logic tr thnh kt qu ca nhng php
ton vi cc gi nh ban u. Vic xem xt cc lp thay v ch tp trung vo nhng con s m
ng cho l thuyt tp hp, hin l tm im ca nn tng ton hc.
Boole l ngi u tin dng cc php ton i s biu din thay cho cc mnh logic, t
to ra mt ng lc mnh m cho lnh vc logic hnh thc. Trc y, khng c ai nh gi cao
bn cht ton hc xut hin trong ngn ng hng ngy. Ngnh i s hc pht trin bi Boole
hin nay tr thnh cng c l tng trong vic x l thng tin v hot ng my tnh da theo
cc nguyn tc ca n. i s Boolean bao gm rt nhiu ch , trong c l thuyt tp hp,
s nh phn, khng gian xc sut, cu trc mch in t v cng ngh my tnh. Nhiu tng
ca Boole hin gi c coi nh hin nhin v c tm thy trong l thuyt tp hp v xc sut
s cp. N mang li nhng ng dng rng ri trong cc lnh vc nh chn on y t, bo him v
chng c php l.

4. Boole v Hamilton
Boole tng th lm th, tuy nhin ging vi ngi bn ca mnh - William Rowan Hamilton,
nhng tc phm th ca ng khng c g c sc. C mt chng c dnh ring ni v s
thiu lin lc ng ngc nhin gia hai ngi n ng. Boole sinh sau Hamilton 10 nm v cng
qua i trong mt nm. H c rt nhiu s thch chung trong c ton hc ln nim lut th, v
cng c rt nhiu c hi c th cng tc hay t nht l tng tc. Tuy nhin, s hn ch tip
xc gia hai ngi cho thy gia h c th c mt s kh khn ng k hoc mt s bt ng
quan im, mc d khng c bng chng r rng no v iu ny. Vo nm 1985; MacHale vit
rng nhng b mt v mi quan h gia h vn cha c bt m. Chng ny c tiu Mt s
cu hi cha c tr li. Tuy nhin, iu ng tht vng l sau hn 30 nm, khng h c thm
thng tin no v iu ny.3

MacHale ni vi ti rng anh y c mt suy on mi v s rn nt trong mi quan h, th nhng chng ta


bt buc phi kin nhn ch i cho n khi thng tin v iu ny xut hin u .

57

Tp ch Epsilon, S 06, 12/2015

5. Gia nh
Chng cui cng kh l th v mc d l thng tin ngoi l. George v Mary Boole c 5 ngi
con gi, tt c u c bit theo nhng cch khc nhau. Alicia c nhng khm ph quan trng
trong hnh hc 4 chiu. C a vo thut ng polytope ch s tng ng 4 chiu ca mt
khi a din. GI Taylor, nh thy kh ng lc hc tin phong ca th k 20; l con ca con gi
Boole-Margaret. Mt ngi con gi khc, Ethyl Lilian (Voynich) sng mt cuc i phiu lu v
l tc gi cun tiu thuyt Rui Tru v cng ph bin Nga. Cu chuyn v cuc i c cng
rt hp dn.
Kt thc chng cui- nhng nm thng cui cng, MacHale vit v Boole: Tn ca ng s
sng mi ta nh nhng my tnh k thut s lun ph thuc vo i s Boolean vn hnh, v
min l khi cc sinh vin ton hc vn cn nghin cu l thuyt vnh, phng trnh vi phn, l
thuyt xc sut, phng trnh sai phn, l thuyt bt bin, l thuyt ton t, l thuyt tp hp v
tt nhin, cun SCH 101 logic. ng kt bt bng ghi ch rng Boole s v cng vui mng khi
c bit tt c cc thng tin lin lc hin i, cho d l d liu, vn bn hay hnh nh, lun
bao gm cc chui di ca nhng biu tng Boolean 0 v 1:

6. Li kt
Cun sch c xut bn mt cch rt k cng v dng nh khng c li. Mt vn nh c
th lm ngi c kh hiu l vic nhc i nhc li v Lebesgue. Nh phn tch Henri Lebesgue
cha c sinh ra cho n nm 1875; rt lu sau ci cht ca Boole. Ti liu tham kho l ra
thuc v nh s hc Victor-Amedee .1791 1875/ nhng li lit k Henri trong phn bng
mc lc. Mt vn nghim trng hn l vic b qua ph lc danh sch 27; phn Cc ti liu
tham kho b sung, tng xut hin trong phin bn u tin nhng li b b i t nhng
phin bn tip theo. Mt s trch dn qu di km theo mt s on lc v ti ngha. V d
nh vic thm vo mt danh sch di dng dc nhng nh t tng tn gio cng thi i nh
hng ln n Boole. Vic lm c nn hay khng l tt nhin mt vn v quan im v ti
cm thy rng n s khin cho cu chuyn v Boole ca chng ta b gin on.
Ngi c tp san i khi cm thy mc chi tit ca ton hc l khng y . Tht s, vi
nhng c gi ton hc c cng tng vi ca cng trnh ca Boole v mun o su hn na
v mng kin thc ny s khng tht s thy cun sch l l tng cho mc ch , v h s
phi c li nhng n phm gc ca Boole. Tuy nhin, c mt danh sch kh y v gi tr v
cc vn c nu trong cun sch.
Peter Lynch Peter Lynch l Gio s Khoa hc Kh tng ti UCD. Chuyn mn ca
ng bao gm kh ng hc, d bo thi tit s, c hc Hamilton v lch s ca kh
tng hc. ng vit mt s n phm ton hc trong phn y Ton hc trong
Thi bo Ai-len. Xem blog ca ng ti http://thatsmaths.com
Trng Khoa hc Ton hc, i hc College Dublin.
E-mail: Peter.Lynch@ucd.ie

58

NHN DNG CH MO
Bnh Nguyn
(Trng i hc Khoa hc T nhin, HQG - TP.HCM)
TM TT
Ton hc khng kh khan nh nhiu ngi tng ngh m gn lin mt thit vi
cuc sng xung quanh chng ta. t lu nay, c rt nhiu ng dng thit thc
c xy dng da trn cc c s ton hc bao gm cc php bin i v thut
ton c bn nh bin i Fourier, bin i Wavelet, thut ton SVD (singular value
decomposition), thut ton LDA (Linear Discrimination Analysis), thut ton phn
r tr ring (eigen decomposition)... Trong s , c th ni n cc ng dng lin
quan n nhn dng hnh nh (image recognition) trn in thoi thng minh (smart
phones), ni ngi ta c th quan st hnh nh trong ngi nh ca mnh thng qua
mt camera, t ng nhn c tin nhn khi c ngi l t nhp vo nh, hoc bit
c cc th cng nh c ang ln lt lm g khng. Trong bi vit ny, tc gi s
gii thiu c gi xy dng mt h thng nhn dng ch mo n gin thng qua
cc thut ton c bn trong Ton hc.

1. Gii thiu
Bi vit tp trung vo vic xy dng mt h thng thng minh c th phn bit c ch v mo.
Nu nhn bng mt thng, khng kh bt c ai c th thc hin chnh xc vic ny. Vn
t ra y l trong trng hp c mt b nh ch mo cho trc, lm sao chng ta tm ra mt
phng php gip cho mt my tnh c th nhn bit hnh nh ch mo. H thng cng thng
minh khi t c chnh xc gn ging nh con ngi.
nhn bit c nh ch mo t mt bc nh, thng thng chng ta s da vo cc ng
nt bn trong bc nh . nh ch s cha mt s nt c trng gc cnh (edge) ring bit gip
chng ta phn bit c vi nh mo. Trong phn tch thi gian - tn s, bin i Wavelet c

Hnh 7.1: Ch v mo
59

Tp ch Epsilon, S 06, 12/2015

Hnh 7.2: Mt s nh ch mo trong tp hun luyn.


bit nh l mt trong nhng phng php hu ch biu din thng tin tn hiu theo nhiu t l
khc nhau. Khng nhng th, bin i wavelet cn cc k hu ch trong vic d tm cnh (edge
detection) trong mt bc nh. Chnh v th, trong bi ton ny, chng ti s s dng bin i
wavelet biu din thng tin trong cc nh ch mo.
Nh cp trc , bi vit ny s gii thiu mt thut ton n gin gip my tnh c th
phn bit c gia ch v mo. Thut ton s bao gm cc bc sau y:
 Bc 0: cho trc M nh ch v M nh mo trong tp hun luyn (training set).
 Bc 1: Phn r (decompose) cc nh ch mo thng qua cc hm wavelet c s
(Wavelet basis functions). Mc ch ca bc ny l thc hin thao tc d tm cnh
(edge detection) trong bc nh.
 Bc 2: T cc nh m rng wavelet (wavelet expanded images) trong tp hun
luyn, dng thut ton SVD tm kim cch thnh phn chnh (the principal
components) lin quan tng ng n ch v mo.
 Bc 3: Tm kim mt ngng quyt nh phn bit gia ch v mo bng thut
ton LDA (liner discrimination analysis).
 Bc 4: Kim tra tnh hiu qu ca thut ton trn N nh ch v N nh mo. Thng
thng, ngi ta chn t l M W N D 4 W 1 .
Trong nhng phn tip theo, c gi s c tm hiu su hn tng bc trong thut ton trn.
60

Tp ch Epsilon, S 06, 12/2015

Hnh 7.3: Ch Bull.

2. Xy dng thut ton


minh ho cho thut ton, chng ta xt bc nh ch Bull trong Hnh 7.3. Trong tp hun luyn,
chng ta c tt c 98 nh ch v 98 nh mo v tt c cc nh c iu chnh kch c v kch
thc 64x64. Qu trnh tin x l ny c m t nh trong hm preprocess.m di y.
function [newImage] = preprocess(imagePath)
oldImage = imread(imagePath);
newImage = imresize(oldImage,[64,64]);
end
Sau , tt c cc nh hun luyn s c phn r thnh cc c s wavelet bng cch s dng
bin i wavelet ri rc. Trong Matlab, c gi c th s dng hm dwt2 sau y thc hin
php bin i wavelet ri rc hai chiu cho cc nh ny:
[cA, cH, cV, cD] = dwt2(X,wname)
y, cA l ma trn h s thnh phn xp x (the approximation coefficient matrix) cn cH ,
cV , v cD l ma trn h s thnh phn chi tit (the detail coefficient matrix) tnh c thng
qu vic phn tch nh u vo X. Tham s wname i din cho tn hm wavelet c s dng.
Trong v d ny, chng ta s s dng b lc Haar thc hin qu trnh phn r wavelet.

Trong hnh 7.4 v hnh 7.5, chng ta c th thy phn r Wavelet ca hnh mt con ch. Sau khi
phn r nh ch mo bng biu din wavelet, chng ta s tm phng php tip cn c th
phn bit c nh ch v mo. thc hin iu ny, mt trong nhng cch chng ta c th
tip cn l phn r tp nh ch mo bng thut ton SVD, ngha l ch mo s c biu din
bng cc thnh phn chnh (principal components). Sau , chng ta s dng phng php LDA
hun luyn mt h thng phn bit c ch mo.

Nhn chung, thut ton chng ta s tin hnh nhng bc sau y:


61

Tp ch Epsilon, S 06, 12/2015

Hnh 7.4: Bin i Wavelet trn bc nh ch Bull (nh 7.3) to thnh bn ma trn: cA, cH , cV
v cD (t tri sang phi, t trn xung di).

Hnh 7.5: Trong hnh ny, t tri qua phi, t trn xung di ln lt biu din theo chiu ngang,
chiu dc, nh d tm cnh v hnh nh tng hp li t bin i Wavelet ca nh ch Bull ban
u.

62

Tp ch Epsilon, S 06, 12/2015


dog_folder_path=./training/dog;
cat_folder_path=./training/cat;
dog_wave = dc_wavelet(dog_folder_path);
cat_wave = dc_wavelet(cat_folder_path);
feature=20;
[result,w,U,S,V,th] = dc_trainer(dog_wave,cat_wave,feature);
Trong thut ton trn, cc nh v ch v mo s c a vo h thng v sau c a vo
thut ton Wavelet c m t nh sau:
function [dcData] = dc_wavelet(dc_folder_path)
allFiles=dir(dc_folder_path);
allNames = fallFiles.nameg;
nw=32*32;
index=1;
for i = 1:length(allNames)
filename=fullfile(allNamesfig);
filename=fullfile(dc_folder_path,filename);
display(filename)
I=imread(filename);
J=imresize(I,[64,64]);
J=rgb2gray(J);
[ ,cH,cV, ] = dwt2(J,Haar);
nbcol = size(colormap(gray),1);
cod_cH1 = wcodemat(cH,nbcol);
cod_cV1 = wcodemat(cV,nbcol);
cod_edge=cod_cH1+cod_cV1;
dcData(:,index)=reshape(cod_edge,nw,1);
end end
Thut ton hun luyn c m t nh trong hm dc_trainer.m.
function [result,w,U,,V,threshold]=dc_trainer(dog0,cat0,feature)
nd=length(dog0(1,:));
nc=length(cat0(1,:));
[U,,V] = svd([dog0,cat0],0);
animals = *V;
U = U(:,1:feature);
dogs = animals(1:feature,1:nd);
cats = animals(1:feature,nd+1:nd+nc);
md = mean(dogs,2);
mc = mean(cats,2);
Sw=0;
for i=1:nd
Sw = Sw + (dogs(:,i)-md)*(dogs(:,i)-md);
end

63

Tp ch Epsilon, S 06, 12/2015


for i=1:nc
Sw = Sw + (cats(:,i)-mc)*(cats(:,i)-mc);
end
Sb = (md-mc)*(md-mc);
[V2,D] = eig(Sb,Sw);
[lambda,ind] = max(abs(diag(D)));
w = V2(:,ind); w = w/norm(w,2);
vdog = w*dogs; vcat = w*cats;
result = [vdog,vcat];
if mean(vdog)>mean(vcat)
w = -w;
vdog = -vdog;
vcat = -vcat;
end
sortdog = sort(vdog);
sortcat = sort(vcat);
t1 = length(sortdog);
t2 = 1;
while sortdog(t1)>sortcat(t2)
t1 = t1-1;
t2 = t2+1;
end
threshold = (sortdog(t1)+sortcat(t2))/2;
end
C th thy, trong bc u tin ca qu trnh hun luyn, chng ta s dng phn r SVD trn d
liu to ra t bin i Wavelet cho tng nh hun luyn. Vi tp hun luyn gm 98 nh mo v
98 nh ch, sau khi thc hin php bin i Wavelet, chng ta thu c mt vc t c tng
di 32  32 D 1024. Kt hp d liu hun luyn (98 C 98 D 196 nh), chng ta s to ra c
ma trn mi c kch thc 1024  196. n bc ny, chng ta s s dng thut ton SVD dng
rt gn trn ma trn ny c th trch ra c cc thnh phn chnh U , v V .
Tuy nhin, chng ta khng nht thit phi s dng tt c cc thnh phn chnh (principal
components) thu c t thut ton SVD lm c trng cho vic nhn dng. y, chng
ta ch cn dng mt s lng va cc thnh phn chnh, ci m c xc nh thng qua
bin feature. Gi s s lng thnh phn chnh chng ta chn l 20. Khi , t ma trn U v
V  thu c, chng ta s trch ra 20 ct u tin ca ma trn U v 20 dng u tin ca ma
trn V  . Trong hnh 7.6, c gi c th thy cc hnh nh minh ha tng ng bn thnh phn
chnh u tin ca nh ch mo trch ra t tp hun luyn.
Mt cu hi t ra ti sao chng ta li s dng ma trn V  ? Nh bit, l ma trn ng
cho m cc thnh phn ng cho ca n chnh l singular value ca ma trn 1024  196 ban
u. Chnh v th, vic chn ma trn V  s to nn trng s cho cc mode ca mt nh (mo
hoc ch) cho trc khi chiu ln V  .
Sau khi chng ta thc hin cc bc phn r Wavelet v SVD cho tng bc nh ch mo trong
tp hun luyn, chng ta s tm cch s dng nhng thng tin ny quyt nh xem nh cho
64

Tp ch Epsilon, S 06, 12/2015

Hnh 7.6: Minh ho cho bn thnh phn chnh u tin (principal components) ca tp nh ch
mo. Nhng thnh phn th ny s c s dng lm c s phn loi mt nh ch hoc mo.
trc l nh con vt no. Thut ton LDA 2-lp c s dng trong bi ton ny. Mt trong
nhng mc tiu ca thut ton LDA chnh l tm ra php chiu hiu qu nht sao cho d liu khi
chiu xung mt khng gian con cho trc c th phn chia tt.
Nh trong hnh 7.7, vi d liu hai lp cho trc, php chiu LDA to ra mt phn b l tng
cho d liu khi gi trung bnh ca d liu hai lp trn h trc mi 1 v 2 hon ton tch ri
nhau. Nu xt v kha cnh ton hc, php chiu trn tho mn:
w D arg max
w

w T SB w
;
w T SW w

(2.1)

trong , cc ma trn phn tn gia cc lp SB v trong mt lp SW c tnh nh sau:


SB D .2 1 /.2 1 /T
2 X
X
.x i /.x i /T
SW D
i D1

(2.2)
(2.3)

D thy, nghim ca bi ton 2.4 chnh l vc t ring tng ng vi tr ring ln nht  ca bi


ton tr ring:
SB w D Sw w:
(2.4)
gii c bi ton tr ring trn, chng ta c th dng hm eig trong Matlab tm gi tr
w tng ng. Nh vy vi cc nh ch mo nm trong tp hun luyn ban u, chng ta s
chiu xung c s LDA thu c phn b tng ng. Khi , chng ta cn tm mt ngng
(threshold) gip phn loi mt bc nh nm trong nhm con vy no. Tuy nhin, thut ton
vn khng trnh khi mt s trng hp nh mo nhn nhm thnh nh ch v ngc li. Cng
nhiu c trng s dng, chng ta hi vng cng gim s li trong qu trnh nhn dng.
65

Tp ch Epsilon, S 06, 12/2015

Hnh 7.7: Php chiu LDA to ra mt phn b l tng v d liu thu c trn h trc mi hon
ton c th phn chia thnh hai lp khc nhau.
kim tra cht lng t c ca h thng va xy dng, chng ta tin hnh kim tra vi 16
nh mo v 16 nh ch (hon ton c lp vi tp nh hun luyn ban u).
%Test on the testing dataset:
testing_set=./testing
% wavelet transformation
testing_wavelet = dc_wavelet(testing_set);
% SVD projection
TestMat = U*testing_wavelet;
% LDA projection
pval = w*TestMat;
% 16 cats and 16 dogs
hiddenlabels=[1,1,1,1,1,1,1,1,1,1,1,1,1,1,1,1
,0,0,0,0,0,0,0,0,0,0,0,0,0,0,0,0];
TestNum = length(pval);
%cat = 1, dog = 0
ResVec = (pval>threshold)
disp(Number of mistakes);
errNum = sum(abs(ResVec - hiddenlabels))
disp(Rate of success);
sucRate = 1-errNum/TestNum
Vi b d liu chng ti xt, t l nhn dng ng t c khong 72%. Chng ti xin nhng
li phn ci tin cho c gi ca Epsilon trong s bo ny. on codes tham kho c th tm thy
ti: Pet-recognition.

Acknowledgement
Tc gi chn thnh cm n TS. L Phong v TS. ng Nguyn c Tin sa li v gp cho
bn tho.

66

BI TON CN TIN
ng Nguyn c Tin
(i hc Trento, Italia)
LI GII THIU
Tip ni nhng s bo trc, chuyn mc Ton hc Gii tr k ny gii thiu vi
c gi mt bi ton logic kinh in: Bi ton cn tin (counterfeit coin problem).
Dng thc chung ca cc bi ton cn tin l c mt hoc mt s ng tin gi
c t ln ln vi cc ng tin tht. Cc ng tin gi c b ngoi ging ht tin
tht nhng khc khi lng so vi tin tht. Cn s dng cn a hoc cn s vi
mt s ln cn xc nh tm ra (mt hoc mt s) ng tin gi ny. y chng
ti cng nhc li hai khi nim v hai loi cn c s dng. Cn a gip chng ta
so snh khi lng ca cc vt c t 2 bn ca a cn v kt qu tr v hoc
nng hn, hoc nh hn, hoc cn thng bng. Cn s cho php xc nh chnh xc
khi lng ca vt c t trn cn. Trong cc bi ton c trnh by s ny,
chng ti lun gi s rng cc cn c s dng lun c chnh xc tuyt i.
Tng t nh bi ton i nn, bi ton cn tin u tin theo chng ti ra i
t rt sm, nhng mi n gia th k 20, nhng li gii u tin mi chnh thc
c ghi nhn v cng b. V cng tng t nh bi ton anh em ca mnh, bi ton
cn tin c rt nhiu bin th khc nhau, m hin ti chng ti su tp c hn 53
phin bn ca bi ton ny.
Trong chuyn mc Ton hc Gii tr k ny, chng ti chn lc gii thiu vi
bn c 5 bi ton cn tin s dng cn a. Trong s tip theo, chng ti s gii
thiu nhng bi ton s dng cn s.
V by gi, xin mi bn c hy sang trang tip theo n vi bi ton cn tin
u tin ...

67

Tp ch Epsilon, S 06, 12/2015

1. Bi ton cn tin vi 9 ng xu
Bi ton cn tin u tin c pht biu nh sau:
C 9 ng xu ging ht nhau, trong c mt ng gi nng hn 8 ng cn li. S dng
cn a v khng s dng thm qu cn, bng 2 ln cn, hy ch ra ng gi.
Bi ton ny kh n gin v xut hin trong rt nhiu tp ch cng nh sch v ton hc vui,
gii tr. . . Bn c hn s nhanh chng tm ra phng n cho bi ton vi cch cn nh sau:
Ln 1: t mi a cn 3 ng. Ta c 2 trng hp: cn thng bng hoc cn lch. Nu cn
thng bng, ta suy ra ng gi thuc v 3 ng cha cn. Nu cn lch, ta suy ra ng gi thuc
v nhm 3 ng nng hn. Nh vy, ta lun xc nh c nhm 3 ng no c cha ng gi.
Ln 2: Chn 2 ng trong 3 ng va xc nh v t mi a cn 1 ng. Ta cng c 2 trng
hp y: Nu cn thng bng, suy ra ng cha cn l ng gi. Nu cn lch, th ng nng
hn l ng gi.
Nh vy, vi 2 ln cn, ta xc nh c ng gi (bit trc nng hn) trong s 9 ng.

Ta c th nng cp bi ton ln 27 ng vi 3 ln cn, trong ln u cn mi bn 9 ng v p


dng phng php tng t khi bit ng gi 9 ng no. Tng qut ha bi ton vi n ln
cn, ta c th xc nh c ng gi (bit trc nng hn) trong s 3n ng!
Bi ton trn kh n gin, nhng khi thay i mt t iu kin, ta c c nhng bi ton hc
ba v v cng th v, m tiu biu l bi ton tip theo y.

2. Bi ton 12 ng xu
Theo chng ti, y l bi ton kinh in v ni ting nht trong s cc bi ton cn tin. Phin
bn u tin m chng ti tm c xut pht t Dyson v Lyness vo nm 1946 [1].
Pht biu bi ton nh sau:
68

Tp ch Epsilon, S 06, 12/2015

C 12 ng xu ging ht nhau, trong c mt ng gi c khi lng khc vi 11 ng


cn li, nhng khng bit nng hn hay nh hn. S dng cn a v khng s dng thm
qu cn, bng 3 ln cn, hy ch ra ng gi.
Thot nhn, bi ton ny kh ging vi bi ton 9 ng xu u tin. Tuy nhin, vi yu t khc
bit l chng ta khng bit c ng gi nh hn hay nng hn so vi ng tht. V vi yu t
ny, bi ton tr nn kh khn hn kh nhiu.
Trc khi bc sang phn li gii, chng ti hi vng c gi hy dnh ra t thi gian th sc
vi bi ton ny thy c s th v ca n. Mt trong nhng kho st chi tit u tin ca
bi ton ny c gi c th xem [2] bi Richard Bellman, mt trong nhng nh ton hc tin
phong v Quy hoch ng.
V by gi, chng ti gii thiu 3 cch gii khc nhau cho bi ton ny cng nh m rng cho
bi ton.

2.1. Cch gii logic


Chng ti s dng bi ton ny cc din n Ton hc, cc chuyn mc vui cng nh vi
bn b trong hn 15 nm qua v y l cch gii m chng ti hay nhn c nht. V bn cht
cch gii ny l chui suy lun logic nn chng ti tm gi phng php ny l cch gii logic.
Cch lm ny nh sau: Chia 12 ng thnh 3 nhm v t tn cho cc ng tng ng vi cc
nhm l aaaa, bbbb v cccc.
Ln 1: Cn aaaa v bbbb
Trng hp 1. Nu cn thng bng, ta suy ra 8 ng aaaa v bbbb u tht.
Ln 2: t 3 ng bt trong 8 ng ny v 3 ng trong 4 ng cn li, v d aaa v ccc.
y s c 2 trng hp:
Trng hp 1.1. Nu cn thng bng, suy ra ng c cn li gi. Ta cn ln 3 s bit nng nh.

Trng hp 1.2. Nu cn lch, ta bit ng gi trong 3 ng ccc, v ta bit c nng hay


nh. Ln cui n gin cn 2 ng s 3 ng ccc ny vi nhau. Nu cn bng, c cn li gi (
bit nng nh), nu khng cn bng, cng xc nh c.
Trng hp 2. Nu cn lch, ta suy ra 4 ng cccc u tht. Khng mt tnh tng qut, gi s
aaaa < bbbb.
Ln 2: cn accc v baaa (mu cht y l tch nhm a v b ra sao cho thnh 1 b 1 ng
v 1 b 3 ng). C 3 trng hp nh sau:
Trng hp 2.1. Nu cn bng, suy ra 3 ng bbb cn li l gi nng, v vi ln cui ta s
xc nh c.
69

Tp ch Epsilon, S 06, 12/2015


Trng hp 2.2. Nu accc < baaa th hoc ng a ny gi nh, hoc ng b gi nng. Ln
cui cn c v a hoc b ta s xc nh c.
Trng hp 2.3. Nu accc > baaa th ng gi trong 3 ng aaa, v l gi nh. Vi ln
cui ta cng xc nh c.
V nh vy, bi ton c gii quyt trn vn vi 3 ln cn.

2.2. Cch gii gn nhn


Cch gii ny theo ghi nhn ca chng ti c xut bi Brian D. Bundy vo nm 1996 [4].
Trc khi i vo phn tch cch gii ny, chng ti nu mt p n v d nh sau: gn s t 1 n
12 cho 12 ng v thc hin 3 ln cn:
Ln 1: Cn (1, 2, 7, 10) v (3, 4, 6, 9)
Ln 2: Cn (1, 3, 8, 11) v (2, 5, 6, 7)
Ln 3: Cn (2, 3, 9, 12) v (1, 4, 5, 8)
Khi vi kt qu 3 ln cn, cch lm ny s xc nh c ng no l gi v gi nng hay nh!
V d nu nh ta c 3 kt qu l ln 1 bng nhau, ln 2: bn phi nng hn, ln 3: bn tri nng
hn, ta s bit ng gi l ng 8 v ng ny nh hn.
Lm sao c c iu ny? Ta thy rng mi ln cn s c 3 kh nng, cn bng (k hiu 0),
bn tri nng hn (k hiu T ) hoc bn phi nng hn (k hiu P ). V ng vi 12 ng th ta
c 24 kh nng l mi ng l gi v gi nng hay gi nh. Do vy, cch lm l tm cch chia
cc qu cn sao cho mi b kt qu ca 3 ln cn s ng vi mt p s duy nht, v d nh kt
qu T TP ta s c tng ng p s ng 1 gi nng, kt qu TP T s c p s ng 2 gi
nng ... Cch lm ny khng nhng ch ra ng gi, v gi nng hay nh m cn ch ra cch xp
cc qu cn nh th no cho ph hp.
By gi ta cng phn tch cch lm. Xt 2 nhn xt sau:
Nhn xt 1: Ta c 27 kh nng khc nhau ca cc ln cn (33 D 27) v 24 kt qu phn bit. Ta
loi b trng hp cng t mt ng cng 1 bn cn trong c 3 ln cn (do t nh vy th s
khng cn ). Khi 3 trng hp 000, T T T v PPP b loi b v ta cn li 24 kh nng
ng vi 24 p s!
Nhn xt 2: Ta thy nu kt qu 0P T ng vi p n ng 8 gi nh th kt qu i xng l
0TP , phi cho ra ng 8 gi nng do 0TP cho thy ng 8 (gi) xut hin 2 ln cn sau. V
vy ta c th phn 24 kh nng thnh 2 nhm, mi nhm ng vi 12 ng xu.
Hai nhn xt trn chnh l c s tm ra cch gn: Ta ghi ra 12 phng n phn bit v khng
cha phng n i xng thnh 12 ct, 3 dng, mi dng c ng 4 0, 4 T v 4 P v khng c
ct no c 000, T T T , hoc PPP . Sau gn th t t 1 n 12 ng vi 12 ng.
70

Tp ch Epsilon, S 06, 12/2015


T
0
T
1

T
0
P
2

T T P
0 T T
0 P 0
3 4 5

P P
T T
T P
6 7

P
P
0
8

0 0
P P
T P
9 10

0
P
0
11

0
0
T
12

V d nh sau ( tm ra cch gn tha yu cu khng kh nhng cng cn phi c chin thut,


y chng ti xin php lc bt v cch gn sau y cng chnh l gi ):
chn cc ng xu trong cc ln cn, ta t theo tng dng, nu l T , ta t ng xu bn tri
v P , ta t bn phi. Khi nu phng n ra ng vi th t, s l gi nng v ngc li ta xt
i xng v c gi nh. Vi phng n nh v d trn, ta c:
Ln 1: (1, 2, 3, 4) v (5, 6, 7, 8)
Ln 2: (4, 5, 6, 7) v (8, 9, 10, 11)
Ln 3: (1, 6, 9, 12) v (2, 4, 7, 10)
Ta thy y l mt cch khc sp xp cc ng xu khc vi cch nu ra u tin. Th xt vi v
d nh sau:
- Nu nh kt qu l T TP , tra vo bng lp ta thy s ng vi 4, v v vy y l 4 gi
nng.
- Nu kt qu l P 0T , tra vo bng ta thy khng c, vy l gi nh, v ly i xng ta c
T 0P , ng vi ng 2, vy y l 2 gi nh.
V vi cch lm ny, ta c th tng qut ln vi n ln hn 3. Cc m rng khc ca bi ton cng
c th lm da trn c s ny. Tuy nhin, nhc im l vi n ln, trnh sai st trong gn nhn
thng ta cn n s tr gip ca my tnh.

2.3. Cch gii Jack Wert


y l cch gii yu thch nht ca ngi vit bi ny, theo ghi nhn ca trang Cut The Knot th
li gii c a ra bi Jack Wert [5], do vy chng ti dng tn tc gi gi cho phng php
ny.
V mt tng, cch lm ny kh ging nh cch u tin, nhng c tc gi pht biu hnh
thc d hiu hn v c th tng qut ha ln vi trng hp n > 3 kh d dng.
Cch lm ca Jack Wert cho bi ton ny nh sau:
Chia 12 ng thnh 3 nhm, mi nhm 4 ng v trong mi nhm chia thnh mt "ng" 3 ng
v mt "ng" 1 ng l (v d nh gi 3 ng ny vo mt bao giy v gi s khi lng bao
bng 0).
Ln 1: t mi a cn 2 nhm u tin v ghi nhn tnh trng cn.
71

Tp ch Epsilon, S 06, 12/2015


Ln 2: Hon i cc "ng" ln: "ng" cn bn tri qua cn bn phi, "ng" cn bn phi
t ra ngoi v "ng" cha cn t cn bn tri. Ghi nhn tnh trng cn.
Nu tnh trng cn thay i: ta xc nh c ng no c cha ng gi, v gi nng hay gi
nh. Vi 1 ln cn cn li, ta s bit c ng gi ny. Nu tnh trng cn khng thay i: ta
bit ng gi s vo mt trong cc ng l v vi 1 ln cn, ta cng xc nh c ng gi
ny v bit c gi nng hay gi nh. Mt cch l gii tuyt p m khng cn phi gn nhn!

By gi chng ta hy th xem lm cch no m rng cch gii ny cho n ln hn, v d 1092


ng vi 7 ln cn: ta cng chia thnh 3 nhm, mi nhm gm 364 ng. Sau , trong mi
nhm, ta gi cc ng thnh tng ng 243, 81, 27, 9, 3, 1 ng (6 ng).
Ln 1: t ton b 6 ng ca mi 2 nhm ln cn. Ghi nhn tnh trng cn.
Ln 2: Hon i ng ln nht (cha 243 ng) t cn bn tri sang cn bn phi, t cn bn
phi ra ngoi v t bn ngoi vo cn bn tri. Ghi nhn tnh trng cn.
Nu tnh trng cn thay i, ta bit ng no cha ng gi v nng hay nh tng ng. V ta tm
c ng gi trong 243 ng ny sau 5 ln cn (35 D 243, do bit gi nng hay gi nh).
Nu cn khng thay i, hon chuyn n ng th 2 (ng cha 81 ng) v ghi nhn tnh trng
cn. Nu cn thay i, ta bit ng gi ng no cng vi tnh nng/nh tng ng, v tm ra
c ng ny sau 4 ln cn. Nu cn li khng thay i tnh trng, ta li tip tc hon chuyn
ng tip theo (27 ng) v c nh vy cho n ng cui cng, v ta lun s dng ti a 7 ln
cn!
Bng cch lm ny, d thy li gii tng qut vi n ln cn, ta c th xc nh c ng gi v
l gi nng hay gi nh trong .3n 3/=2 ng!
72

Tp ch Epsilon, S 06, 12/2015

3. Nng hn hay nh hn?


Bi ton cn tin th 3 ny chng ti trch ngun quen thuc t blog ca Tanya Khovanova, l
mt bin th kh th v (nhng d hn) ca bi ton trn.
C N > 2 ng xu ging ht nhau, trong c mt ng gi c khi lng khc vi cc
ng cn li, nhng khng bit nng hn hay nh hn. S dng cn a v khng s dng
thm qu cn, hy xc nh s ln cn t nht xc nh ng gi l nng hn hay nh hn
so vi cc ng khc m khng yu cu phi xc nh c th ng gi ny.
Sau y l li gii ca bi ton c ng bi tc gi (Tanya Khovanova): Ch cn 2 ln cn l ta
c th xc nh c ng gi l nng hn hay nh hn so vi cc ng cn li.
Trng hp N D 3, n gin ta ch so snh tng cp hai ng xu vi nhau.
Xt trng hp N D 4. Ta cn ln u mi bn 2 ng. Cn khng th cn bng. Khng mt
tnh tng qut, gi s cn bn tri nh hn. Ln 2 so snh 2 ng bt k cn bn tri, nu cn
bng, ta bit ng gi nhm cn li v bit c l gi nng. Nu cn lch, ta bit c ng
nh hn l gi nh.
Nh vy, ta c th tng qut ln trng hp N D 3k v N D 4k vi cch lm tng t.
Vi N bt k, ta c th tch N ra lm 3 nhm vi s lng a, a, v b sao cho a  b  2a. Ln
u ta cn 2 nhm a, nu cn thng bng, ta c 2a ng u tht, v cn ln 2 vi b ng mi
bn. Nu cn lch, ta bit b ng cn li l tht v ta s cn tip a ng t b vi mt trong 2
nhm a cn.
Cch lm ny, cn thiu cho trng hp N D 5, v vi N D 5 ta khng th tch ra thnh 3 nhm
nh yu cu ca cch gii trn. Vi N D 5, ln u ta cn mi bn 2 ng. Nu cn thng bng,
ta cn ln cui vi mt ng bt k trong 4 ng ny v ng cn li. Nu cn lch, ta lm nh
ln cn th 2 cho trng hp N D 4. V nh vy bi ton c gii quyt.

4. Bi ton 10 ng xu
bi ton ny, chng ti gii thiu mt bi ton kh kh, v c gii quyt t cch y gn 20
nm (1997) bi mt nh ton hc Vit Nam quen thuc vi Epsilon cc s trc: Gio s V
H Vn (bn c c th xem chi tit hn [3]).
Chng ti pht biu bi ton ny dng n gin vi 10 ng xu nh sau:
Cho 10 ng xu, trong c mt s ng gi. Bit rng cc ng tht nng bng nhau. Cc
ng gi cng i mt nng bng nhau nhng c khi lng khc vi ng tht. Vi mt
cn a v khng s dng thm qu cn, bng 3 ln cn hy cho bit 10 ng ny liu c
phi cng loi?
73

Tp ch Epsilon, S 06, 12/2015


Pht biu tng qut hn ca bi ton ny nh sau:
Cho mt tp gm m ng xu vi ti a 2 loi khi lng khc nhau. Hy xc nh liu tt c cc
ng ny c cng khi lng bng t ln cn nht.
Bn c c th tham kho li gii chi tit hn [3], trong mc ny chng ti s dng li gii s
cp v c th trong trng hp N D 10 ny nh sau:

Trc tin, nh s cc ng xu t 1 n 10. D thy, nu nh trong mt ln cn bt k, cn


chnh lch th ta gii quyt c bi ton. Do vy, gi s rng cn lun cn bng trong c 3 ln
cn.
Xt 2 nhm: nhm 1 ch gm ng 1, nhm 2 gm 2 ng 2 v 3. Gi a v b ln lt l s ng
gi 2 nhm ny. Nh vy ta c a D 0 hoc a D 1 v b D 0, hoc b D 1, hoc b D 2.
Ln 1: cn 3 ng (1, 2, 3) v 3 ng (4, 5, 6). Do cn cn bng nn ta bit s ng gi 3 ng
(4, 5, 6) l a + b.
Ln 2: cn 4 ng cn li (7, 8, 9, 10) v 4 ng (1, 4, 5, 6). Do cn cn bng nn ta bit s ng
gi 4 ng (7, 8, 9, 10) ny l 2a C b.
Ln 3: cn (1, 7, 8, 9, 10) v (2, 3, 4, 5, 6). Do cn cn bng nn ta c 3a C b D a C 2b, suy ra
2a D b. iu ny ngha l hoc a D b D 0 (ngha l tt c cc ng u tht) hoc a D 1 v
b D 2 (ngha l tt c cc ng u gi). Trong c 2 tnh hung, ta u ch ra c tt c cc
ng ny c cng khi lng vi nhau hay khng!

5. Bi ton 9 ng xu
Chng ti bt u v kt thc cho chuyn mc ny u vi cc bi ton 9 ng xu, 2 ln cn, v
bi ton cui cng ny nh sau:

74

Tp ch Epsilon, S 06, 12/2015


C 9 ng xu, trn mi ng c ghi cc con s t 1 n 9 cho bit khi lng tng ng ca
chng l 1gr, 2gr, . . . 9gr. Bit rng c mt ng trong s ny b li v c khi lng nh
hn khi lng c quy nh. Vi mt chic cn a v khng s dng thm qu cn, liu
rng vi 2 ln cn, c th ch ra ng b li ny?

Li gii bi ton ny kh n gin v chng ti xin dnh cho c gi. Chng ti cng kt thc
chuyn mc ti y v hn gp li trong s ti, vi chuyn bi ton cn tin vi cn s.

Ti liu tham kho


[1] F. J. Dyson and R. C. Lyness, Math. Gazette, vol. 30, 1946.
[2] R. Bellman and B. Glass, On various versions of the defective coin problem, Information
and Control, vol. 4, pp. 118131, 1961.
[3] D. N. Kozlov, V. H. Vu, Coins and Cones," Journal of Combinatorial Theory, Series A,
vol. 78, no. 1, pp 114, 1997.
[4] B. D. Bundy, The Oddball Problem, Mathematical Spectrum, vol. 29, no. 1, pp 1415,
1996/7.
[5] A. Bogomolny, Odd Coin Problems: the 120 marble problem - five weighings from Interactive Mathematics Miscellany and Puzzles, www.cut-theknot.org//blue//OddCoinProblems.shtml, Accessed 09 October 2015.

75

Tp ch Epsilon, S 06, 12/2015

76

XUNG QUANH BI TON HNH HC TRONG


K THI VMO 2014
Nguyn Tin Dng - H Ni

Tm tt
Bi vit a ra mt gc nhn ca tc gi v bi hnh hc s 4 trong k VMO 2014
cng nh nhng khai thc xung quanh cu hnh ca bi ton.

Bi hnh hc s 4 trong k VMO 2014 c ni dung c cp trong [1] nh sau:


Bi ton 1. Cho tam gic nhn ABC ni tip ng trn .O/ vi AB < AC . Gi I l trung
im cung BC khng cha A. Trn AC ly im K khc C sao cho IK D IC . ng thng
BK ct .O/ ti D.D B/ v ct ng thng AI ti E. ng thng DI ct ng thng
AC ti F .
BC
1. Chng minh rng EF D
.
2
2. Trn DI ly im M sao cho CM song song vi AD. ng thng KM ct ng thng
BC ti N . ng trn ngoi tip tam gic BKN ct .O/ ti P .P B/. Chng minh rng
ng thng PK i qua trung im ca on thng AD.
C th thy rng hai ca bi ton hu nh khng lin quan ti nhau, 2. mi l chnh ca bi
ton. V vy, bi c pht biu gn li nh sau
Bi ton 2. Cho tam gic nhn ABC ni tip ng trn .O/ vi AB < AC . Gi I l trung
im cung BC khng cha A. Trn AC ly im K khc C sao cho IK D IC . ng thng
BK ct .O/ ti D.D B/. Trn DI ly im M sao cho CM song song vi AD. ng
thng KM ct ng thng BC ti N . ng trn ngoi tip tam gic BKN ct .O/ ti P
.P B/. Chng minh rng ng thng PK i qua trung im ca on thng AD.
Li gii th nht. Do I l trung im cung BC khng cha A ca ng trn .O/ nn IB D
IC D IK. Ta thy AKI D 180 CKI D 180 ICK D ABI nn AIB D
AIK.g:c:g/. V th K i xng vi B qua AI . Ta c DCK D ABK D AKB D
DKC nn d thy DI l trung trc ca on CK. Ch n tnh i xng qua trc DI v
CM k AD ta c MKC D M CK D DAC D KBN nn AC l tip tuyn ca ng
trn .BKN /. Gi E l giao ca DI v AC , th th EKP D KBP D EIP nn t gic
EP IK ni tip, suy ra IPK D IEK D 90 . PK ct .O/ ti F .F P /. Ch rng K l
trc tm ca tam gic ADI nn ta d dng chng minh c AFDK l hnh bnh hnh. Do
KF i qua trung im AD. T ta c pcm.
77

Tp ch Epsilon, S 06, 12/2015

F
A

D
K

E
M
B

P
I

Li gii th hai. Trong li gii th nht, ta chng minh c AC l tip tuyn ca ng


trn .BKN /. T , ch n tnh i xng qua AI , ta cng c AB l tip tuyn ca ng
trn .BKN /. Gi L l giao ca AP v ng trn .BKN /.L P /. VKLP D KBP D
DAP nn KL k AD. PK ct AD ti S . Bng bin i gc n gin, ta thu c cc cp
tam gic ng dng SAK v KPL, SDK v BPL. Ch rng t gic BPKL iu ha ta c
KP
BP
SD
SA
D
D
D
nn SA D SD. Suy ra pcm.
SK
KL
BL
SK

L
K
O
M
B

N
P
I

78

Tp ch Epsilon, S 06, 12/2015

Nhn xt.
1. Trong li gii th nht, ta c KPN D KBN D DAC D KIE D KPE nn
P; N; E thng hng.
2. Vic pht biu li lm cho ton hay v c ngha hn. Trn y, tc gi trnh by hai li
gii thun ty hnh hc cho bi ton. Li gii th nht l ca tc gi. Li gii th hai da trn
tng s dng cc kin thc v t gic iu ha v chm iu ha ca thnh vin din n Ton
hc Mathscope.org c nickname vinhhai (trong [3]), c tc gi chnh l li c c li gii
thun ty hnh hc. Ch rng K l trc tm ca tam gic IAD, ta c th pht biu li bi ton
2 nh sau:

Bi ton 3. Cho tam gic ABC ni tip ng trn .O/. Cc ng cao h t B v C theo th
t ct .O/ ti D v E. K EF k BC.F 2 AB/. HF ct DE ti K. ng trn .HKD/ ct
.O/ ti P .P D/. Chng minh rng PH chia i BC .

A
E

F
P

K
O

D
H

T nhn xt 1. trong bi ton 2 v bi ton 3, ta c th xut cc bi ton sau:

Bi ton 4. Cho tam gic ABC ni tip ng trn .O/, c trc tm H . Cc ng cao BD v
CE theo th t ct .O/ ti F v G. K FM k GN k BC.M 2 AC; N 2 AB/. HM v HN
theo th t ct F G ti K v L. Chng minh rng DK v EL ct nhau ti mt im thuc .O/.
79

Tp ch Epsilon, S 06, 12/2015

A
M
P

K
L

D
N

E
H
B

Li gii. D thy t gic ADHE ni tip ng trn ng knh AH . Gi P l giao ca ng


trn ng knh AH vi .O/.P A/. Ch rng AB l trung trc ca GH v GN k BC
ta c HLF D LGH C LHG D FBC C NGH D HBC C H CB D BAC
D 90 ABF D APH APF D HPF nn t gic HLPF ni tip. T , ta c
LPH D LFH D EAH D EPH , v th P; E; L thng hng. Chng minh tng t
P; D; K thng hng. Do DK; EL ct nhau ti im P thuc .O/.
Bi ton 5. Cho tam gic ABC c cc ng cao BD; CE ct nhau ti H . Gi F; G theo th
t l im i xng vi H qua AC; AB. I l trung im F G. K; L theo th t giao im khc
I ca ng trn ngoi tip cc tam gic HID; HIE vi F G. DK; EL ct nhau ti P . Chng
minh rng PH chia i BC .

A
F
K

I
D

L
G
E

Nu nhn nhn bi ton 5 da trn cu hnh ca cc bi ton 3 v 4 th vn tr nn rt n


gin. Nhng c c mt li gii p v trc tip cho bi ton th khng phi l iu d dng.
Tip tc khai thc, ta c bi ton sau:
80

Tp ch Epsilon, S 06, 12/2015


Bi ton 6. Cho tam gic ABC ni tip ng trn .O/, trc tm H , cc ng cao BK; CL.
AH; CH theo th t ct .O/ ti D v F . Ly im M sao cho hai tam gic MFH v HBC
ng dng (M; A nm cng pha i vi KL/. ML ct .O/ ti N . K DP k KL.P 2 .O//.
Chng minh rng NP chia i KL.

A
N
E
M

R
O

H
B

Q
P

Li gii. D thy HM ? AF; FM k KL. BH ct .O/ ti E th KL l ng trung bnh ca


tam gic HEF nn M thuc EF . T , theo bi ton 4 d dng suy ra N thuc ng trn
ng knh AH . NH ct BC ti Q th Q l trung im ca BC , QK v QL l tip tuyn ca
ng trn ng knh AH . Theo tnh cht quen thuc th AO ? KL nn AO ? DP , suy ra
AD D AP . Gi R l trung im KL. Ch rng NH l ng i trung ca tam gic NKL
ta c ANR D ANK C KNR D ALK C LNH D ACB C BAD D ACD D
ANP nn N; R; P thng hng. Vy NP chia i KL.
T cu hnh bi ton 4 v bi ton trong [4], ta thu c bi ton hay sau:
Bi ton 7. Cho tam gic ABC ni tip ng trn .O/. Cc ng cao h t B; C ca tam
gic ABC theo th t ct ng trn .O/ ti D v E. DE theo th t ct AC v AB ti M v
N . K DP k EQ k BC.P 2 AC; Q 2 AB/. Chng minh rng:
1. ng thng ni trung im MQ v NP i qua tm ng trn Euler ca tam gic ABC .
2. Trc tm cc tam gic ABC; AMQ; ANP thng hng.
Li gii. gii bi ton ny ta cn s dng n b ERIQ (Equal Ratios In Quadrilateral)
c pht biu nh sau:
A1 A2
B ERIQ. Cho hai b ba im thng hng A1 ; A2 ; A3 v B1 ; B2 ; B3 sao cho
D
A1 A3
B1 B2
D k. Nu C1 ; C2 ; C3 theo th t chia A1 B1 ; A2 B2 ; A3 B3 theo cng mt t s th
B1 B3
C1 C2
C1 ; C2 ; C3 thng hng v
D k.
C1 C3
81

Tp ch Epsilon, S 06, 12/2015


B ERIQ l mt kt qu tr nn quen thuc vi nhiu bn c, c th tm thy mt s cch
chng minh trong cc ti liu khc nhau (chng hn trong [2]), xin khng trnh by y.
Tr li bi ton,
1. Chng minh ng thng ni trung im MQ v NP i qua tm ng trn Euler ca tam
gic ABC .

A
P
M

X2
N
Q

X1
X'
O

L
H
B

Cc ng cao BK; CL ca tam gic ABC ct nhau ti H . D thy KL k DE nn ta c


CMD D AMN D ABC D BQE, t CDM  BEQ. Chng minh tng t ta
BQ
CM
LQ
KM
c CDP  BEN . Do
D
D k v
D
. Gi X; X 0 ; X1 ; X2 theo th
BN
CP
LN
KP
t l trung im BC; KL; MQ; NP . Theo b ERIQ, ta c X 0 ; X1 ; X2 thng hng; X; X1 ; X2
XX1
thng hng v
D k. T vi ch rng XX 0 i qua tm ng trn Euler ca tam gic
XX2
ABC ta c pcm.
2. Chng minh trc tm cc tam gic ABC; AMQ; ANP thng hng.

Y2
G2
X2
N
Q
B

Z2
Z1
Z
N

P
G1
G

P
O2

Y1
Y

O1
O

X1
X

82

Tp ch Epsilon, S 06, 12/2015


Gi G; G1 ; G2 theo th t l trng tm cc tam gic ABC; AMQ; ANP . Ta c
AG2
AX2

AG
AX

AG1
AX1

2
GG1
D k.
nn theo nh l Thales G; G1 ; G2 thng hng v
3
GG2

Gi Y; Y1 ; Y2 theo th t l hnh chiu ca O; O1 ; O2 trn AC ; Z; Z1 ; Z2 theo th t l hnh


1
1
chiu ca O; O1 ; O2 trn AB. Ta c Y Y1 D AY1 AY D .AM AC / D CM v
2
2
1
1
YY1
CM
Y Y2 D AY2 AY D .AP AC / D CP nn
D
D k. Chng minh tng t ta
2
2
Y Y2
CP
ZZ1
c
D k. Ch rng OY k O1 Y1 k O2 Y2 v OZ k O1 Z1 k O2 Z2 , theo nh l Thales,
ZZ2
OO1
ta c O; O1 ; O2 thng hng v
D k. Gi H1 ; H2 theo th t l trc tm cc tam gic
OO2
HG
H1 G1
H2 G2
GG1
OO1
2
AMQ; ANP th
D
D
D . Ch rng
D
D k, theo b
3
HO
H1 O1
H2 O2
GG2
OO2
ERIQ th H; H1 ; H2 thng hng (pcm).

Nhn xt. Bi ton c gii da trn nhng tng ca Nguyn Vn Linh (trong [2]) v Kostas
Vittas (trong [4]). Ta cng chng minh c trc tm ca bn tam gic ABC; AMQ; ANP; AKL
thng hng. Mt cch tng qut, ta c kt qu sau: Cc im chia cc on thng ni trng tm
v tm ng trn ngoi tip cc tam gic ABC; AMQ; ANP; AKL theo cng mt t s th
thng hng.
T bi ton 7, ta c th xut bi ton sau:

Bi ton 8. Cho tam gic ABC ni tip ng trn .O/. Cc ng cao h t B; C ca tam
gic ABC theo th t ct ng trn .O/ ti D v E. DE theo th t ct AC v AB ti M
v N . K DP k EQ k BC.P 2 AC; Q 2 AB/. Chng minh rng ng thng ni trung im
cc on MQ; NP v ng thng ni tm ng trn Euler ca cc tam gic AMQ; ANP
ct nhau ti mt im thuc ng thng Euler ca tam gic ABC .

Gin lc cc yu t trong bi ton 7, tc gi tm ra bi ton hnh hc th v sau:

Bi ton 9. Cho tam gic ABC ni tip ng trn .O/ c trc tm H . BH; CH theo th t
ct .O/ ti D v E. DE ct AC ti F . K EG k BC.G 2 AB/. Chng minh rng tip tuyn
ti F; G ca ng trn ngoi tip tam gic AF G ct nhau ti mt im thuc CE.
83

Tp ch Epsilon, S 06, 12/2015

P
F
N
E

K
G

H
T
B

Li gii. Gi BK; CL l cc ng cao ca tam gic ABC . ng trn .AKL/ ct .O/ ti


P khc A. Tng t bi ton 8, ta chng minh c hai tam gic BEG v CDF ng dng
LB
KC
c EL; DK l cc ng cao tng ng nn
. Ch rng hai tam gic PBL v
D
LG
KF
P CK ng dng, ta c hai tam gic PLG v PKF ng dng suy ra hai tam gic PLK v
P GF ng dng. T t gic AP GF ni tip. Tip tuyn ti F; G ca ng trn .AF G/ ct
nhau ti T . Tip tuyn ti K; L ca ng trn .AKL/ ct nhau ti trung im M ca BC . D
thy P; H; M thng hng. PL ct DE ti N . Ta c NPH D LAH D ECB D NDH
nn t gic NHDP ni tip. T cc cp tam gic ng dng PLG v PKF , LGH v KFD
d thy hai tam gic P GH v PFD ng dng. Ch rng t gic NHDP ni tip ta c
PHG D PDF D PHN nn N; G; H thng hng. P T ct ng trn .AF G/ ti Q khc
P . Do hai tam gic PLK v P GF ng dng nn ta c cc cp tam gic ng dng PLM v
P GT , PLH v P GQ. T GAQ D GPQ D LPH D LAH nn Q thuc AH . Ch
rng KL k DE, ta c AF N D AKL D 180 APN nn t gic APNF ni tip. Ta thy
F NQ D FAQ D DBC D FEC do vy NQ k CE. V PNG D P GT D PLM
PN
PH
PQ
nn NH k LM . Ta c
D
D
suy ra NQ k LT . T LT k NQ k CE nn T
PL
PM
PT
thuc CE (pcm).
Nhn xt.
1. y l mt bi ton hay bi cch pht biu gin n v tnh khng cn xng ca cc yu t
trong gi thit bi ton. Li gii trn c hon thin vi s cng tc ca thy Trn Quang Hng
gio vin Trng THPT Chuyn KHTN, HKHTN HQG H Ni.
PH
PQ
2. Trong li gii trn, ta chng minh c
D
nn HQ k M T . Ch rng Q thuc
PM
PT
AH nn ta c M T ? BC , suy ra T thuc trung trc ca BC .
Khi phc tnh cn xng ca cc yu t trong gi thit bi ton 9, ta c th xut cc bi ton
sau:
84

Tp ch Epsilon, S 06, 12/2015


Bi ton 10. Cho tam gic ABC ni tip ng trn .O/. Cc ng cao h t B; C ca tam
gic ABC theo th t ct .O/ ti D v E. DE theo th t ct AC v AB ti M v N . K
DP k EQ k BC.P 2 AC; Q 2 AB/. Tip tuyn ti M; Q ca ng trn ngoi tip tam gic
AMQ ct nhau ti S . Tip tuyn ti N; P ca ng trn ngoi tip tam gic ANP ct nhau
ti T . Chng minh rng ST vung gc vi BC .

Bi ton 11. Cho tam gic ABC ni tip ng trn .O/ c trc tm H . BH; CH theo th t
ct .O/ ti D v E. K DM k EN k BC.M 2 AC; N 2 AB/. Trung trc ca BC theo th
t ct BH; CH ti P v Q. Chng minh rng ng trn ngoi tip cc tam gic HMP v
HNQ ct nhau ti mt im thuc .O/.

A
M

G
P

L
H
B

O
Q

Li gii. Gi BK; CL l cc ng cao ca tam gic ABC . ng trn ng knh AH ct


.O/ ti G khc A. Gi S l trung im ca BC th G; H; S thng hng. V LGS D LAH D
LCS nn t gic C GLS ni tip. V SC D SL nn GS l phn gic LGC . T cc cp tam
gic ng dng GLH v GSC , LHN v SCQ ta c hai tam gic GLN v GSQ ng dng. Suy
ra hai tam gic GNQ v GLS ng dng. Do ta c NGQ D LGS D LAH D NHL
nn t gic HNGQ ni tip hay ng trn .HNQ/ i qua G thuc .O/. Chng minh tng t
ta c ng trn .HMP / cng i qua G. T ta c pcm.

Nhn xt. T li gii bi ton 11, chng ta c mt cch tip cn n gin hn cho bi ton 9 nh
sau:
85

Tp ch Epsilon, S 06, 12/2015

A
D

F
K

G
L

O
H

T
M

Li gii th hai cho bi ton 9. Gi BK; CL l cc ng cao ca tam gic ABC . ng


trn .AKL/ ct .O/ ti P khc A, ta chng minh c t gic AP GF ni tip. Gi M l trung
im ca BC th P; H; M thng hng. Trung trc ca BC ct CH ti T . Tng t nh chng
minh bi ton 11, ta c hai tam gic P GT v PLM ng dng. Ch rng hai tam gic P GF
v PLK ng dng, ta thy hai tam gic MKL v TF G ng dng. Ch rng MK; ML l
tip tuyn ca ng trn .AKL/, suy ra TF; T G l tip tuyn ca ng trn .AF G/. Ta c
pcm.

n giu cu hnh v trc tm thng qua tm ng trn ni tip tam gic, chng ta c th
xut nhiu bi ton hay v kh. Chng hn nh:

Bi ton 12. Cho tam gic ABC ngoi tip ng trn .I /. D; E; F theo th t l trung im
ca AI; BI; CI . M l trung im BC . A1 ; A2 theo th t l giao im khc M ca cc ng
trn .IEM /; .IFM / vi BC . EA1 ; FA2 ct nhau ti A3 . B3 ; C3 xc nh tng t. Chng
minh rng AA3 ; BB3 ; C C3 ng quy.
86

Tp ch Epsilon, S 06, 12/2015

A
Z
D
Y
I

O
E
B

F
A2

A1 M

A3

Gi . Gi AI; BI; CI theo th t ct ng trn .O/ ngoi tip tam gic ABC ti X; Y; Z.
Th th I l trc tm ca tam gic XY Z. T a v cu hnh bi ton 4, ta c kt qu EA1
ct FA2 ti A3 l giao im ca .O/ v ng trn ng knh IX. Theo tnh cht quen thuc,
A3 l tip im ca ng trn mixtilinear ni tip ng vi gc A ca tam gic ABC . Tng t
vi cc im B3 v C3 , t AA3 ; BB3 ; C C3 ng quy ti mt im thuc OI .
Bi ton 13. Cho tam gic ABC ni tip ng trn .O/, ngoi tip ng trn .I /. ng
trn .Oa / tip xc vi AB; AC v tip xc trong vi ng trn .O/ ti S. D; E theo th t l
cc giao im khc B; C ca cc ng trn SIB v SIC vi BC . ng thng qua B vung
gc vi AI ct IE ti M , ng thng qua C vung gc vi AI ct ID ti N . Ly P; Q thuc
BC sao cho IP k AB; IQ k AC . Chng minh rng ng thng ni trung im cc on
MQ; NP chia i OI .
Bi ton 14. Cho tam gic ABC ni tip ng trn .O/, ngoi tip ng trn .I /. AI ct
.O/ ti D khc A. E; F theo th t l trung im ca IB v IC . DE; DF theo th t ct BC
ti M v N . ng thng qua I vung gc vi AI theo th t ct AB; AC ti K; L. Gi P; Q
theo th t l giao im khc B; C ca cc ng trn .BIK/; .CIL/ vi BC . IP ct DF
ti R, IQ ct DE ti S. Chng minh rng ng thng ni trung im cc on MR; NS v
ng thng ni tm ng trn Euler ca cc tam gic DMR; DNS ct nhau ti mt im
thuc OI .
Bi ton 15. Cho tam gic ABC ni tip ng trn .O/, c I l giao im cc ng phn
gic. AI ct .O/ ti D khc A. E; F; M theo th t l trung im ca IB; IC v BC . Gi
H; K theo th t l giao im ca BC vi DE v ng trn .IMF /.K M /. IK ct DF ti
L. Chng minh rng tip tuyn ti H; L ca ng trn ngoi tip tam gic .DHL/ ct nhau
ti mt im thuc CI .
87

Tp ch Epsilon, S 06, 12/2015


Nh vy, tc gi trnh by xong mt gc nhn ca mnh v bi ton hnh hc s 4 trong k
VMO 2014. T nhng cu hnh rt n gin, ta c th a ra nhng khai thc rt th v thng
qua nhng gc nhn khc nhau.
Cui cng, tc gi xin gi li cm n ti thy Trn Quang Hng gio vin Trng THPT
Chuyn KHTN, HKHTN HQG H Ni, ngi c ton b bi vit v a ra nhng
kin qu bu gip bi vit c hon thin hn.

Ti liu tham kho


[1] Trn Nam Dng, L Phc L v Phan c Minh. thi chn hc sinh gii cp quc gia
THPT mn ton nm hc 2013 2014, Li gii chi tit v bnh lun. Din n Ton hc
Mathscope.org.
http://forum.mathscope.org/showthread.php?t=46403
[2] huynhcongbang. [VMO 2014] Bi 4 Hnh hc phng. Mathscope.org.
http://forum.mathscope.org/showthread.php?t=46327
[3] Nguyn Vn Linh. nh l ERIQ v ng dng. Euclidean Geometry Blog.
https://nguyenvanlinh.wordpress.com/2015/03/02/eriq-theorem/
[4] shobber. Collinearity of orthocentres. Art of Problem Solving.
http://www.artofproblemsolving.com/community/c6h97493p550605

88

MI LIN H EUCLIDE, AFIN V X NH QUA


MT BI TON TRONG SCH "CC PHNG PHP
GII TON QUA CC K THI OLYMPIC"
Nguyn Ngc Giang TPHCM

Tm tt
Bi vit a ra cc cch gii khc nhau cng nh nhng khm ph v pht trin ca
mt bi ton. Qua , th hin mi quan h bin chng gia hnh hc Euclide, Afin
v x nh.

1. Gii thiu
Chng trnh "Gp g ton hc 2015" kt thc tt p. Ngoi nhng hot ng nh dy hc,
giao lu th vic cho ra i cun sch "Cc phng php gii ton qua cc k thi Olympic" cng
phn no ni ln s thnh cng ca chng trnh. Vi ni dung phong ph, sch cp n
cc phng php gii v pht trin t duy dnh cho cc em hc sinh gii. Trong cc ton ra
cho cc em hc sinh gii ti c bit ch trng n bi ton sau:
Bi ton 1. ([1], tr. 205). Cho tam gic ABC: Cc im M; N thuc BC: Cc im P; Q theo
th t thuc AC; AB: t O D MP \ NQ; K D BO \ NP; L D CO \ MQ: Chng minh
rng: AO; BL; CK ng quy.
y l mt bi ton hay v gi ln nhiu suy ngh. V bi ton ch cp n cc khi nim, cc
tnh cht nh "tam gic", "ng quy", "giao im" nn bi ton l mt bi ton ca hnh hc x
nh. V l mt bi ton hnh hc x nh nn hin nhin n l mt bi ton hnh hc afin v cng
l mt bi ton trong hnh hc Euclide.
Trong bi bo, chng ti s cp n mi quan h gia ton s cp v cao cp qua bi ton.
Mi quan h th hin mt dng t duy rt hay l t duy sng to. Trong thuyt Gio dc
Bloom mi th t duy sng to l t duy cao nht v kh nht. y l dng t duy rt cn thit
khng nhng cho Gio dc m cn cho ton b mi lnh vc khoa hc v i sng. Sng to v
sng to hn na l kim ch nam cho mi hnh ng ca con ngi trong th k XXI.

2. Cc cch gii ton


2.1. Phng php chiu xuyn tm
t I D BL \ CK; U D BO \ MQ; V D CO \ NP: Ta c
B.ALOC / D .QLUM /
D .M ULQ/ (theo tnh cht t s kp ca hng im)
D .PK V N / (xt php chiu xuyn tm O)
D C.AKOB/:
89

Tp ch Epsilon, S 06, 12/2015

p dng nh l: Cho hai chm s; a; b; c v s; a0 ; b 0 ; c 0 . Khi .sabc/ D .sa0 b 0 c 0 / khi v ch khi


a \ a0 I b \ b 0 I c \ c 0 thng hng.
Vi s D BC; a D BA; a0 D CA; b D BL; b 0 D CK; c D BO; c 0 D CO th ta c A; I; O thng
hng. Vy AO; BL; CK ng quy. [1]
Ngoi cch gii ny ta cn c th gii bi ton bng cc phng php ta . Phng php ta
c u im l phng php c tnh cht thut ton. D vic tnh ton trong nhiu bi ton
nhiu khi gp kh khn nhng ngy nay ta c cc phn mm tr gip tnh ton nn cch gii
bng phng php ta vn l cch gii c a dng.

2.2. Phng php ta khi tm thun nht


Ta khi tm thun nht (Homogeneous barycentric coordinates) l phng php ta do
Mobius tm ra. Ni dung ca phng php nh sau. Cho tam gic ABC; im P bt k c ta
l b ba cc s .u W v W w/ hiu theo ngha l h cc khi tm u ti A; v ti B; v w ti C
sao cho im cn bng ca tam gic ti P . Cc khi tm t l vi din tch tam gic PBC; P CA
v PAB: Khi im P nm ngoi tam gic, ta s dng din tch c du ca cc tam gic nh
hng. Cc ta khi tm thun nht ca P vi tam gic tham chiu ABC l b ba cc s
.x W y W z/ tha mn
x W y W z D PBC W P CA W PAB:
V d: 1. Trng tm G ca tam gic c ta khi tm thun nht l G D .1 W 1 W 1/:
2. Tm ng trn ni tip I c ta khi tm thun nht l I D .a W b W c/:
Sau y l li gii bi ton bng phng php ta khi tm thun nht.
Ta k hiu phng trnh ng thng XY c cc thnh phn ta x; y; z l XY D x W y W z:
Phng trnh ng thng MP c ta l




0 1 1 p p 0

W
W

1 m m 0 0 1 D 1 W mp; p:
Tng t, phng trnh ng thng NP c ta l
1 W np W

p:

Phng trnh ng thng NQ c ta l





q 0 0 1 1 q



1 n W n 0 W 0 1
90

D q n W

n W 1:

Tp ch Epsilon, S 06, 12/2015

Tng t, phng trnh ng thng MQ c ta l


q m W

m W 1:

O D MP \ NQ nn ta im O tha mn h phng trnh



nqx1 nx2 C x3 D 0
x1 C mpx2 px3 D 0
Gii h ta c O D .p.n m/ W npq C 1 W n.1 C mpq//:
Phng trnh ng thng AO c ta l



npq C 1 n.1 C mpq/ n.1 C mpq/ p.n m/ p.n m/ npq C 1

W
W



0
0
0
1
1
0
D 0 W n.1 C mpq/ W .npq C 1/:

Phng trnh ng thng BO c ta l





npq C 1 n.1 C mpq/ n.1 C mpq/ p.n m/ p.n m/ npq C 1

W
W



1
0
0
0
0
1
D n.1 C mpq/ W 0 W p.n m/:

Phng trnh ng thng CO c ta l





npq C 1 n.1 C mpq/ n.1 C mpq/ p.n m/ p.n m/ npq C 1

W
W



0
1
1
0
0
0
D npq C 1 W p.m n/ W 0:
K D NP \ BO nn ta im K tha mn h phng trnh

x1 C npx2 px3 D 0
n.1 C mpq/x1 C p.n m/x3 D 0
Gii h ta c
K D np.n


m/ W m.npq C 1/ W n2 .1 C mpq/ :

Phng trnh ng thng CK c ta l m.npq C 1/ W np.m


91

n/ W 0.

Tp ch Epsilon, S 06, 12/2015


L D CO \ MQ nn ta im L tha mn h phng trnh

.npq C 1/x1 C p.m n/x2 D 0
mqx1 mx2 C x3 D 0
Gii h ta c
L D .p.n

m/ W npq C 1 W m.1 C mpq//:

Phng trnh ng thng BL c ta l


m.1 C mpq/ W 0 W p.n

m/

Xt nh thc

m.npq C 1/
np.m n/

0
D m.1 C mpq/

0
n.1 C mpq/
D m.npq C 1/:. n.1 C mpq//:p.n
D mnp.npq C 1/.1 C mpq/.n m/.
D 0:

p.n m/
.npq C 1/
m/ C m.1 C mpq/:np.m
1 C 1/
0

n/:. .npq C 1//

Vy BL; CK; AI ng quy ti mt im.

2.3. Phng php ta afin


Cc tnh ton trong cch gii ny c tr gip bi phn mm Maple XVIII.

! !
Xt mc tiu afin fAI AB; AC g, ta c
A D .0I 0/; B D .1I 0/; C D .0I 1/:
Phng trnh ng thng BC l
.BC / W x C y
M; N 2 BC nn M D .mI 1 m/; N D .nI 1
P 2 AC nn ta im P l

n/:

92

1 D 0:

Tp ch Epsilon, S 06, 12/2015

P D .0I p/:
Q 2 AB nn ta im Q l
Q D .qI 0/:
Phng trnh ng thng MQ l
x xM
y yM
D
xM xQ
yM yQ
hay
y

x m
m q

.MQ/ W

1Cm
D 0:
1 m

Tng t, phng trnh ng thng MP l


m

y
1

1Cm
D 0:
m p

x
n

n
q

1Cn
D 0:
1 n

y
1

1Cn
D 0:
n p

x
m
Phng trnh ng thng NQ l

Phng trnh ng thng NP l


n

V O D MP \ NQ nn ta ca im O tha mn h phng trnh


8 x

<
x

:
n

m
m

n
q

y 1Cm
D0
1 m p
y 1Cn
D0
1 n

Gii h ta c

OD

m.np nq pq C q/
mnp
I
mq np C pq m C n q

mnq npq mp C mq C nq C pq
mq np C pq m C n q

Phng trnh ng thng CO l


x.mq np C pq m C n q/
m.np nq pq C q/

y
1

mnp mnq npq mpCmqCnqCpq q


mq npCpq mCn q

D0

Phng trnh ng thng BO l


x
1C

m.np nq pqCq/
mq npCpq mCn q

mnp

y.mq np C pq m C n q/
mnq npq mp C mq C nq C pq
93

D 0:


:

Tp ch Epsilon, S 06, 12/2015


L D CO \ MQ; nn ta im L l L D .xL I yL / vi
xL D

m2 p

.np nq pq C q/m2
2mnp C mnq C npq m2 C mn

nq

m2 p m2 q mpq mp C 2mq C pq q n
:
yL D 2
m p 2mnp C mnq C npq m2 C mn nq

K D BO \ NP; nn ta im K l K D .xK I yK / vi
xK D

mnp

.mnp mnq mpq C mq np C nq C pq q/n


2mnq mpq C n2 q C mn C 2mq n2 np C pq m C n


mn2 p mn2 q n2 pq 2mnp C 2mnq C n2 q C 2npq C mp mq 2nq pq C q
:
mnp 2mnq mpq C n2 q C mn C 2mq n2 np C pq m C n q

yK D

Phng trnh ng thng CK l


2mnq mpq C n2 q C mn C 2mq n2 np C pq m C n
.mnp mnq mpq C mq np C nq C pq q/n

x mnp

y
1C

.mn2 p

mn2 q n2 pq 2mnpC2mnqCn2 qC2npqCmp mq 2nq pqCq /


mnp 2mnq mpqCn2 qCmnC2mq n2 npCpq mCn q

D 0:

Phng trnh ng thng BL l


x
1C
y m2 p
C
.m2 p

.np nq pqCq/m2
m2 p 2mnpCmnqCnpq m2 Cmn nq


2mnp C mnq C npq m2 C mn nq
D 0:
m2 q mpq mp C 2mq C pq q/n

Phng trnh ng thng AO l



nq
x C mqm.np
npCpq
AO D


pqCq
mCn q

m.np

mnp mnq npq mpCmqCnqCpq


mq npCpq mCn q

mnp

mnq

npq

np C pq

mCn

q/

nq pq C q/

q
.mq np C pq

mCn

q/

.mq

mp C mq C nq C pq

D 0:

I D CK \ BL nn ta im I D .xI I yI / l
xI D
2

Cn pq

.mnp
2

m n

mnq
2

m p

mpq C mq
2

np C nq C pq

m q C mn C 2mnp

n q

q/mn/

m2 nq C m2 pq

npq C m
mn C nq

mn2 p

yI D n m2 np m2 nq mnpq m2 p C m2 q mnp C 2mnq C mpq C npq C mp


2mq nq pq C q// 2
m nq C m2 pq mn2 p mnpq
C n2 pq m2 n m2 p

m2 q C mn2 C 2mnp n2 q npq C m2 mn C nq
94

mnpq

Tp ch Epsilon, S 06, 12/2015


Xt hiu 
D .xA
D .mnp

xI /.yO
mnq

yI /

mpq C mq

.yA

yI /.xO

xI /: 

mnq npq mp C mq C nq C pq
mq np C pq m C n q
mnp C 2mnq C mpq C npq C mp 2mq nq pq

np C nq C pq

q/mn

mnp

n m2 np m2 nq mnpq m2 p C m2 q
Cq// 2
m nq C m2 pq mn2 p mnpq C n2 pq m2 n m2 p m2 q C mn2 C 2mnp n2 q npq C m2
mn C nq/// 2
m nq Cm2 pq mn2 p mnpq C n2 pq m2 n m2 p m2 q C mn2 C 2mnp n2 q
2
Ta c npq C m mn C nq C n m2 np m2 nq mnpq m2 p C m2 q mnp C 2mnq C mpq C npq
m.np nq pq C q/
Cmp 2mq nq pq C q/
mq np C pq m C n q
C. .mnp mnq mpq C mq np C nq C pq q /mn m2 nq C m2 pq mn2 p mnpq C n2 pq

m2 n m2 p m2 q C mn2 C 2mnp n2 q npq C m2 mn C nq 2
m nq C m2 pq
mn2 p

mnpq C n2 pq m2 n m2 p m2 q C mn2 C 2mnp n2 q npq C m2 mn C nq :

n gin ha bng lnh simplify ca Maple XVIII, ta c D 0:


Vy AO; BL; CK ng quy.

2.4. Phng php ta x nh

Xt mc tiu x nh fA; B; C I O/: Ta c:


A D .1; 0; 0/I B D .0; 1; 0/I C D .0; 0; 1/:
Phng trnh ng thng AB l x3 D 0:
Phng trnh ng thng AC l x2 D 0:
Phng trnh ng thng BC l x1 D 0:
V M thuc BC nn M D B C C hay ta c h phng trnh
2
3
2 3
2 3
m1
0
0
4 m2 5 D 4 1 5 C 4 0 5:
m3
0
1
Vy M D .0; 1; m/:
Tng t, ta cng c N D .0; 1; n/:
Phng trnh ng thng MO c ta l
95

Tp ch Epsilon, S 06, 12/2015


1 1

1 m




1 1 1 1
;

m 0 ; 0 1 D m

1; m; 1:

Tng t, phng trnh ng thng NO l n 1; n; 1:


P D MO \ AC nn ta im P tha mn h phng trnh

.m 1/x1 mx2 C x3 D 0
x2 D 0
Vy P D . 1; 0; m 1/:
Q D NO \ AB nn ta im Q tha mn h phng trnh

.n 1/x1 nx2 C x3 D 0
x3 D 0
Vy Q D .n; n 1; 0/:
Phng trnh ng thng MQ c ta l



1
m m 0 0
1

n 1 0 ; 0 n ; n n 1
Phng trnh ng thng NP c ta l


0 m 1 m 1
1
;

1
n n
0

D m.1

n/; mn; n:


1 0
D 1
0 1

m; n; 1:

Phng trnh ng thng BO l x1 x3 D 0.


Phng trnh ng thng CO l x1 x2 D 0:
K D BO \ NP nn ta im K tha mn h phng trnh

.1 m/x1 C nx2 x3 D 0
x1 x3 D 0
Vy K D .n; m; n/:
L D CO \ MQ nn ta im L tha mn h phng trnh

m.1 n/x1 C mnx2 nx3 D 0
x1 x2 D 0
Vy L D .n; n; m/:
ng thng BL c ta l




1 0 0 0 0 1

;
;

n m m n n n D m; 0; n:
ng thng CK c ta l




0 1 1 0 0 0

;
;

m n n n n m D m; n; 0:
ng thng AI c ta l


0 0 0 1


1 1 ; 1 0



1 0
;

0 1 D 0; 1; 1
96

Tp ch Epsilon, S 06, 12/2015


Xt nh thc

D m
0

0
n
1

n
0
1

D m:n

. m/:. n/ D 0:

Vy AI; BL; CK ng quy ti mt im.

3. Tng t ha bi ton
T mt bi ton x nh trong khng gian x nh, bng cch chn cc siu phng khc nhau ng
vai tr siu phng v tn ta c th c nhiu bi ton afin khc nhau m cc kt qu ta c th suy
ra t nhng kt qu bit trong khng gian x nh.
Bng cch chn BC l ng thng v tn, ta c bi ton sau
Bi ton 2. Cho hnh bn nh APOQ. ng thng qua Q song song vi PO ct ng thng
qua O song song vi AP ti L: ng thng qua P song song vi QO ct ng thng qua O
song song vi AQ ti K: Dng hnh bnh hnh OLIK. Chng minh rng A; I; O thng hng.

Bng cch chn ng thng AC lm ng thng v tn, ta c bi ton sau


Bi ton 3. Cho hai tia Bx; By. Trn Bx ly hai im M; N v trn By ly im Q. Gi O l
im trn QN: ng thng qua N song song vi MO ct BO ti K: ng thng qua O song
song vi Bx ct QM ti L: ng thng qua K song song vi Bx ct BL ti I: Chng minh
rng OI k By:

4. M rng bi ton
T bi ton 1, chng ti tng qut ha thnh bi ton cho ng trn. Ta c bi ton sau
Bi ton 4. Cho 6 im M; B; Q; P; C; N nm trn ng trn. Gi I l giao im ca MP
v NQ: L; K ln lt l giao im ca CI vi MQ v BI vi NP: Gi T l giao im ca BL
v CK: Chng minh rng A; T; I thng hng.
97

Tp ch Epsilon, S 06, 12/2015

Sau y l mt s cch gii ca bi ton.


Chng minh. Cch 1 [2]

Gi S D BM \ CN:
p dng nh l Pascal cho lc gic MBQP CN ta c A; I; S thng hng.
Gi X1 ; X2 ; Y1 ; Y2 ln lt l giao im ca BC vi MQ; NP; MP; NQ:
chng minh A; I; T thng hng ta s chng minh B.CAT S/ D C.BAT S/ tc l .X1 QLM / D
.X2 PKN / .1/
Ta c, p dng nh l Mnlaus cho tam gic QX1 Y2 vi L; I; C thng hng
.X1 QLM / D

LX1 MX1
I Y2 CX1 MQ
W
D
:
:
LQ MQ
IQ C Y2 MX1

Tng t cho .X2 PKN /:


98

Tp ch Epsilon, S 06, 12/2015


Khi (1) tng ng vi
I Y2 CX1 MQ
I Y1 BX2 NP
:
:
:
D
:
IQ C Y2 MX1  IP BY1 NX
 2
MQ IP
CX1 BX2
I Y1 I Y2
:
,
:
W
D
W
:
NP IQ
MX1 NX2
BY1 C Y2
CX1
CQ BX2
BP
D
I
D
:
M X1
MB NX2
CN
.1/ tng ng vi
Ch

!

\ sin I[
sin QIM
CQ CN
I Y1 I Y2
NP
:
D
W
:
:
:
MB BP
BY1 C Y2
\ sin PI
[
sin IMQ
N

.2/

p dng nh l sin cho tam gic IBY1 v IBM ta c


[
[
I Y1
sin IBC
sin IBC
D
D
:
\
BY1
\
sin BIM
MB: sin BMP
IB
Tng t cho
Vy ta c

I Y2
:
C Y2
[
[
sin IBC
I Y1 I Y2
sin ICB
W
D
W
:
\
\
sin CNQ
BY1 C Y2
MB: sin BMP
NC:
IB
IC

.3/

T (2), (3), ta cn chng minh


[ sin CNQ
\
CQ NC
I Y1 C Y2
IB NC sin IBC
:
D
:
D
:
:
:
MB BP
BY1 I Y2! MB IC !sin BMP
\ sin ICB
[
[
\
IB
sin IBC
sin CNQ
CQ
,
:
:
D
IC
BP
[
\
sin ICB
sin BMP
CQ
BP
CQ
BP
BC
BC
,
D
,
D
,
D
:
\
\
\
[
\
\
sin CNQ
sin BMP
sin QBC
sin BCP
sin BQC
sin BP
C
iu ny l hin nhin. Vy, ta c iu phi chng minh.
Cch 2 [3]
Gi D D BM \ CN; U D BI \ MQ v V D CI \ NP:
p dng nh l Pascal cho b 6 im B; Q; N; C; P; M nm trn ng trn .O/ ta c A D
BQ \ CP; I D QN \ PM v D D NC \ MB nn A; I v D thng hng.
Tip theo ta chng minh T nm trn ng thng AID bng phng php t s kp. Ta c
B.ALID/ D .QLUM /
D .M ULQ/ (theo tnh cht t s kp ca hng im)
D .PK V N / (xt php chiu xuyn tm I )
D C.AKID/:
T , suy ra BL v AK ct nhau ti mt im T trn AID hay A; T; I thng hng.
99

Tp ch Epsilon, S 06, 12/2015

Thc hin php chiu xuyn tm t im O 0 nm ngoi mt phng t giy xung mt phng
khng song song vi mt phng t giy th th ng trn khng cn l ng trn na m tr
thnh thit din cnic (elip, hypebol, parabol). T bi ton 4 ta thu c bi ton sau
Bi ton 5. Cho 6 im M; B; Q; P; C; N nm trn thit din cnic. Gi I l giao im ca
MP v NQ: L; K ln lt l giao im ca CI vi MQ v BI vi NP: Gi T l giao im
ca BL v CK: Chng minh rng A; T; I thng hng.
Bi ton tng qut ca bi ton 5 l bi ton sau
Bi ton 6. Cho 6 im M; B; Q; P; C; N nm trn cnic .S/. Gi I l giao im ca MP v
NQ: L; K ln lt l giao im ca CI vi MQ v BI vi NP: Gi T l giao im ca BL v
CK: Chng minh rng A; T; I thng hng.
chng minh bi ton 6, ta s dng cch chng minh 2 ca bi ton 4.
Chng minh. Gi D D BM \ CN; U D BI \ MQ v V D CI \ NP:
p dng nh l Pascal cho b 6 im B; Q; N; C; P; M nm trn cnic .S/ ta c A D BQ \
CP; I D QN \ PM v D D NC \ MB nn A; I v D thng hng.
Tip theo ta chng minh T nm trn ng thng AID bng phng php t s kp. Ta c
B.ALID/ D .QLUM /
D .M ULQ/ (theo tnh cht t s kp ca hng im)
D .PK V N / (xt php chiu xuyn tm I )
D C.AKID/:
T suy ra BL v AK ct nhau ti mt im T trn AID hay l A; T; I thng hng. Vy
AI; BL; CK ng quy.
100

Tp ch Epsilon, S 06, 12/2015

5. Kt lun
Chng ta va c mt s khm ph th v t mt bi ton. Cc cch gii khc nhau, cc bi ton
m rng mang n cho chng ta nhiu iu b ch. Bi vit ny cn trao i g thm? Mong
c s chia s ca cc bn.

Ti liu tham kho


[1] Hunh Ch Ho (2015), "Hng im iu ha", Cc phng php gii ton qua cc k
thi Olympic (Trn Nam Dng (ch bin), V Quc B Cn, Trn Quang Hng, L Phc
L, Nguyn Tt Thu), Chng trnh trng im quc gia pht trin ton hc giai on
2010-2020.
[2] V Duy Khnh, Din n T1K23-Huyn Thoi CHT.
[3] Nguyn Chng Ch, Din n BI TON HAY LI GII P AM M TON
HC

101

Tp ch Epsilon, S 06, 12/2015

102

V BI HNH HC THI IMO NM 2009


NGY TH HAI
Trn Quang Hng THPT chuyn KHTN, H Ni

Tm tt
Bi vit xoay quanh bi ton hnh hc thi IMO nm 2009 ngy th hai vi cc m
rng v khai thc ng thi ng dng cc m rng v khai thc vi cc cng c
hnh hc thun ty.

1. M u
Trong k thi IMO nm 2009 c ngy th nht v ngy th hai u c cc bi ton hnh hc. Trong
khi bi ton hnh hc ngy th nht l mt bi ton ln c khai thc rt nhiu vi cc m
rng v ng dng th bi hnh hc ngy th hai t c ch hn, nhng thc ra cng l
mt bi ton v cng th v v su sc. Bi ton nh sau
Bi ton 1. Cho tam gic ABC cn ti A. Phn gic cc gc CAB v ABC ct cc
cnh BC v CA ln lt ti D v E. Gi K l tm ni tip tam gic ADC . Gi s rng
BEK D 45 . Tm tt c gi tr c th c ca gc CAB.
Bi ton ny c ngh bi ba tc gi l Jan Vonk, Peter Vandendriessche t nc B v
Hojoo Lee t nc Hn Quc. Bi ton c xp vo v tr th 4 c coi l bi ton d ca
ngy 2. y c th coi l mt bi ton tnh gc rt p mt. C rt nhiu li gii c ngh
trong [1]. Ti xin gii thiu li gii ti coi l n gin nht ly theo tng c ngh bi
nick name BlackMax trong [1]

A
F
E
I
K
B

Li gii. Gi I l giao ca AD v BE cng l tm ni tip tam gic ABC . F l hnh chiu ca


I trn AC . Nu F trng E th BE va l ng cao va l phn gic nn tam gic ABC u.
Trong trng hp ny d thy BEK D 45 tha mn bi.
Nu F khng trng E. D thy F v D i xng nhau qua IC nn IFK D IDK D 45 D
IEK. T t gic IFEK ni tip, ta suy ra IKE D 90 vy tam gic IEK vung cn hay
103

Tp ch Epsilon, S 06, 12/2015


KI D KE. Vn t t gic IFEK ni tip nn FK l phn gic IFE. T K l tm bng
tip tam gic IAF m IKE D 90 , ta suy ra IAE D 45 hay tam gic ABC vung cn.
Trong trng hp ny cng d thy BEK D 45 tha mn bi.
Nhn xt. Theo chng ti tng ca li gii trn l ngn gn v thun ty hnh hc rt p.
Trong [1] cn rt nhiu li gii tnh ton khc. im th v ca bi ton ny l gc BAC c
th nhn hai gi tr ng vi trng hp tam gic u v tam gic vung cn cng l hai tam gic
c bit nht. Chng ta thy rng trng hp tam gic u "tm thng" hn trng hp tam
gic vung cn. V vy ta c th pht biu thnh mt bi ton thun nh sau
Bi ton 2. Cho tam gic ABC vung cn ti A vi cc phn gic AD; BE. K l tm ni tip
tam gic ADC . Chng minh rng BEK D 45 .
Mc d trong trng hp trn bi ton hon ton khng kh nhng trn m hnh bi ton ny li
c rt nhiu c im th v khai thc.

2. M rng v ng dng
Trong cch gii bi ton 1 trng hp khi F khng trng E ta ch n chi tit th v l D v
F i xng nhau qua IC . Nh ta c ngay mt m rng th v sau
Bi ton 3. Cho tam gic ABC c phn gic AD; BE. K l tm ni tip tam gic ADC . Gi
s BEK D ADK. Chng minh rng CB D CA hoc DA D DC .

A
F

I
K
B

Li gii. Gi F l i xng ca D qua IC . Nu F trng E d thy IE; ID i xng nhau qua


IC nn CB D CA.
Nu F khng trng E th bn im I; E; F; K thuc mt ng trn. Nu F nm ngoi
1
on EC th ta c 180 D IEK C EKI C KIE D ADC C IFA C CIE D
2
B
A
A
B
C
3A
3C
C
C
C C C
C
D
C B C
. Ta suy ra A D 2C hay
2
4
2
2
2
4
2
DA D DC . Nu F nm trong on EC ta cng c biu thc gc tng t. T ta c iu phi
chng minh.
Nhn xt. Tc gi s dng tng li gii ca bi ton 1 tm ra m rng ny tuy nhin khi
xem k li [1] th m rng cng xut hin trong [1]. Tuy nhin li gii trn khc hon ton
cc li gii trong [1]. Chng ta cng hon ton c th pht biu thnh mt bi ton thun nh sau
104

Tp ch Epsilon, S 06, 12/2015


Bi ton 4. Cho tam gic ABC ni tip .O/ c phn gic AD; BE. K l tm ni tip tam gic
ADC . Gi s tam gic DAC cn ti D. Chng minh rng t gic BDKE ni tip.
Khi pht biu bi ton thun nh bi ton 2, ta c rng EK khi cng i qua im L i
xng ca I qua BC mt khc khi d thy BAL D 45 D BEL nn t gic BAEL ni
tip. Ngay lp tc cho ta mt cu hi l liu ta c th thay th c tam gic vung cn dn
ti mt bi ton tng qut trn tam gic vung bt k. Cu hi c gii quyt trn vn trong
bi ton sau
Bi ton 5. Cho tam gic ABC vung ti A. ng cao AH vi H thuc BC . I l tm ni
tip tam gic ABC . BI ct CA ti E. K l tm ni tip tam gic AH C . KE ct IH ti L.
Chng minh rng t gic BAEL ni tip.

A
E
I
K
H

B
L

Li gii. T cc h thc lng trong tam gic vung, ta d chng minh CI:CK D CE:CH
t 4CEK  4CIH suy ra CEK D CIH vy t gic CEIL ni tip. T ta c
IL:IE
IL
IL
IK
IK CE
IB
D
suy ra
D
:
. Ta s chng minh
D
ng thc tng
CL:CE
KC
CL
KC IE
CL
AC
ng
IK CE
IB
:
D
KC IE
AC
CE
IB IE
,
D
:
(Do 4CEK  4CIH )
CK
AC IK
CI
IB
IE
D
: 2
(Do 4AIK  4CIA)
,
CH
AC AI =CI
BC
IB:IE
,
D
AC 2
AI 2 :AC
, BC:AI 2 D IB:IE:AC .
ng thc cui ta d kim tra nh cc h thc lng trong tam gic vung. Vy kt hp
BIL D ACL suy ra 4BIL  4ACL suy ra LBI D LAC nn t gic BAEL ni
tip. Ta c iu phi chng minh.
Nhn xt. Ta nhn thy ngay mt h qua n gin l t BAEL ni tip th BLE D 90 . Ta li
c ELC D EIC D 45 . Suy ra BLC D 90 C 45 D 135 . Li t BLC D 135 ta d
d cha A ca ng trn ngoi
chng minh tm ngoi tip tam gic BLC l trung im cung BC
tip tam gic ABC .
Ta c ngay mt h qu rt n gin ca bi ton ny l bi ton sau
105

Tp ch Epsilon, S 06, 12/2015


Bi ton 6. Cho tam gic ABC vung ti A vi ng cao AH v phn gic BE. K l tm
ni tip tam gic AHB. Gi s BEK D 45 , chng minh rng tam gic ABC vung cn.
Li gii. Gi I l tm ni tip tam gic ABC v EK ct IH ti L. Theo bi trc th t gic
BAEL ni tip nn BAL D BEL D 45 D BAI . T AL i qua I hay A; H; L; I
thng hng, vy tam gic ABC vung cn.
Ngay lp tc mt cu hi li c t ra l bi ton trn pht biu cho tam gic vung, vy th
vi mt tam gic bt k th sao ? Ta c mt m rng th v bi ton trong phn nhn xt cho tam
gic bt k nh sau, bi ton tham kho [2]
Bi ton 7. Cho tam gic ABC c phn gic BE. ng trn qua A; B tip xc AC ct BC
ti D khc B. K l tm ni tip tam gic ADC . EK ct ng trn ngoi tip tam gic ABE
ti L. Chng minh rng tm ca ng trn ngoi tip tam gic LBC nm trn ng trn
ngoi tip tam gic ABC .

E
J
I

B
L

Li gii th nht. Gi I l tm ni tip tam gic ABC . ng trn ngoi tip tam gic AIE
CK
ct CI ti J khc I . Ta thy CI:CJ D CE:CA. Li c 4CKA  4CIB nn
D
CI
CA
CD
CK
D
. T CK:CJ D
:.CI:CJ / D CE:CD nn 4CEK  4CJD suy ra
CB
AC
CI
CEK D CJD. Ta nh ngha li im L l giao ca DJ v KE vy t gic CEJL ni tip.
Suy ra DLK D KCE D KCD suy ra t gic DKCL ni tip. Cng t 4CKA  4CIB
nn AKJ D CIE D CAJ . T JA2 D JK:J C D JD:JL suy ra 4JAD  4JLA
1
suy ra ALD D JAD D DAC JAC D ABC AKJ D ABC
.DAB C
2
1
1
ACB/ D .ABC ACB/. Suy ra ALE D ALDCDLE D .ABC ACB/C
2
2
1
1
ACB D ABC D ABE. T t gic ABLE ni tip. T d chng minh BLC D
2
2
1

d cha A ca ng
180
BAC nn tm ngoi tip S ca tam gic BLC l trung im BC
2
trn ngoi tip tam gic ABC . Ta c iu phi chng minh.
106

Tp ch Epsilon, S 06, 12/2015

N
A

E
K

L
D

1
1
DAC D ABC D CBE
2
2
vy suy ra t gic AEFB ni tip. Do BE l phn gic ABC nn E l im chnh gia cung
LA
KA
CA
D
D
. Gi CL ct ng trn ngoi tip
AF nn LE l phn gic ALB suy ra
LF
KF
CF
tam gic ABE ti L khc M . D thy cc tam gic ng dng gc gc 4CEM  4CLA v
ME
LA
LF
MB
4CBM  4CLF . T
D
D
D
suy ra LC l phn gic BLE hay
MC
CA
CF
MC
1
BLC D CLE D BAE. Gi N l im chnh gia cung BC cha A ca ng trn
2
ngoi tip tam gic ABC th NB D NC v BNC D BAC D 2BLC . T N l tm
ngoi tip tam gic BLC .
Li gii th hai. Gi AK ct BC ti F . Ta c FAC D

E
I
K

N
L
Li gii th ba. Gi I l tm ni tip tam gic ABC . LC ct ng trn ngoi tip tam gic
ABE ti N khc L. AK ct BC ti M . D c AMB D MAC C ACB D EBC C
107

Tp ch Epsilon, S 06, 12/2015


ACB D AEB nn M cng nm trn ng 
trn ngoi tip
 tam gic ABE. Ta ch C; K; I
EN M
thng hng, p ng nh l Pascal o cho b
suy ra A; I; N thng hng. T
AB L
1
1
ELC D CAN D BAC vy BLC D 180
BAC nn tm ngoi tip S ca tam
2
2
d cha A ca ng trn ngoi tip tam gic ABC .
gic BLC l trung im BC
Nhn xt. Li gii th nht do tc gi ngh. tng li gii ny da vo tng trong li
gii bi ton 5 cng ca tc gi. Li gii th hai do bn Nguyn L Phc ngh. Li gii th
ba do bn L Th Hi Linh ngh. Cc li gii ny ph thuc nhiu vo v tr cc im trn
hnh v gc, tuy vy n c nt p c trng ca hnh hc thun ty. trnh vic ph thuc
hnh v cc bn c th a vo cc bin i gc nh hng tuy nhin iu ny cng khng thc
s cn thit. Bi ton 5 v bi ton 7 l cc bi ton tng qut c nhiu gi tr ng dng, cc bn
c th lm cc bi ton sau ng dng cc bi ton
Bi ton 8. Cho tam gic ABC vung ti A vi ng cao AH . Phn gic BE; CF ct nhau
ti I . Gi K; L l tm ni tip cc tam gic AH C; AHB. J i xng vi I qua BC . Chng
minh rng EK v FL ct nhau trn ng trn ngoi tip tam gic JBC .
Bi ton 9. Cho tam gic ABC vung ti A vi ng cao AH . Phn gic BE; CF ct nhau
ti I . Gi K; L l tm ni tip cc tam gic AH C; AHB. Chng minh rng EK; FL v IH
ng quy.
Bi ton 10. Cho tam gic ABC vi ng cao AH . M; N thuc BC sao cho AM ?
AB; AN ? AC . I l tm ni tip tam gic AMN . NI; MI ct CA; AB ln lt ti E; F . Gi
K; L l tm ni tip tam gic AHM; AHN . Chng minh rng EK; FL v IH ng quy.
Bi ton 11. Cho tam gic ABC vi phn gic BE; CF . ng trn qua A; B tip xc AC ct
BC ti M khc B. ng trn qua A; C tip xc AB ct BC ti N khc C . Gi K; L l tm
ni tip cc tam gic ACM; ABN . EK; FL ln lt ct cc ng trn ngoi tip tam gic
ABE; ACF ti P; Q khc E; F . Chng minh rng P; Q; B; C cng thuc mt ng trn.
Bi ton 12. Cho tam gic ABC ni tip .O/ c phn gic AD; BE. K l tm ni tip tam
gic ADC . Gi EK ct AD ti L. LB ct ng trn ni tip tam gic ABE ti N khc B.
Gi s tam gic DAC cn ti D, chng minh rng tm ngoi tip tam gic NBC nm trn .O/.
Cui cng, tc gi xin gi li cm n ti bn Nguyn Tin Dng sinh vin K50 i hc ngoi
thng ngi c ton b bi vit v a ra nhng gp gip tc gi hon thin bi vit ny.

Ti liu tham kho


[1] IMO 2009, problem 4
http://artofproblemsolving.com/community/c6h289054p1562847
[2] Mi tun mt bi ton: Tun 4 thng 9 nm 2015
http://analgeomatica.blogspot.com/2015/09/moi-tuan-mot-bai-toan-tuan-4-thang-9.html

108

THUT TON THAM LAM


TRONG XY DNG CU HNH T HP
Trn Minh Hin
(THPT Chuyn Quang Trung, Bnh Phc)
Thut ton tham lam l tm ti u a phng mi bc i vi hy vng tm c ti u ton
cc. D nhin thut ton tham lam khng m bo c ti u a phng s cho ta li gii ti
u ton cc. Nhng trong nhiu bi ton, vic ny s xy ra. V d:"Cho trc mt tp hp S cc
ng xu. Khi cn ly t nht bao nhiu ng xu trong S c tng s tin M cho trc.
Khi S D f1; 5; 10; 25g, chng ta c th p dng thut ton tham lam xy dng phng n ti
u nh sau:
 Thm ng xu x ln nht trong S sao cho x  M ;
 p dng li thut ton cho M

x.

V d, vi M D 91, u tin ta chn 25 trong S, sau li tip tc p dng thut ton cho
91 25 D 66. Khi ta li tip tc chn s 25.Tip tc p dng thut ton cho 66 25 D 41, ta
chn s 25. Li p dng thut ton cho 41 25 D 16, ta s chn s 10. Li tip tc p dng thut
ton cho 16 10 D 6, th ta chn s 5. V cui cng p dng thut ton cho 6 5 D 1, ta chn
s 1. T tp hp ti u l f25; 25; 25; 10; 5; 1g. Tc l vi tp S D f1; 5; 10; 25g thut ton
tham lam cho ta phng n ti u. Tuy nhin thut ton ti u khng phi lc no cng
cho ta phng n ti u. V d vi S D f1; 3; 4g v M D 6, thut ton tham lam cho ta tp
f1; 1; 4g, tuy nhin phng n ti u l f3; 3g."
Di y l mt s bi ton minh ha.
Cu 1 (BMO 1998). Mt i l v tu ha phn phi v tu ha cho 200 i l. Trong mt ngy
c bit, c tt c 3800 ngi 200 i l n mua v, mi ngi c mua mt v.
1. Chng minh rng c t nht 6 i l c cng s ngi n mua v trong ngy hm .
2. trn khng cn ng nu thay 6 bi s 7.
Li gii.
1. Gi s1 ; s2 ; : : : ; s200 l s ngi n i l th nht, i l th hai,: : :, i l
th 200 mua v tu ha trong ngy . Khng mt tnh tng qut, ta c th gi s
s1  s2  : : :  s200 .
t S D s1 C s2 C    C s200 th S D 3800. By gi ta s tm hiu S nh nht bao nhiu
khi iu kin bi ton khng tha, tc khng tn ti dy si ; si C1 ; si C2 ; : : : ; si C5 no m
si D si C1 D : : : D si C5 . R rng S nh nht khi ta ly c cng nhiu s nh, mi s
nh khng ly qu 5. Do S nh nht khi
s1 D s2 D : : : D s5 D 1;
s6 D s7 D : : : D s10 D 2;
:::
s196 D s197 D : : : D s200 D 40:
109

Tp ch Epsilon, S 06, 12/2015


Nhng khi th tng S s t c
1 C 1 C    C 1 C 2 C 2 C    C 2 C    C 40 C 40 C    C 40 D 4100:

5 s

5 s

5 s

Do S > 3800, iu ny mu thun. iu chng t phi tn ti 6 s hng lin tip


trong dy bng nhau. Hay c 6 i l c cng s ngi n mua v trong ngy hm .
2. Chng ta xy dng mt v d tha mn cng bng thut ton trn. Vi mi 1  i  198 th


i 1
si D
C 1:
6
Khi s1 C s2 C    C s198 D 3366. Khi chn s119 D s200 D 220. Khi S D 3800
v r rng khng c 7 phn t lin tip no bng nhau, tc khng c 7 ca hng no c
cng s ngi n mua v. Bi ton c gii quyt hon ton.
Cu 2 (Iran 1997). Cho hai s nguyn dng m; k. Chng minh rng tn ti duy nht cc s
nguyn dng ak > ak 1 > : : : > a1  0 sao cho
!
!
!
ak
ak 1
a1
mD
C
C  C
k
k 1
1
!
!
!
ak
ak 1
a1
Tng
C
C  C
c gi l khai trin Macaulay ca m ng vi d .
k
k 1
1
V d khai trin Macaulay ca 17 ng vi 3 l
!
!
!
5
4
1
C
C
D 10 C 6 C 1 D 17;
3
2
1
trong khi khai trin Macaulay ca 16 ng vi 3 l
!
!
!
5
4
0
C
C
D 10 C 6 C 0 D 16:
3
2
1
Li gii.
1. u tin ta chng minh s duy nht. Gi s m c biu din bi hai dy
ak ; : : : ; a1 v bk ; : : : ; b1 . Khi trong hai dy a1 ; : : : ; ak v fb1 ; : : : ; bk g (theo th t)
phi c t nht hai phn t khc nhau. Ta gi v tr u tin m chng khc nhau, khng
mt tnh tng qut l k, tc ak bk . Gi s ak > bk m khng lm mt tnh tng qut
ca bi. V b1 < b2 < : : : < bk 1 < bk l dy s nguyn nn
bk
V

 bk

1; bk

 bk

1  bk

2; : : : ; b1 < bk

!
!
!
bk
bk 1
b1
mD
C
C  C
k
k 1
1
110

k C 1:

Tp ch Epsilon, S 06, 12/2015


nn
!
!
bk
bk 1 1
bk
m
C
C  C
k
k 1
Theo tnh cht nh thc
!
!
bk
bk 1 1
bk
C
C  C
k
k 1
!
bk C 1
)m<
:
k

!
kC1
:
1

!
!
kC1
bk C 1
<
1
kC1

V bk < ak nn bk C 1  ak . Do
!
!
bk C 1
ak

:
k
k
Dn n
!
!
!
!
!
a1
ak 1
ak
ak
bk C 1
m
C  C
C
<

m<
1
k 1
k
k
k
v l. Vy iu phn chng l sai.
2. chng minh s tn ti, ta s dng thut ton tham lam nh sau: u tin ta tm s ak
ln nht sao cho
!
ak
 m:
k
!
ak
Sau li p dng thut ton trn, thay v cho m v k, th by gi cho m
v k 1.
k
C tip tc quy trnh ny ta dng c dy tha mn. Dy fa1 ; a2 ; : : : ; ak g l dy tng dn
v, theo cch xy dng nn
!
!
!
ak C 1
ak
ak
m<
)m
<
k
k
k 1
!
!
!
ak C 1
ak
ak
do tnh cht nh thc
D
C
. Nhng li theo cch dng th
k
k
k 1
!
ak 1
<m
k 1

ak
k

!
!
ak 1
ak
)
<
) ak
k 1
k 1

Bi ton c chng minh hon ton.

111

< ak :

Tp ch Epsilon, S 06, 12/2015


Cu 3 (Russian 2005). Trong mt bng vung kch thc 2  n (n l s nguyn dng khng
nh hn 2) ngi ta in vo mi vung mt s thc dng sao cho tng ca hai s trn mi
ct lun bng 1. Chng minh rng ta c th chn ra trn mi ct mt s tng ca cc s c
nC1
.
chn trn mi dng khng vt qu
4
nC1
, tc l yu
4
cu tng cc s trn mi dng cng nh cng tt. T ta ngh n vic chn trn mi
ct mt s nh nht th kh nng thnh cng s cao. Tuy nhin nu tt c cc s nh hn
(trn mi ct) li nm trn cng mt dng th sao? Khi thut ton tham lam ny s tht
bi, tht vy vi minh ha

Li gii.

 Do bi yu cu tng cc s trn mi dng khng vt qu

0.4
0.6

0.4
0.6

0.4
0.6

...
...

0.4
0.6

th nu ta chn trn mi ct mt s nh nht, khi ta ch chn ton s 0.4 dng u


nC1
tin. Nhng khi th tng cc s dng u tin l 0:4n, vt khi
.
4
 Ta cn ci tin thut ton ny mt cht. Tc l chng ta s m bo cho c hai dng
cng tha iu kin mt lc. Tc l chng ta mong mun bng cch no phi chn
c ng thicc s nh nht c th c dng u tin v cc s nh nht c th c
dng th hai. Bng cch thay i th t cc ct, ta c th sp xp cc s dng th nht
theo th t khng gim, th v tng ca hai s trn mi ct bng 1, nn dn n cc s
dng th hai s theo th t khng tng. Gi a1 ; a2 ; : : : ; an l cc s dng th nht m
a1  a2  : : :  an .
a1
1 a1

a2
1 a2

a3
1 a3

an
1 an

...
...

 Vi sp th t ny, th dng th nht ta s chn cc s t tri qua phi, cn dng th hai


ta s chn cc s t phi qua tri. By gi ta chn i l ch s ln nht sao cho
a1 C a2 C    C ai 

nC1
4

(tc l cc s c chn trn dng u tin tha mn). By gi ta ch cn kim tra


nC1
 .1
4
D .n

ai C1 / C .1
i/

ai C2 / C    C .1

.ai C1 C ai C2 C    C an /

hay
ai C1 C ai C2 C    C an 

3n

1
4

V a1  a2  : : : ai C1 nn
a1 C a2 C    C ai C1
 ai C1
i C1
112

i:

an /

Tp ch Epsilon, S 06, 12/2015


v do ai C1  ai C2  : : :  an nn
ai C1 C ai C2 C    C an
 ai C1 :
n i
Kt hp hai bt ng thc trn ta c
ai C1 C ai C2 C    C an
a1 C a2 C    C ai C1

n i
i C1
hay
a1 C a2 C    C ai C1
i/
> .n
i C1

ai C1 C ai C2 C    C an  .n

i/

nC1
4

i C1

(bt ng thc cui c c do i l ch s ln nht tha a1 C a2 C    C ai 

nC1
).
4

Cui cng ta ch cn chng minh


n i nC1
3n 1


i C1
4
4

i ) .n

1/2  4i.n

1/:

iu ny c c do bt ng thc Cauchy
4i.n

1/  4 

.i C n

i
4

1/2

D .n

1/2 :

n y bi ton c chng minh hon ton.


Cu 4 (IMO 2003). Cho A l mt tp con cha 101 phn t ca tp S D f1; 2; : : : ; 1000000g.
Chng minh rng tn ti cc phn t t1 ; t2 ; : : : ; t100 trong S sao cho cc tp
Aj D fx C tj jx 2 Ag;

j D 1; 2; : : : ; 100

ri nhau i mt.
Li gii.
 Ta mong mun tm mt thut ton xy dng t1 ; t2 ; : : : ; t100 . Gi s trong tay ta
c t1 , vy th t2 phi c chn sao cho (theo gi thit)
x C t1 y C t2 ; 8x; y 2 A ) t2 t1 C x

y; 8x; y 2 A:

m bo dy fti g phn bit th ta s xy dng dy fti g tng dn. T ta cn t2


t1 C jx yj; 8x; y 2 A th s m bo c tnh tng dn. Nhng khi chn c t2 th
cng phi chn t3 ; : : : ; t100 . Do chn t2 phi va m bo " xa", tc khc tt c cc
gi tr trc , nh phn tch trn, va m bo " gn", c th xy dng tip
cho t3 ; : : : ; t100 . T y tng xy dng t2 c hnh thnh:
Trn trc s ta biu din cc phn t ca S theo th t tng dn.
Chn ngay t1 D 1 2 S , v ta t mu s t1 v tt c cc s dng: ft1 C jx yj D
1 C jx yjj8x; y 2 Ag trn trc s (cc gi tr hiu jx yj c th trng nhau
vi cc cp
! s .x; y/ khc nhau trong A, nn bc ny ta nh du khng qu
101
1C
D 5051 s, k c s t1 D 1).
2
113

Tp ch Epsilon, S 06, 12/2015


Sau bc ny, ta chn t2 l s nh nht trn trc s m cha b t mu. Tip tc
ta li t mu s t2 v cc s trn trc s dng: ft2 C jx yjjx; y 2 Ag. bc ny
c th mt vi s c t mu bc trn. Khi trn trc s, sau bc ny, c
ti a 2  5051 s trong S c t mu. Vi cch xy dng ny th t2 > t1 .
C tip tc thut ton trn, sau khi xy dng c cc s t1 ; : : : ; t99 th chng ta
t mu ti a 500049 phn t trong S trn trc s. V 500049 < 1000000 nn ta tip
tc xy dng c s t100 theo thut ton trn. Vy cc s t1 < t2 < : : : < t100 c
xy dng.
 By gi ta kim tra li, vi cch xy dng t1 ; t2 ; : : : ; t100 nh thut ton trn, cc tp
Aj ; Ak s ri nhau i mt vi mi 1  j < k  100. Tht vy, gi s ngc li, tc tn
ti x C tj D y C tk vi x; y 2 A no . Do cch xy dng trn m bo tk > tj . Do
x > y. iu ny dn n
tk D tj C x

y D tj C jx

yj;

v l v tk c chn bc th k th phi khc tt c cc s c t mu trn trc s:


fti C jx yj W 8i D 1; 2; : : : ; k 1g. Vy Ai \ Aj D ;. Bi ton c chng minh hon
ton.
Cu 5. Cho A l tp hp cc s nguyn dng tha mn: vi mi phn t x; y thuc A th
jx

yj 

xy
:
30

Hi A cha nhiu nht bao nhiu phn t?


Li gii.
1. Ta nhn thy x; y khng th l s qu ln. Bt u vi tp A D f1g, p dng
thut ton tham lam, mi ln chng ta s th tm phn t nh nht tip theo c th nm
trong A. Qu trnh ny ta thu c A D f1; 2; 3; 4; 5; 6; 8; 11; 18; 45g v sau qu trnh ny
th khng th thm c phn t no vo A. T d on A c nhiu nht 10 phn t.
2. t A D fa1 ; a2 ; : : : ; an g. Ta chng minh n  10. Khng mt tnh tng qut, gi s
1  a1 < a2     < an . Khi
ai C1

ai 

ai C1 ai
1
)
30
ai

1
ai C1

Tng tt c cc nh gi trn vi i D 5; 6; : : : ; n
1
a5

1
; 81  i  n
30

1 ta c

1
n 5
1
n 5

)

:
an
30
a5
30

V ai  i; 8i D 5; : : : ; n nn t nh gi trn ta c
5  a5 <

30
n

T bi ton c chng minh hon ton.

114

) n  10:

1:

Tp ch Epsilon, S 06, 12/2015


Cu 6 (IMO 2014). Mt tp hp cc ng thng trong mt phng gi l "tt" nu khng c
hai ng thng no trong chng song song v khng c ba ng thng no trong chng ng
quy. Tp hp cc ng thng "tt" chia mt phng thnh cc min, vi mt s min c din
tch hu hn gi l min hu hn. p
Chng minh rng vi n ln, th vi bt k tp hp n ng
thng "tt" ta lun c th t mu n ng thng mu xanh khng c min hu hn no c
ton b bin l mu xanh.
Li gii. (Da theo li gii ca GS Nguyn Tin Dng) Ta cng th lm theo kiu thut ton,
tc l tm thut ton t mu xanh cc ng. Ti sao li ra con s cn bc hai, th cht xu na s
r. Thut ton n gin nh sau:
 u tin t 2 ng ty mu xanh (hin nhin l ch c 2 thi th cha th vy ton b
min no).
 Tip theo l chn ng t: nu chn c thm 1 ng t xanh (m khng phm
lut ca bi) th t, cn n khi khng cn ng no na th dng.
Gi l thut ton dng li sau khi t c k ng. By gi phi chng minh l n  k 2 . Hay l
ta chng minh ngc li: nu n > k 2 th t tip c.
Nu n > k 2 th s ng cn li cha t ln hn k 2 k D k.k 1/, tc l ln hn 2 ln s im
nt xanh (im nt xanh = im giao nhau ca 2 ng xanh). Gi 1 ng (trong s cc ng
cn li ) l ng cm nu nh m t thm ng mu xanh th to min b chn vi bin
ton mu xanh. By gi ch cn chng minh l s ng b cm khng vt qu k.k 1/, khi
s dn n c t nht 1 ng khng b cm, c th t xanh. chng minh iu , ta s tm
cch lp mt mi quan h gia cc ng b cm vi cc nt xanh, sao cho tt c cc ng cm
u ng vi t nht 1 nt, v ngc li th mi nt ng vi khng qu 2 ng cm. Nu c quan
h nh vy th s ng cm khng vt qu 2 ln s nt). Quan h nh th no?
Mt quan h hin nhin l: mt ng cm th tc l to ra t nht 1 min cm (min c 1 cnh
bin trn ng , cc cnh cn li u xanh). Ly min cm , v ly 2 ci nt xanh l 2 nh
min cm m k st ng cm. 2 nt c th trng nhau nu min cm l tam gic.
phn bit, th thay v t quan h vi nt , ta s t quan h vi on cm: gm nt v cnh
xanh ca min cm i t nt chm vo ng cm. Nh vy, mi ng cm ng vi t
nht 2 on cm.
Ngc li, mi nt cho nhiu nht 4 on cm, v t mi nt ch c 4 na ng thng xanh, v
trn mi na ng thng c nhiu nht 1 on cm xut pht t nt (khng th c 2 on
cm ln nhau cng t 1 nt xanh). Nh vy, y l quan h m theo 1 chiu th  2, theo
chiu ngc li th  4, v do s ng cm  2 ln s nt xanh.
Cu 7 (IMO 1983). Chng minh rng c th chn 2048 s nguyn dng phn bit, tt c cc
s u nh hn hoc bng 100000; sao cho khng c ba s hng bt k no ca tp to thnh
ba phn t lin tip mt cp s cng.
Li gii.
 y c mt trc gic l: s 100.000 dng nh khng thch hp v qu ln. Do
lm chng ta suy ngh gii quyt bi ton vi s nh ri m rng n. Chng ta chc
chn nn bt u vi vic xy dng mt dy nh bng thut ton tham lam.
 Bt u vi dy 1; 2. Khi
115

Tp ch Epsilon, S 06, 12/2015


3 khng th thm vo v to thnh cp s cng 1 ! 2 ! 3;
Chng ta thm 4, 5 vo dy.
6 li khng th thm vo dy c, v to thnh cp s cng 2 ! 4 ! 6.
7 khng thm thm vo dy c v to thnh cp s cng 3 ! 5 ! 7.
8 khng th thm vo dy c v to thnh cp s cng 2 ! 5 ! 8.
9 khng th thm vo dy c v to thnh cp s cng 1 ! 5 ! 9.
Chng ta thm 10, 11, li khng th thm 12. Thm c 13, 14, v c tip tc nh
vy ta thm vo c s tip theo gn nht l 28,29.
1. Ta dng li phn tch khi c mt s khong trng xut hin: 2 ! 4; 5 ! 10; 14 !
28. Tc l cc s gp 2 ln xut hin.
2. Ngoi ra cn c 4 ! 10 ! 28, iu ny vit li 31 C 1 ! 32 C 1 ! 33 C 1. T
ta nhn thy s 3 ny ng vai tr quan trng, c bit l cc ly tha ca 3.
iu ny gi ta n vic xy dng dy s theo c s 3, nhng tr i 1 t mi s.
Do dy ca ta l
S D 1 C f03 ; 13 ; 103 ; 113 ; 1003 ; 1013 ; 1103 ; 1113 ; 10003 ; : : : ; 111111111113 g
gm tt c cc s trong c s 3 ch gm hai ch s 0 v 1.
 Ta c jSj D 211 D 2048 s.
 Phn t ln nht trong S l 1 C 111111111113 < 100:000;
 Gi x; y; z l ba phn t phn bit ty trong S , gi s x C y D 2z. Do 2z ch gm hai
ch s 0 v 2. M ch c s phn tch duy nht 0 C 0 D 0; 1 C 1 D 2 nn x; y phi c ch
s 0 v 1 trng nhau mi v tr, tc x D y, v l. Tc ba phn t ty ca S khng th l ba
phn t lin tip ca mt cp s cng.

Cu 8. Chng minh rng vi mi s nguyn dng n, lun c th biu din di dng duy nht
thnh tng ca mt s ly tha phn bit ca 2.
Phn tch: u tin ta chng minh s biu din ny c th s dng thut ton tham lam. Ta chn
ly tha ln nht, gi l 2k , sao cho 2k  n. Nu n D 2k th bi ton kt thc. Ngc li, ta p
dng thut ton cho s n 2k . Chc chn l cc ly tha ca 2 s phn bit trong mi bc
chn, v theo cch xy dng cho 2k n < 2kC1 , dn n n 2k < 2k . Do bc tip theo ta
chn c 2k 1 : 2k 1  n 2k th 2k 1 < 2k . Tuy nhin khi ta vit li gii s vit di dng
quy np.
Li gii. Vi n D 1, khi ta c 1 D 20 .
Gi s bi ton ng cho vi mi s n m 1  n < 2k , vi k  1. Ta s chng minh bi ton
ng cho vi mi n m 1  n < 2kC1 . Nh ta bit rng trn khong 1  n < 2k , th biu din
ca n s c s hng ln nht l 2k 1 . Cng thm vo mi s biu din cho s 2k , ta s c
s biu din cho mi s n m
1 C 2k  n < 2k C 2k D 2kC1 :
116

Tp ch Epsilon, S 06, 12/2015


Ngoi ra n D 2k bn thn n l mt biu din di ly tha ca 2. Do , bi ton ng cho
vi mi 1  n < 2kC1 . chng minh s duy nht, nh theo hng quy np, ta phi chng t
vi mi 2k  n < 2kC1 , phi c 2k trong biu din ca n. Tht vy, gi s ngc li, th trong
mi biu din ca n, th tng ln nht l
n  20 C 21 C 22 C    C 2k

D 2k

1 < n;

mu thun.
Cu 9. Cho A1 ; A2 ; : : : ; An l cc tp con phn
 bit ca f1; 2; : : : ; ng v jAi j D 3; 8i D
2n
1; 2; : : : ; n. Chng minh rng ta c th t mu
phn t ca f1; 2; : : : ; ng sao cho mi tp
3
Ai u c t nht mt phn t c t mu.
Phn tch: Ta s thc hin t mu cc phn t trong f1; 2; : : : ; ng theo thut ton tham lam nh
sau: Mi bc, ta s t mu phn t x thuc cng nhiu tp cng tt. Gi s x thuc vo cc tp
A1 ; A2 ; : : : ; Ak . n bc th hai ta loi b cc phn t thuc vo cc tp A1 ; A2 ; : : : ; Ak , ri
ta li tip tc t mu phn t y thuc cng nhiu tp cn li cng tt, c tip tc nh vy ta s
c iu phi chng minh.
Li gii. Gi A l tp hp cc tp trong s cc tp A1 ; A2 ; : : : ; An m mi tp u cha c phn
t no c t mu sau khi thc hin k ln thut ton tham lam trn. Ta lu rng sau khi
thc hin k bc, th tt c cc tp hp trong A u "ri nhau". Khi r rng thut ton s dng
khi thc hin tip k C jAj bc (v mi tp hp trong A phi t mu mt phn t, do cc tp ny
ri nhau). Ta ch rng:
n
 k  bi v mi bc thc hin thut ton, s tp hp gim i t nht hai ln v trong
2
k ln thc hin thut ton u tin, ta lun tp trung vo t mu cc phn t thuc vo t
nht hai tp tr ln.
 jAj 

bi v sau khi thc hin thut ton k ln, ta t mu k phn t, tc l


3
loi khi t nht l k phn t ca f1; 2; : : : ; ng, cn li nhiu nht n k phn t li c
chia thnh cc tp ri nhau kch thc 3:

Do
k C jAj  k C

k
3

n 2k
n n
2n
2n
C
 C D
)k
:
3
3
3
3
3
3



2n
V k nguyn dng, chng t thut ton s kt thc nu ta thc hin nhiu nht l
ln. Bi
3
ton c chng minh.
Cu 10 (China TST 2015). Cho X l mt tp khc rng v A1 ; A2 ; : : : ; An l n tp con ca X
sao cho
1. jAi j  3; 8i D 1; 2; : : : ; n;
2. Bt k mt phn t no ca X cng nm trong t nht 4 tp trong s A1 ; A2 ; : : : ; An .
 
3n
Chng minh rng c th chn
tp hp trong s cc tp A1 ; A2 ; : : : ; An m hp ca chng
7
bng X.
117

Tp ch Epsilon, S 06, 12/2015


Li gii. Kt lun bi ton yu cu chn c mt s tp hp, hp li bng X , do ta s
1. Chn tp u tin c 3 phn t, gi s A1 ; jA1 j D 3. Sau , ta chn tip tp A2 m
jA2 j D 3 v A2 \ A1 D ;. Sau khi chn c tp A2 , ta chn tip tp A3 m jA3 j D 3 v
A3 \ A1 D ;; A3 \ A2 D ;, c tip tc nh vy n khi khng th chn thm c tp no
na vo trong h. Trong tt c cch cch la chn cc tp hp trong A1 ; A2 ; : : : ; An , ta chn
ra h tp hp S3 cc i, gi s S3 D fA1 ; A2 ; : : : ; Ai g (i  n) (tc l h S3 cha nhiu
tp hp nht c th c) m jAt j D 3; 8t D 1; 2; : : : ; i v Ar \ As D ;; 81  r < s  i
(iu ny c ngha, mi ln b sung mt tp hp vo S3 th tp X3 c s lng phn t
tng ln 3).

t X3 D

Ar . Do tnh ti i ca tp S3 , nn vi bt k tp hp Aj (j > i) th

Ar 2S3

jAj \ .XnX3 /j  2
v nu khng th ta tip tc b sung Aj vo tp S3 , mu thun vi tnh ti i ca S3 . V
khi
jX3 j D 3i:
2. By gi ta tip tc chn h S2 cc i cha cc tp Aj cn li trong s Ai C1 ; : : : ; An ,
sao cho mi ln thm mt tp hp vo h S2 , th s lng phn t trong hp ca chng
tng ln 2. Khng mt tnh tng qut, gi s
S2 D fAi C1 ; Ai C2 ; : : : ; Aj g
118

Tp ch Epsilon, S 06, 12/2015

t
X2 D

Ar \ .XnX3 /

Ar 2S2

th theo cch xc nh ca S2 ta c jX2 j D 2j v theo tnh ti i ca tp S2 th


jAt \ .Xn.X2 [ X3 //j  1; 8t D j C 1; : : : ; n:
3. By gi ta tip tc chn h S1 cha cc tp As cn li trong s
Aj C1 ; Aj C2 : : : ; An ;
sao cho mi ln thm mt tp hp vo h S1 , th s lng phn t trong hp ca
chng tng ln 1 v d nhin cc tp hp trong h S1 cha ht tt c cc phn t ca
X n.X3 [ X2 / D X1 . Khng mt tnh tng qut, gi s
S1 D fAj C1 ; Ai C2 ; : : : ; Ak g:
4. Khi jXj D jX1 j C jX2 j C jX3 j D 3i C 2j C k, X D X1 [ X2 [ X3 v
jS3 j C jS2 j C jS1 j D i C j C k D m:
Ta cn chng minh m 

3n
.
7

 V mi phn t trong X1 nm trong t nht 4 tp hp, nhng do jAr \ X1 j  1; 8r D


j C 1; : : : ; n nn
n  i C j C 4k: .1/
Mi phn t trong X1 [X2 xut hin t nht trong 4 tp hp, nh do jAr j\.X1 [X2 / 
2; 8r D i C 1; : : : ; n nn
niC

4.2j C k/
D i C 4j C 2k:
2

.2/

Mi phn t trong X xut hin t nht trong 4 tp hp, v jAr \ X j  3; 8r D


1; 2; : : : ; n, do
4.3i C 2j C k/
n
: .3/
3
119

Tp ch Epsilon, S 06, 12/2015


 Ly 20  .1/ C 12  .2/ C 27  .3/ ta c
59n  140.i C j C k/ D 140m ) m 

59n
3n
<
:
140
7

Bi ton c chng minh hon ton.



1
Cu 11 (IMO 2014). ng xu c gi l "may mn" nu gi tr ca n l n 2 ZC . Mt b
n
1
su tp cc ng xu may mn c tng gi tr khng qu 99 C . Chng minh rng c th chia
2
cc ng xu ny vo 100 nhm sao cho mi nhm c gi tr khng qu 1.
Li gii. Ta chng minh bi ton tng qut hn: "Gi s tng gi tr cc ng xu khng qu
1
v th lun c th chia vo N nhm sao cho mi nhm c tng gi tr khng qu 1."
N
2
Gi s b su tp c cc ng xu mnh gi l
1 1
1
1; I I : : : ;
2 3
m

(c th c nhiu ng xu cng mnh gi):

Bc 1. Ghp cc ng xu cng mnh gi: Ta ghp cc ng xu c cng mnh gi li thnh mt


ng xu mi (ng xu mi vn l "may mn") , c ghp nh vy cho ti khi no khng th ghp
th dng li.
Chng hn lc u trong tay ta c tp cc ng xu may mn sau


1 1 1 1 1 1 1 1 1 1 1
S D 1I I I I I I I I I I I :
2 2 2 3 3 3 3 4 4 6 6
1
1
1
1
 Ta ghp hai ng xu thnh mt ng xu , 2 ng xu thnh mt ng xu . Khi ta
6
3
4
2
1
1
c nm ng xu , bn ng xu , v mt ng xu 1.
3
2
 Tip tc ghp nm ng xu

1
1
1
c mt ng xu 1, hai ng xu v bn ng xu ghp
3
3
2

li thnh hai ng xu 1.
 Cui cng c

1 1
S D I I 1I 1I 1
3 3
0

khng ghp tip c, tng khng i so vi S .


Kt thc qu trnh ghp cc ng xu ta c:
 C p ng xu mnh gi 1.
 Nu k chn th cn li nhiu nht 1 ng xu mnh gi
 Nu k l th cn li nhiu nht k

1
.k > 1/.
k

1 ng xu mnh gi
120

1
.k > 1/.
k

Tp ch Epsilon, S 06, 12/2015


Khi p ng xu mnh gi 1 ta chia v p nhm v cn cc ng tin mnh gi khc 1 c tng
1
khng qu N
p ta chia vo N p nhm nh sau (vi ch : vi mi k 2 ZC th ng
2
1
1
tin mnh gi
cn nhiu nht l 1 ng v ng tin mnh gi
cn nhiu nht l
2k
2k 1
.2k 1/ 1 ng, tr trng hp k D 1).
1
Bc 2. Chia cc ng tin mnh gi ln hn hoc bng
vo N p nhm. Vi cc
2.N p/
1
1
ng tin mnh gi I
, k D 1I 2I : : : I N p ta chia v nhm Gk .k D 1I 2I : : : I N p/
2k 2k 1
th tng s tin nhiu nht ca nhm Gk l
.2k

2/:

1
2k

Nu vn cn cc ng tin mnh gi nh hn

1
< 1:
2k

1
2.N

Bc 3. Ta thy rng do tng s tin chia vo N

p/

th tin hnh bc nh sau:

p nhm khng ln hn N

ti mt nhm c tng s tin nh hn hoc bng




1
1
 N p
D1
N p
2
 Ta ly mt ng xu c mnh gi nh hn

1
2.N

1
2.N

p/

p/

1
2

p, nn tn

b vo nhm v nhm c tng

s tin khng vt qu 1.
 V s ng tin l hu hn nn sau mt s ln th kt thc.

Ta kt thc chuyn bng mt v d, cho thy thut ton tham lam khng cho ta c mt ti
u ton cc, tuy nhin gia chng vn xp x c vi nhau
Cu 12 (IMO Shortlist 2014). Cho dy s thc x1 ; x2 ; : : : ; xn , chng ta gi "gi" ca dy s
cho l i lng
max jx1 C x2 C : : : C xi j:
1i n

Cho trc s thc n, Dave v George mun sp xp thnh 1 dy c gi nh nht. Chm ch


hn nn Dave kim tra tt c cc cch c th v tm ra gi G nh nht. Theo mt cch khc,
George chn x1 sao cho jx1 j nh nht, trong s cc s cn li anh ta chn x2 sao cho jx1 C x2 j
nh nht, c tip tc nh vy n bc th i anh ta chn xi trong s cc s cn li sao cho
jx1 C x2 C : : : C xi j nh nht, trong mi bc nu c nhng s hng bng nhau th anh ta chn
bt k. Cui cng anh ta nhn c dy gi D. Tm hng s c nh nht c th sao cho vi mi
s nguyn dng n ta lun c:
G  cD:
Li gii. Xt dy s gm cc s thc: 1; 2;
nh sau
Dave W

1; 2;

2;

1;

hoc 1;
121

2. Khi ta c sp xp ca Dave v George


2; 2;

1: Khi W D D 1

Tp ch Epsilon, S 06, 12/2015


George W 1;

1; 2;

hoc 1;

1; 2;

2: Khi W G D 2:

s
T ta thu c: c  2. Ta s chng minh c D 2 l gi tr cn tm, tc G  2D. Gi
x1 ; x2 ; : : : ; xn l dy s ban u, d1 ; d2 ; : : : ; dn v g1 ; g2 ; : : : ; gn l dy m Dave v George
thu c. t
M D max jxi j; S D jx1 C x2 C : : : C xn j:
1i n

Nhn xt: D  S (hin nhin do cch xc nh D). Gi s jdi j D M ta c:


M D jdi j D j.d1 C d2 C : : : C di / .d1 C d2 C : : : C di
 jd1 C : : : C di j C jd1 C : : : C di 1 j  2D:

1 /j

Do chng minh G  2D ta ch cn chng minh


G  maxfM; Sg:
t N D maxfM; Sg ta s chng minh G  N . t hi D g1 C : : : C gi . Khi
G  N , jhi j  N; 8i D 1; n:
Ta s chng minh: jhi j  N bng quy nap. Vi i D 1 ta c
jh1 j D jg1 j  M  N:
Gi s jhi

1j

 N ta s chng minh jhi j  N . Ta c


jhi j D j.g1 C : : : C gi

1/

C gi j D jhi

C gi j:

 Nu trong cc s gi ; gi C1 ; : : : ; gn c 2 s khc du nhau, gi s j  i l ch s m


hi 1 :gj  0. Khi

jhi 1 C gi j  hi 1 C gj  max jhi 1 j; gj  N ) jhi j  N:


 Nu cc s gi ; gi C1 ; : : : ; gn cng du, khng mt tng qut gi s chng u l cc s
dng. Khi , ta c
hi

 hi  hi C1  : : :  hn ) jhi j  maxfjhi
) jhi j  N

1 j;

jhn jg  N

vi 8i D 1; n tc l G  N D maxfM; Sg  2D.
Vy c D 2 l gi tr cn tm.

Sau y l mt s bi ton bi c c th tham kho, rn luyn thm.


Cu 13 (IMO Shortlist 2014). Cho n l s nguyn dng. Tm s nguyn dng k nh nht
tha mn: cho trc cc s thc a1 ; a2 ; : : : ; ad c
a1 C a2 C    C ad D n v 0  ai  1; 8i D 1; 2; : : : ; d
th lun c th chia d s ny vo k nhm (mt s nhm c th l tp rng) sao cho tng cc phn
t trong mi nhm khng vt qu 1.
122

Tp ch Epsilon, S 06, 12/2015


Cu 14 (USA TST 2003). Vi cp s nguyn dng a v b vi 0 < a < b < 1000, tp
S  f1; 2; : : : ; 2003g c gi l "tp b qua" cho .a; b/ nu vi mi cp phn t s1 ; s2 2 S th
js1 s2 j 62 fa; bg. t f .a; b/ l tp c kch thc ln nht cho cp .a; b/. Xc nh gi tr ln
nht v nh nht ca hm f .x; y/, vi 0 < x < y < 1000.
Cu 15. Chng minh rng vi mi s hu t dng x, lun c th tm c cc s nguyn dng
phn bit a1 ; a2 ; : : : ; an sao cho
xD

1
1
1
C
C  C :
a1
a2
an

Cu 16 (IMO Shortlist 2001). Mt b ba phn t cc s nguyn khng m .x; y; z/ vi


x < y < z c gi l "p" nu fz y; y xg D fa; bg vi 0 < a < b l cc s nguyn dng
cho trc. Chng minh rng tp hp N c th vit thnh hp ri nhau ca cc b ba "p".
Cu 17. Cho dy .an / gm cc s t nhin tha mn: khng tn ti b ch s .i; j; k/ m
i < j < k th ai < aj < ak . Chng minh rng c th phn hoch dy .an / thnh hai dy con
tng.

Ti liu tham kho


[1] Algorithms, Cody Johnson, Mathematical Reflection volume 4, 2015.
[2] thi hc sinh gii cc nc trn trang Mathlinks.ro.

123

Tp ch Epsilon, S 06, 12/2015

124

NH BERTRAND
Lu B Thng
(Khoa Ton Tin, i hc S Phm H Ni)
Chng ta bit rng dy cc s nguyn t 2; 3; 5; 7; ::: l v hn. kim tra khong cch gia hai
s nguyn t lin tip l khng b chn, tc l vi mt s nguyn dng k ty , lun tn ti hai
s nguyn t k nhau c hiu vt qu k. chng minh iu ny, ta ch cn ch ra tn ti k s
t nhin lin tip khng l s nguyn t. Tht vy, nu ta t N D 2:3:5::::p l tch tt c cc s
nguyn t khng vt qu k C 2 th khng s no trong dy gm k s nguyn lin tip
N C 2; N C 3; :::; N C .k C 1/
l s nguyn t. Nhng vn c chn trn cho khong cch trong dy cc s nguyn t. Nm 1845,
Bertrand a ra gi thuyt, thng gi l nh Bertrand (Bertrands postulate):
Cho s nguyn dng n  1, lun c s nguyn t p vi n < p  2n.
Bertrand chng minh gi thuyt trn cho n < 3000000. Nm 1950, Tchebychev l ngi u
tin a ra mt chng minh bng gii tch cho nh trn. Nm 1932, Paul Erdos a ra mt
chng minh p cho nh Bertrand ch dng cc kin thc Ton s cp, khi ng mi 19 tui.
T tng
! chnh ca Erdos trong vic chng minh nh Bertrand chnh l vic c lng gi
2n
tr
. Trong bi vit ny, chng ti s gii thiu n cc bn hc sinh, sinh vin v cc bn
n
tr yu Ton chng minh ca Erdos cng mt vi s kt qu gn y v vn ny.
Ta thy rng, vi n  2 th
!
2n
4n
:
(0.1)

2n
n
!
!
!
!
!
2n
2n
2n
2n
2n
Tht vy,
l s hng ln nht trong 2n s hng
C
;
; :::;
ca
n
0
2n
1
2n 1
tng
!
2n
X
2n
D .1 C 1/2n D 4n :
i
i D0
Erdo!
s chng minh nu khng c s nguyn t p sao cho n < p  2n th chng ta s suy ra
2n
4n
<
tr mt s gi tr n nh. iu ny chng minh c nh Bertrand cho s n
n
2n
ln. i vi gi tr n nh, Erdos s kim tra trc tip.
chng minh cc tng trn, chng ta cn mt s b sau:
B 2. nh Bertrand ng vi n < 4000.
Chng minh. chng minh iu ny, ta khng cn kim tra tt c 4000 trng hp m ch cn
kim tra,
2; 3; 5; 7; 13; 23; 43; 83; 163; 317; 631; 1259; 2503; 4001;
125

Tp ch Epsilon, S 06, 12/2015


l dy cc s nguyn t m mi s ng sau khng vt qu 2 ln s ng ngay ng trc n.
Do , mi khong fm W n < m  2ng vi n < 4000 u cha ng mt trong 14 s nguyn t
ca dy trn.
B 3 (nh l Legendre). Cho s nguyn t p v s t nhin n. Ta k hiu vp .n/ l s m
ln nht ca p sao cho p vp .n/ j n, khi
vp .n/ D

X n
i :
p
i 1

Chng minh. Trc


 ht ta thy tng trn thc cht ch gm
 hu hn s hng v vi i ln th
n
n
D 0: Mt khc, trong tch n c ng
p i > n nn
tha s l bi ca p. Do ta vit
i
p
p
n  

n
n D p p
q1 trong .q1 ; p/ D 1:
p

Tng t,
2

 
n
Dp
p

6 7
6 p 7
72
6
7
6
n 3
6 p 7
5
4

6 p 7
4
5q2 trong .q2 ; p/ D 1:
p

2 n 3
 

n
6 p 7
D kim tra rng 4
nn
5D
p
p2
n  
 
4
5
n
n
2
p
Dp
q2 :
p
p2
2

Suy ra
n  
n
4 5C4
5
n
p2
q1 q2 trong .q1 ; p/ D 1:
n D p p
p2
2

3 2

Tip tc qu trnh ny ta thu c


n
n
n
5C4
5C4
5C
2
3
p
p
p
n D p
A trong .A; p/ D 1:
2

3 2

3 2

Vy s m ca s nguyn t p trong phn tch tiu chun ca n l


X n   n   n   n 
p .n/ D
D
C
C
C  :
i
2
3
p
p
p
p
i >1

126

Tp ch Epsilon, S 06, 12/2015


!
2n
2n
B 4. Nu p j
th p vp .. n //  2n:
n
Chng minh. t r.p/ l s t nhin tha mn p r.p/  2n  p r.p/C1 . Ta c
!
2n
vp .
/ D vp ..2n// 2vp .n/
n
r.p/

r.p/

X 2n
D
i
p
i D1

X n
i
p
i D1

r.p/

X 2n
. i
p
i D1

D


n
/
pi

r.p/;

2n

v do p vp .. n //  p r.p/  2n:
!
2n
T B trn ta thy s nguyn t p > 2n xut hin trong s phn tch ca
nhiu nht
n
!
2n
2n
vi n  3:
1 ln. Hn na, s nguyn t p tha mn
< p  n s khng l c ca
n
3
p

B 5. Vi mi s thc x  2, ta c
Y

p  4x

(0.2)

px

trong tch chy qua tt c cc s nguyn t p.


Chng minh.
Trc ht ta ch rng nu q l s nguyn t ln nht tha mn q  x th
Y

pD

px

p v 4q

 4x 1 :

pq

Do ta ch cn chng minh cho trng hp x D q l s nguyn t bng quy np. Vi q D 2,


(0.2) hin nhin ng. By gi ta xem xt (0.2) cho trng hp s nguyn t l q D 2m C 1. Gi
s (0.2) ng cho tt c cc s t nhin x  2m. Cho q D 2m C 1, ta c
Y

pD

pmC1

p2mC1

Y
mC1<p2mC1

Ta c
2m C 1
p
m
mC1<p2mC1
Y

127

p:

Tp ch Epsilon, S 06, 12/2015


.2m C 1/
l s t nhin v tt c cc s nguyn t v tri l c ca t s .2m C 1/
m.m C 1/
nhng khng phi l c ca mu s m.m C 1/. Li c
!
!
!
2mC1
X 2m C 1
2m C 1
2m C 1
D
v
D 22mC1
m
mC1
k
kD0
!
2m C 1
nn
 22m . Do ,
m
!
Y
2m
C
1
p  4m
 4m 22m D 42m :
m
p2mC1
!
2n
By gi, ta s chng minh nh Bertrand. Trc ht ta nh gi
: T (0.1) v B 4,
n
vi n  3, ta c
!
Y
Y
Y
4n
2n
2n
p


p;
2n
n
p
p
2n
n<p2n
v

p 2n

hay
4n  .2n/1C

2n<p

2n

p
2n<p 2n
3

p:

n<p2n

Kt hp vi B 0.2, ta c
4n  .2n/1C

p
2n

2n

43

p;

n<p2n

hay
p

4 3  .2n/1C

2n

p:

(0.3)

n<p2n

Gi s rng tn ti s nguyn dng n sao cho khng


Yc s nguyn t p tha mn n < p  2n.
Theo B 2, ta suy ra n  4000. T (0.3), ta c
p D 1 v
n

n<p2n
p
1C 2n

4 3  .2n/
hay

22n  .2n/3.1C

p
2n/

(0.4)
:

(0.5)

Dng bt ng thc a C 1 < 2a ; a  2, ta c


p
p
p
p
6
6
6
6
2n D . 2n/6 < . 2n C 1/6 < 26 2n  26 2n ;
v do
22n  .2n/3.1C

p
2n/

<2

p
6

p
2n.18C18 2n/

< 220

p
6

p
2n 2n

D 220.2n/

2=3

vi n  4000: Suy ra .2n/1=3 < 20 hay n < 4000 (mu thun).


Vy ta chng minh xong nh Bertrand.
T cch chng minh nh Bertrand ca Erdos trn, chng ta c th chng minh kt qu sau:
128

Tp ch Epsilon, S 06, 12/2015


H qu 1. C hng s c; C > 0 sao cho vi mi s dng x, ta c
c ln x
ln x
 .x/  C
x
x
trong .x/ l s cc s nguyn t khng vt qu x.
Vic chng minh h qu trn nh l mt bi tp dnh cho cc bn. Mt nh l ni ting v c
lng hm .x/
.x/
lim
D1
x!C1 x= ln x
c chng minh ln u tin bi Hadamard v de la Valle-Poussin nm 1896 vi cng c
t gii tch phc. Dng nh l ny, chng ta cng c th chng minh nh Bertrand. Cu
chuyn v nh Bertrand vn cha kt thc khi c kh nhiu cu hi tng t c t ra.
Gn y, nm 2006 El Bachraoui [2], a ra mt chng minh s cp cho v vic lun c s
nguyn t nm gia 2n v 3n vi s nguyn dng n > 1. Mt kt qu tng t nm 2011 c
Loo [5] c chng minh cho vic lun tn ti s nguyn t nm gia 3n v 4n: Mt cu
hi t ra cho vic chng minh bi ton tng qut: Cho k; n l cc s nguyn dng, k  n v
n > 1, khi lun tn ti s nguyn t nm trong on k nI .k C 1/n. Nu tr li c cu hi
ny, chng ta s c li gii cho mt vn rt ni ting m cho n nay cha c li gii:
Gi thuyt. Lun tn ti s nguyn t gia n2 v .n C 1/2 vi mi s nguyn dng n?
Sau y l mt s bi ton lin quan n cc vn m chng ti cp trn.
Bi tp 3 ([4]). Cho s nguyn dng n, tp f1; 2; : : : ; 2ng c th c chia thnh cc cp
fa1 ; b1 g; : : : ; fan ; bn g
sao cho vi mi 1  i  n, ai C bi l s nguyn t.
Chng minh. Ta chng minh quy np theo n. Vi n D 1, kt qu l tm thng. Vi n > 1, t
p l s nguyn t tha mn 2n < p  4n. T 4n khng l s nguyn t nn p D 2n C m vi
2n m C 1
1  m < 2n v m l s l. R rng tp fm; : : : ; 2ng c chia thnh
cp
2
f.m; 2n/; .m C 1; 2n

1/; : : : ; .n C

1
2

;n C

mC1
/;
2

sao cho tng ca mi cp l s nguyn t p. Theo gi thit quy np, r rng tp f1; 2; : : : ; m 1g
m 1
c chia thnh
cp sao cho tng ca mi cp l s nguyn t. T suy ra iu phi
2
chng minh.
Bi tp 4. Chng minh rng vi mi s nguyn dng k; lun tn ti dy cc s nguyn dng
fa1 ; a2 ; :::; a2k g sao cho c th chia chng thnh k cp phn bit m tng ca mi cp l s
nguyn t.
Chng minh. Chn ai D i , theo bi trn ta c iu phi chng minh.
129

Tp ch Epsilon, S 06, 12/2015


Bi tp 5. Cho p l s nguyn t v
n D ak p k C ak 1 p k

C    C a1 p C a0

l s khai trin ca n trong c s p: K hiu sp .n/ D ak C ak


p .n/ D

C    C a0 ; chng minh rng

sp .n/
:
p 1

Chng minh. p dng nh l Legrendre, ta c


vp .n/ D

X n
n
n
n
i D C 2 C k
p
p
p
p
i 1

D .ak p k

C    C a2 p C a1 / C .ak p k

D a1 C a2 .1 C p/ C    C ak .p k
n sp .n/
D
:
p 1

C    C a2 / C    C ak

C    C p C 1/

T bi tp trn, ta thy rng vi mi s nguyn dng n th sp .n/  1 nn chng ta lun c bt


ng thc: Vi mi s nguyn dng n
p .n/ 

n
p

1
:
1

ng thc xy ra khi v ch khi n D p r :


T , p dng bt ng thc trn vi p D 2, chng ta cng d dng gii cc bi ton sau:
Bi tp 6. Vi mi s nguyn dng n, ta c 2n n
Bi tp 7. Vi mi s nguyn dng n, ta c 2n

j n nu v ch nu n l ly tha ca 2:

Ngoi ra, bn c c th tham kho [2, 5] cho cc kt qu sau:


Bi tp 8. Cho s nguyn dng n  2. Chng minh rng lun tn ti s nguyn t p tha mn
n<p<

3.n C 1/
:
2

Bi tp 9. Cho s nguyn dng n  3. Chng minh rng lun tn ti s nguyn t p tha mn


n<p<

4.n C 2/
:
3

Bi tp 10. Vi mi " > 0, tn ti s n0 sao cho vi mi n > n0 th c t nht .


nguyn t nm gia n v 2n.
130

2
3

"/

n
s
log2 n

Tp ch Epsilon, S 06, 12/2015

Ti liu tham kho


[1] M. Aiger and Gunter M. Ziegler, Proofs from THE BOOK, Chapter 2, Fourth Edition,
Springer 2010.
[2] M. El Bachraoui, Primes in the Interval 2n; 3n, Int. J. Contemp. Math. Sci., Vol 1, 2006,
no 13, 617-621.
[3] P. Erdos, Beweis eines Satzes von Tschebyschef, Acta Sci. Math. 5 (1932), 194-198.
[4] L.E. Greenfield and S.J. Greenfield, Some Problems of Combinatorial Number Theory
Related to Bertrands Postulate, J. Integer Sequences, Vol 1, 1998.
[5] A. Loo, On the Primes in the Interval 3n; 4n, Int. J. Contemp. Math. Sci., Vol 6, 2011, no
38, 1871-1882.

131

Tp ch Epsilon, S 06, 12/2015

132

CHUI IU HA
Kiu nh Minh
(THPT Chuyn Hng Vng, Ph Th)

1. M u
Chui iu ho

1
X
1
1 1
D 1 C C C :::
k
2 3

kD1

l mt trong cc chui v hn ni ting. Tng ring th n,


Hn D 1 C

1 1
1
C C ::: C
2 3
n

c gi l s iu ho. Dy .Hn / c gi l dy s iu ho (hay dy iu ho). Chui c


dng
1
X
1
1
1
D
1
C
C
C :::
km
2m
3m
kD1

vi m 2 N cn gi l chui iu ho bc m. Chui iu ho tng qut l chui c dng


1
X
1
, vi a 0; b l cc s thc. Ngoi ra
ak C b
kD1

1
X
. 1/kC1
D1
k

kD1

1 1
C
2 3

1
C :::
4

gi l chui iu ho an du.
Trong cc cng trnh nghin cu ca nhiu nh ton hc i khi c lin quan n chui iu
ho v trong cc k thi hc sinh gii chng ta cng hay bt gp n cc bi ton kh. C nhiu
hng tm hiu khc nhau i vi chui ny chng hn nh bi ton v nh gi, bi ton v
gii hn hay cc bi ton v s hc. . . Bi vit ny chng ti s trnh by mt s kt qu c bn
v chui iu ho cng nh cc bi ton lin quan m chng ta thng gp trong cc k thi
Olympic.

2. Cc bi ton i s
V d 1. (ng nht thc Catalan) Chng minh rng vi mi n 2 N , ta c:
1

1 1
C
2 3

1
C :::
4

1
1
1
1
D
C
C ::: C
:
2n
nC1 nC2
2n
133

Tp ch Epsilon, S 06, 12/2015


Chng minh. Bin i v tri ta c

1 1
V T D 1 C C C ::: C
2 3

1 1
D 1 C C C ::: C
2 3


1
2n

1
2n


1 1
1
2
C C ::: C
2 4
2n


1 1
1
1 C C C ::: C
D VP:
2 3
n

y l mt ng nht thc kh n gin, tuy nhin li c nhiu ng dng khi gii ton. Chng ta
s bt gp iu ny trong phn sau.
V d 2. Vi n 2 N , cho
Hn D 1 C

1 1
1
T1
T2
Tn
C C ::: C I Tn D H1 C H2 C ::: C Hn I Un D
C
C ::: C
2 3
n
2
3
nC1

Chng minh rng


Tn D .n C 1/HnC1

.n C 1/ v Un D .n C 2/HnC1

Chng minh. Ta c:
 



1 1
1 1
1
C 1C C
C ::: C 1 C C C ::: C
Tn D 1 C 1 C
2
2 3
2 3

 
n n 1 n 2
1
nC1
nC1
D C
C
C ::: C D
1 C
1
2
3
n
1
2


1
1 1
D .n C 1/
C C ::: C
n
1 2
n
D .n C 1/Hn n D .n C 1/HnC1 .n C 1/
v

.2n C 2/.

1
n




nC1
1 C ::: C
n

T1
T2
Tn
C
C ::: C
D .H2 1/ C .H3 1/ C ::: C .HnC1
2
3
nC1
D H2 C H3 C ::: C HnC1 n D H1 C Tn C HnC1 n
D .n C 2/HnC1 .2n C 2/

Un D


1

1/
:

Vy bi ton c chng minh.


V d 3. (Canada MO 1998, Albania BMO TST 2014) Cho n 2 N; n  2. Chng minh rng




1
1
1
1 1 1
1
1 C C ::: C
>
C C ::: C
:
nC1
3
2n 1
n 2 4
2n
Chng minh. Ta gii bi ton ny theo hai cch sau:
Li gii 1. Bt ng thc cn chng minh tng ng vi
2n

2n

X1
1 X1
2n C 1 X 1
2n X 1
1X 1
>
,
>
C
n C 1 i D1 i
n.n C 1/ i D1 2i
i
n i D1 2i
n i D1 2i
iD1


1
1
1
1 1 1
1
,
C
C ::: C
>
C C ::: C
nC1 nC2
2n
n 2 4
2n
134

./

Tp ch Epsilon, S 06, 12/2015


Ta c
V T ./ >

n
1
1 n
D > : > VP ./:
2n
2
n 2

Vy bi ton c chng minh.


Li gii 2. Bt ng thc cn chng minh tng ng vi




1
1
1 1
1
n 1 C C ::: C
> .n C 1/
C C ::: C
:
3
2n 1
2 4
2n

./

Ta s chng minh ./ bng quy np.


8
9
Tht vy vi n D 2 th tr thnh > (ng).
3
4
Gi s ./ ng vi n D k  2, tc l




1
1 1
1
1
> .k C 1/
C C ::: C
:
k 1 C C ::: C
3
2k 1
2 4
2k
Ta c





1
1
1
kC1
kC1
1 1
1
1 C C ::: C
D
C C ::: C
C
C C
3
2k 1
2k C 1
2 3
2k 1
2 2k C 1


1 1
1
1
kC1
>
C C ::: C
C C
2 4
2k
2 2k C 1


1 1
1
kC1
1
>
C C ::: C
C
C
2 4
2k
2k C 2 2k C 1


1
kC2
1 1
>
C C ::: C
C
:
2 4
2k
2k C 2

Do 
.k C 1/ 1 C

1
1
1
C ::: C
C
3
2k 1 2k C 1


 

1
1
1
1
kC1
D k 1 C C ::: C
C 1 C C ::: C
C
3
2k 1
3
2k 1
2k C 1
 


1
1
1 1
1
kC2
C
C C ::: C
C
> k 1 C C ::: C
3
2k 1
2 4
2k
2k C 2
 


1 1
1
1 1
1
kC2
> .k C 1/
C C ::: C
C
C C ::: C
C
2 4
2k
2 4
2k
2k C 2


1 1
1
D .k C 2/
C C ::: C
:
2 4
2k C 2

Theo nguyn l quy np th ./ ng vi mi n 2 N; n  2.

V d 4. (USA MO 1995) Cho a1 ; a2 ; a3 : : : ; l mt dy cc s thc dng tho mn


p

n
X

aj 

j D1

n vi mi n  1. Chng minh rng vi mi n  1, ta c




n
X
1
1 1
1
2
aj >
1 C C ::: C
:
4
2
3
n
j D1

Chng minh. Ta bt u bng chng minh bt ng thc sau:


Nu a1 ; a2 ; : : : ; an l dng v b1  b2  : : :  bn  0 v nu vi mi k  n; a1 C a2 C : : : C
ak  b1 C b2 C : : : C bk , th
a12 C a22 C : : : C an2  b12 C b22 C : : : C bn2
135

./

Tp ch Epsilon, S 06, 12/2015


S dng cng thc khai trin Abel, ta c th vit
a1 b1 C a2 b2 C : : : C an bn D a1 .b1
 b1 .b1

b2 / C .a1 C a2 /.b2
b2 / C .b1 C b2 /.b2

b3 / C .a1 C a2 C a3 /.b3 b4 / C : : : C .a1 C a2 C : : : C an /bn


b3 / C .b1 C b2 C b3 /.b3 b4 / C : : : C .b1 C b2 C : : : C bn /bn

D b12 C b22 C b32 C : : : C bn2

Hay ta c (*) ng.


p dng bt ng thc Cauchy Schwarz, ta c


2
a12 C a22 C : : : C an2 b12 C b22 C : : : C bn2  .a1 b1 C a2 b2 C : : : C an bn /2  b12 C b22 C : : : C bn2 :
Tr li bi ton, vi ch
n
X

p
p
1
1 > p . Chn bn D n
n
2 n

1 th

n
X

aj 

nD

j D1

bj T ./ suy ra

j D1
n
X
j D1

aj2

n 
X
p
j


p
j

2

j D1



2
n 
X
1 1
1
1
1
1 C C ::: C
:
>
D
p
4
2 3
n
2 j
j D1

Vy bi ton c chng minh.


V d 5. Chng minh ng thc
 
1
1
n
1 C C ::: C D
1
2
n

 
 
 
1 n
1 n
nC1 1 n
C
C : : : C . 1/
:
2 2
3 3
n n

Chng minh. Da vo cc ng thc


   
 
 
n
n
n 2
n n
n
n
.1 x/ D
xC
x C : : : C . 1/
x
0
1
2
n
 
 
 
  :
n
n 2
n 3
n n
n
n 1
1 .1 x/ D
x
x C
x
: : : C . 1/
x
1
2
3
n
Ta c
Z1
1

Z1   
.1 x/n
n
dx D
1
x
0
0
 
 
1
n
n
C
D
2
1
2
Mt khc, t y D 1 x th
Z1

.1 x/n
dx D
x


 
n
n 2
xC
x
2
3
1
3

Z1

n 1

: : : C . 1/


 
n
n
nC1 1
C : : : C . 1/
3
n
n
yn

1
y

dy

Z1
D

1 C y C y2 C : : : C yn


1 2
1 n 1
D y C y C ::: C y
2
n
0
1
1
D 1 C C ::: C
2
n


136

dy



n n 1
x
dx
n

Tp ch Epsilon, S 06, 12/2015

3. Cc bi ton gii tch


Z1
Euler a ra cng thc gii tch Hn D

1 xn
dx cho chui iu ho. Trong mc ny
1 x

chng ta cng tm hiu cc bi ton gii tch lin quan n chui iu ho v cc bi ton lin
quan n hng s Euler.
V d 6. Chng minh rng
1C

1 1
1
C C : : : C > ln.n C 1/; 8n 2 N :
2 3
n

Chng minh. C nhiu cch chng minh bt ng thc ny. Sau y chng ti gii thiu hai
cch chng minh c bn nht.
Cch chng minh th nht: S dng bt ng thc ln.1 C x/ < x; 8x > 0, ta c




1 1
1
1
1
C : : : C ln 1 C
D ln.n C 1/:
1 C C C : : : C > ln.1 C 1/ C ln 1 C
2 3
n
2
n
1
trong on 1I n C 1. Gi S l din tch hnh
x
1
thang cong c gii hn bi cc ng x D 1; x D n C 1; y D 0; y D . Khi
x

Cch chng minh th hai: Xt hm s y D

nC1
Z

SD

dx
D ln.n C 1/
D ln xjnC1
1
x

.1/


1
Gi Ai l cc im vi to iI
; i D 2; 3; : : : ; n. K hiu A1 .1I 0/; AnC1 .n C 1I 0/. Gi
i


1
Bi iI
; i D 2; 3; : : : ; n C 1 v B1 .1I 1/.
i 1
Gi S1 l din tch ca a gic A1 B1 B2 A2 B3 A3 : : : Bn An BnC1 AnC1 . Khi


S1 D 1 C

1 1
1
C C ::: C
2 3
n

.2/

1
Do hm s y D nghch bin trn 1I n C 1 nn S1 > S . Do t .1/; .2/ suy ra iu phi
x
chng minh.
Nhn xt: T kt qu trn ta suy ra c


1
1
D C1:
lim 1 C C : : : C
2
n

./

Kt qu (*) c khong gn 50 chng minh (xem [2]). Tuy nhin ch yu theo cc hng nh gi
tri, so snh din tch hnh phng qua tch phn hay s dng hng s Euler.
137

Tp ch Epsilon, S 06, 12/2015


ln n/ D

V d 7. a) Chng minh tn ti gii hn lim .Hn


n!C1

b) Chng minh rng < Hn C Hm Hnm  1; 8n; m 2 N


c) Tnh gn ng vi sai s cha n 0; 1.
Chng minh. a) t an D Hn
an  an

ln.n C 1/; bn D Hn

ln n th .an / tng v


1
nC1
1 n
ln.n C 1/  0 ,  ln
,e  1C
n
n
n

1
C ln n
n

bt ng thc ny ng. Tng t th .bn / gim. Mt khc an < bn v




n
D 0:
lim .an bn / D lim ln
n!C1
n!C1
nC1
Do tn ti lim an D lim bn D .
n!C1

n!C1

b) Ta c
Hn C Hm

Hnm D .Hn C Hm Hn / .H2n Hn / .H3n


1
1
1
 Hm n:
n:
: : : n:
D1
2n
3n
mn

H2n /

:::

Hmn

H.m

Li c
bmn < bn , Hmn

ln.mn/  Hn

ln n , Hn

Hmn C ln.mn/

ln n  0:

./

V
bm > , Hm

./

ln m > :

Cng ./ v ./ ta c


Hn

Hmn C ln.mn/

ln n C Hm

ln m > , Hn C Hm

Hmn > :

c) ly n D 100 tnh c an D 0; 57I bn D 0; 58. Suy ra  0; 57


1
1
Nh vy th 1C C: : :C D Cln nC"n ; "n ! 0 khi n ! C1 v c gi l hng s Euler
2
n

ZC1
1
1
Mascheroni. Ch rng D lim .Hn ln n/ D
dx D 0; 5772156649 : : :. Cho
x x
1

n nay ngi ta vn cha bit rng l s hu t hay s v t. Tuy nhin th hng s ny c vai
tr rt quan trng trong ton hc.
V d 8. (Trng ng Ton hc 2013) Cho dy s .an / xc nh bi a1 D
anC1 D an

3n C 2
; n D 1; 2; : : :
2n.n C 1/.2n C 1/

Tnh gii hn lim an .


n!C1

138

3
v
2

1/n

Tp ch Epsilon, S 06, 12/2015


Chng minh. Ta c ng nht
3k C 2
1
D
2k.k C 1/.2k C 1/
k
Do
an D

1
2.k C 1/

1
:
2k C 1

1
1
1
C
C ::: C
:
n nC1
2n

Suy ra
an D H2n Hn

D . C ln 2n C "2n / . C ln.n

1/ C "n 1 / D ln

2n
n

C."2n

"n 1 / ! ln 2;

khi n ! C1.


1
1
1
Ch : C th s dng bt ng thc
< ; 8n  1 v nh l gii hn
< ln 1 C
nC1
n
n
kp suy ra lim an D ln 2.
n!C1

V d 9. Gi s xn 2 .0I 1/ l nghim ca phng trnh


1
1
1
C
C ::: C
D 0:
x
x 1
x n
Chng minh dy .xn / hi t. Tm gii hn .
Chng minh. t fn .x/ D
Ta thy 0 < xn < 1 nn

1
1
1
C
C ::: C
, th fn .xn / D 0.
x
x 1
x n

fnC1 .xn / D fn .xn / C

xn

1
n

xn

1
n

< 0:


Trong khi fnC1 0C > 0. Theo tnh cht ca hm lin tc, trn khong .0I xn / c t nht mt
nghim ca fnC1 .x/ , nghim chnh l xnC1 . Suy ra xnC1 < xn , tc l dy .xn / gim. Do
dy ny b chn di bi 0 nn n hi t. Ta chng minh gii hn bng 0.
Gi s lim xn D a > 0. Khi , do dy .xn / gim nn ta c xn  a; 8n:
Do
1 1
1
1 C C : : : C ! C1
2 3
n
khi n ! C1 nn tn ti N sao cho vi mi n  N ta c
1C

1 1
1
1
C ::: C > :
2 3
n
a

Khi vi n  N th
0D

1
1
1
1
1
1
1
1
C
C ::: C
<
C
C
C ::: C
<
xn
xn 1
xn n
xn
1
2
n
a

Mu thun ny chng t phi c lim xn D 0.


139

1
D 0:
a

Tp ch Epsilon, S 06, 12/2015


V d 10. Tn ti hay khng hai a thc f .x/; g.x/ 2 Rx tho mn
f .x/
1 1
1
D 1 C C C : : : C ; n 2 N :
g.x/
2 3
n
Chngminh. Gi s tn ti hai
 a thc f .x/; g.x/ 2 Rx tho mn yu cu bi ton. V
1
1 1
f .x/
lim 1 C C C : : : C
D C1 nn lim
D C1, suy ra deg f > deg g. Gi
n!C1
x!C1 g.x/
2 3
n
a0 ; b0 ln lt l h s dn u ca f .x/; g.x/ th
"
C1
f .x/
a0
.1/
D
lim
x!C1 xg.x/
b0
Mt khc vi N > 0 tu , ta c


1
1
1
1
1
0  lim
1 C C ::: C
C
C ::: C
n!C1 n
2
N
N C1
n

 

1
1
1
1
1
D lim
1 C C ::: C
C
C ::: C
n!C1 n
2
N
N C1
n




1
1
1
1
1
1
1 C C ::: C
C lim
C ::: C
 lim
n!C1 n N C 1
n!C1 n
2
N
n
n N
1
 0 C lim

n!C1 n.N C 1/
N
Do



1
1
1
lim
1 C C ::: C
D0
n!C1 n
2
n

.2/

T .1/ v .2/ suy ra mu thun. Vy khng tn ti a thc f .x/; g.x/ 2 Rx tho mn yu cu


bi ton.

4. Cc bi ton s hc
V d 11. (Brazil MO 1983) Chng minh rng
Hn D 1 C

1 1
1
C C ::: C
2 3
n

khng l s nguyn vi mi n 2 N; n > 1.


Chng minh. K hiu k l s t nhin tho mn 2k  n < 2kC1 v M l tch tt c cc s l
khng vt qu n. Khi
Hn D 1 C

1 1
1
1
C C : : : C kC1 C : : : C
2 3
2
n

Suy ra
k 1

k 1

:M:Hn D 2

Vy Hn D 1 C

k 2

M C2

M C2

k 1

M
M
2k 1 M
:
C ::: C 2 C ::: C
Z:
3
2
n

1 1
1
C C : : : C khng th l mt s nguyn.
2 3
n
140

Tp ch Epsilon, S 06, 12/2015


V d 12. (IMO 1979) Cho p; q 2 N sao cho
p
D1
q

1 1
C
2 3

1
C :::
4

1
1
C
:
1318 1319

Chng minh rng p chia ht cho 1979.


Chng minh. p dng ng nht thc Catalan ta c
p
1
1
1
1
D
C
C ::: C
C
q
660 661
1318 1319


1
1
1
1
1 1
C
C ::: C
C
C ::: C
D
2 660 661
1319 6601
1319

 



1
1
1
1
1
1
1
C
C
C
C
C ::: C
D
2 660 1319
661 1318
1319 660


1979
A
1
1979
1979
D 1979:
D
C
C ::: C
2 660:1319 661:1318
1319:660
B
y B l tch ca cc s nguyn khng vt qu 1319.
Do 1979 l s nguyn t, v vy 1979jp.
V d 13. Cho p > 3 l mt s nguyn t, m v l cc s nguyn nguyn t cng nhau sao cho
1
1
1
m
D 2 C 2 C ::: C
:
n
1
2
.p 1/2
Chng minh rng m chia ht cho p
Chng minh. Ch rng
..p

m
1// : D ..p
n
2

1//

1
1
1
C
C
:
:
:
C
12
22
.p 1/2

l mt s nguyn. Cng ch rng




1 1
1
; ;:::;
1 2
p 1

l mt h thng d y modulo p. Theo nh l Wilson, ta c






1
1
1
2
2 2
2
2
.
.p
..p 1//
C
C
:
:
:
C

1/
1
C
2
C
:
:
:
C
1/
12
22
.p 1/2
.p 1/p.2p 3/

 0.modp/
6
Do p  5 v gcd.6; p/ D 1. V vy p chia ht
Do gcd..p

..p

1/; p/ D 1, ta phi c pjm.

1//2 m
.
n

V d 14. (nh l Wolstenholme) Cho p > 3 l s nguyn t. Chng minh rng




1
1
2
p j.p 1/ 1 C C : : : C
:
2
p 1
141

Tp ch Epsilon, S 06, 12/2015


Chng minh. t


1
1
1/ 1 C C : : : C
2
p 1

S D .p
th

p 1


X 1
1
1/
D .p
C
i
p
i
i D1

2S D .p
p 1

y T D .p

1/

X
i D1

i.p

i/

p 1

1/

X
i D1

p
D p:T
i.p i/

Do 2S l mt s nguyn v p l s nguyn t cng nhau vi cc mu s ca cc s hng trong


T , bn thn T l mt s nguyn. Do p > 3; gcd.p; 2/ D 1 v p phi chia ht S . Ta cn ch ra p
cng chia ht T . Theo bi ton trn, ta c
p 1

T  .p

1/

X
i D1

1
i2


 .p

1/

m
 0.modp/
n

do pjm v gcd.m; n/ D 1.
V d 15. (nh l Euclide) Chng minh rng c v hn s nguyn t
Chng minh. nh l ny c khong gn 20 chng minh v chng minh sau l ca Euler.
Gi s c hu hn s nguyn t p1 < p2 < : : : < pm . t
!
m
m
Y
Y
1
1
1
N D
1C
C ::: C k C ::: D
:
1
p
p
i
i
i D1
i D1 1
pi
Mt khc, bng khai trin v s dng phn tch chun tc ca cc s nguyn dng, ta c
N D1C

1 1
C C :::
2 3

Suy ra
m
Y
iD1

1
1

m
Y
1 1
pi
D 1 C C C : : : ! C1 )
! C1:
1
2 3
p
1
i
i D1
pi

Mu thun ny chng t iu gi s l sai. Vy tp cc s nguyn t l v hn.


Nhn xt: Ta c
m
Y
i D1

1
1

m 
Y
1 1
1
 ! C1 )
D 1 C C C : : : ! C1 ) m 
1
1
Q
1
2 3
i
D1
1
pi
pi
i D1

1
pi


! 0;

C1
X
1
suy ra
D C1.
pi
i D1

Paul Erds cng a ra hai chng minh rt p cho s phn k ca chui


142

C1
X
1
(xem [2]).
p
i
i D1

Tp ch Epsilon, S 06, 12/2015


Ngy nay c nhiu hng nghin cu lin quan n chui iu ho, chng hn nh: Nghin cu
cc cch chng minh s phn k ca chui, m rng hng s Euler v cc ng nht thc, m
rng nh l Wolstenholme, bi ton v s phn b s nguyn t, s P- adic,...
Hy vng bn c s tm thy nhng i u b ch khi c bi vit ny v t tm ti thm nhng vn
nu trn. V bi vit c hon thnh trong thi gian ngn v s hiu bit ca bn thn tc
gi c hn nn khng trnh khi thiu st. Tc gi rt mong nhn c s gp chn thnh t
bn c bi vit c hon chnh hn.
Cui cng chng ti xin nu mt s bi ton lin quan n chui iu ha cung cp thm t
liu cho bn c.
1. (IMO SL 1989) Chng minh rng
1
1C
2

2 1 1
C C
3 4 5

2
1
1
C ::: C
C
6
478 479

159

X
2
641
D2
.161 C k/.480
480
kD0

k/

2. (Canada MO 1973) Chng minh rng


n C H1 C H2 C H3 C : : : C Hn

D nHn ; 8n  2

3. (Rom Math Magazine, July 1998) Cho


AD

1
1
1
1
1
1
C
C:::C
; BD
C
C:::C
:
1:2 3:4
2011:2012
1007:2012 1008:2011
2012:1007

Tnh

A
.
B

4. (IMC, Senior Individual Contest 2013) Cho biu thc



 2



1
1
1
1
1
1 2
1 2
C
C ::: C
1 C C ::: C C 1 C C ::: C
C ::: C
2
n
2
n
2
n
n
bt u t ngoc n th hai, tng bn trong nhn c khi ta b i hng t u tin ca
tng trong ngoc n pha trc. Tnh gi tr ca biu thc khi n D 2013.
5. (APMO 1997) Cho
S D1C

1
1
1C
3

1
1
C ::: C
1 1
1 1
1
1C C
1 C C C ::: C
3 6
3 6
1993006

y cc mu s cha tng ring ca dy cc nghch o ca s tam gic. Chng minh


rng S > 1001.
1
1
1
6. (T2/198 THTT) t kn D 1 C C C : : : C
. Chng minh rng vi mi s
3
5
2n 1
nguyn dng n, ta c
1
1
1
1
C 2 C 2 C ::: C
<2
2
.2n 1/kn2
k1
3k2
5k3
143

Tp ch Epsilon, S 06, 12/2015


7. Cho a1 ; a2 ; : : : ; an l cc s khng m sao cho a1 a2 : : : ak 

1
; 8k. Chng minh
.2k/

rng
a1 C a2 C : : : C ak 

1
1
1
C
C ::: C
nC1 nC2
2n

8. (IMO SL 1978) Cho W N ! N l mt hm n nh. Chng minh rng vi mi s t


n
n
X
.k/ X 1

nhin n th
k2
k
kD1

kD1

9. (Germany MO 1980) Chng minh rng vi cc s tu n; k 2 N ln hn 1, ta c bt


nk
n
X
X
1
1
ng thc
>k
j
j
j D2
j D2
10. Chng minh cc ng thc:
     
 
n
n
n
n
1
2
3
n
1 1
1
a)
C
C : : : C . 1/nC1
D C C ::: C
1:2
2:3
3:4
n.n C 1/
2 3
nC1
     
 
n
n
n
n


0
1
2
n
1
1 1
1
n
b) 2
C 2 C : : : C . 1/
D
1 C C C ::: C
:
1
22
3
nC1
2 3
n
.n C 1/2
r
1
1
n
11. Tnh gii hn lim
1 C C ::: C :
n!C1
2
n
. 1/n
; n  1.
12. Cho dy s .un / xc nh bi u1 D 1; unC1 D un C
nC1
a) Chng minh rng u2n D H2n Hn ; 8n  1.
b) Chng minh rng dy .un / c gii hn hu hn v tm gii hn .
13. Tm gii hn ca dy an D

1
1
1
C
C
:
:
:
C
; n  1.
12
12 C 22
1 2 C 2 2 C : : : C n2

14. Cho dy s .an / c gii hn lim an D a. Chng minh rng


n!C1


lim

n!C1

a1
a2
an
C
C ::: C
nC1 nC2
2n

15. (Olympic 30/4/2015) Cho dy s un D


lim un :

1
en C 1
nC1

1
en C 2
nC2

D a ln 2:

C ::: C

1
e 2n
2n

; 8n  1. Tm

n!C1

16. Dy s .xn / tho mn iu kin jxn


cho khng b chn.

xm j >

144

1
; 8n; m 2 N I n < m. Chng minh dy
n

Tp ch Epsilon, S 06, 12/2015


1
; n  1. Tm gii hn
n cos n
x1 C x3 C : : : C x2n 1
:
lim
x2 C x4 C : : : C x2n

17. (PTNK TST 2015) Cho dy .xn / W xn D

18. Chng minh rng dy .Rn / xc nh bi


1
1
Rn D 1 C C : : : C
2
n



1
1
ln n C C
2 24n

hi t ti
19. (VN TST 1999) Cho dy s thc dng .un /1
nD1 . Vi mi s nguyn dng n, gi s kn


k
n
n
X1 X
knC1
l s nguyn dng nh nht tho mn

ui . Chng minh rng dy
c
i
kn
i D1
i D1
gii hn hu hn khi v ch khi dy .un / c gii hn hu hn.
1 1 1
1
1
C
C:::C
c biu
2 3 4
2n 1 2n
din di dng mt phn s ti gin. Gi s 3n C 1 l mt s nguyn t. Chng minh rng
t s ca phn s ny l mt bi ca 3n C 1.

20. (ITOT, Junior A Level, Fall 2013) S 1

21. Chng minh rng H .m; n/ D

1
1
1
C
C : : : C Z; 1  m < n
m mC1
n

22. (IMO SL 1979) Chng minh rng khng c s nguyn a  1; n  1 sao cho
1C

1
1
1
C
C ::: C
2 Z:
1Ca
1 C 2a
1 C na

1 1
1
a
23. (ARML 2002) Cho a l s nguyn sao cho 1 C C C : : : C
. Tm phn d
D
2 3
23
23
khi a chia cho 13.
pn
1
1
24. (Bulgaria MO 2004) Vi mi n 2 ZC , tng 1 C C : : : C c vit di dng
vi
2
n
qn
.pn ; qn / D 1.
a) Chng minh rng 3 khng chia ht p67
b) Tm tt c n 2 ZC sao cho 3 chia ht pn .
1 1
1
C C ::: C
; k  1.
3 5
2k 1
1
1
a
26. Cho p  5 l mt s nguyn t. Chng minh rng nu 1 C C : : : C
D
th
2
p
b
p 4 j.ap b/.
25. (AMM E1408) Tm v2 .A/, vi A l t s ca 1 C

27. Cho p l s nguyn t l. t


a
.p
Chng minh rng a 

1/

D1

2p

2
p

1 1
C
2 3

.modp/
145

::: C

1
p

1
2

Tp ch Epsilon, S 06, 12/2015


28. Chng minh rng chui

X1
8n

l hi t. y 8 n tc l n khng cha ch s 8 trong

biu din thp phn


29. (VN TST 2005) Mt s nguyn dng c gi l s Kim cng 2005 nu trong biu
din thp phn ca n c 2005 s 9 ng cnh nhau lin tip. Dy .an /; n D 1; 2; 3; : : : l
dy tng ngt cc s nguyn dng tho mn an < nC .C l hng s thc dng no ).
Chng minh rng dy s .an /; n D 1; 2; 3; : : : cha v hn s Kim cng 2005.
30. (VN TST 1998) Cho s nguyn dng m > 3. Gi s p1 ; p2 ; : : : ; pk l tt c cc s

k 
X
1
1
C 2 > ln.ln m/
nguyn t khng vt qu m. Chng minh rng
p
pi
i
i D1
31. (APMO 2006) Cho p  5 l mt s nguyn t v r l s cc cch t p qun c am trn
bn c p  p sao cho tt c cc qun c khng cng mt hng ( nhng c th cng ct).
Chng minh rng r chia ht cho p 5 .
p 1

32. (Ibero American 2005) Cho p > 3 l mt s nguyn t. Chng minh rng nu
n
vi .n; m/ D 1 th p 3 jn.
m

X 1
D
p
i
i D1

p 1

33. Cho p l s nguyn t l. t T .p; k/ D


cho p.

X 1
; k 2 N . Xt s d ca T .p; k/ khi chia
k
i
i D1

34. (Canada MO 1973) Cho m > 1 l mt s nguyn dng. Chng minh rng
ca cc s hng lin tip trong chui

1
X

1
l tng
m

1
.
j .j C 1/
j D1

 .n/
D C1
n!C1 n
X
36. (Brazil MO 1992) t d .n/ D
1. Chng minh vi mi s t nhin n > 1, ta c
35. Chng minh rng lim

0<d jn

X 1 X d .i /
X 1


i
n
i
2i n
1i n
h

37. Cho hm f W N ! N tho mn f .1/ D 1; vi mi k 2 3d 1 I 3d ; d  1 th
n
X
1
f .k/ D kf .d /. t xn D
. Chng minh rng dy .xn / khng b chn.
f .k/
kD1

38. (USA MO 2010) Cho q D


Sq D

3p

5
2

vi p l mt s nguyn t l, v t

1
1
1
C
C ::: C
:
2:3:4 5:6:7
q.q C 1/.q C 2/
146

Tp ch Epsilon, S 06, 12/2015


Chng minh rng nu

1
p

2Sq D

m
vi m; n 2 Z, th m
n

n chia ht cho p.

39. Cho dy .an / tng ngt gm cc s nguyn dng tho mn dy .anC1 an / b chn.
Chng minh rng tp cc c nguyn t ca dy .an / l v hn.



2p 1
40. a) Chng minh vi mi s nguyn t p > 3 th
 1 modp 3
p 1
   

ap
a
b) Chng minh vi mi s nguyn t p > 3 th

modp 3 I a; b 2 N .
bp
b

 


2n 1
2p 1
l

modp 4
c) Chng minh vi p l mt s nguyn t v n D p ; l  2 th
n 1
p 1
41. (Argentina 
MO 2014) Mt s nguyn n 3 c gi l c bit nu n khng chia
1 1
1
ht .n 1/ 1 C C C : : : C
. Tm tt c cc s c bit trong khong
2 3
n 1
10I 100.
p 1

42. Cho p > 3 l mt s nguyn t, th th

Hk 

kD1

p3
Bp
3


p C 1 modp 4 Trong

Bn l cc s Bernoulli c xc nh bi
B0 D 1; B1 D

1
1
; B2 D v Bn D
2
6

1 X
nC1

kD0


nC1
Bk ; n  1
k

Ti liu tham kho


[1] T. -W. Leung, Harmonic Series," Mathematical Excalibur, vol. 15, no. 16, 2011.
[2] S. J. Kifowit and T. A. Stamps. The Harmonic Series Diverges Again and Again," AMATYC
Review, 2006.

147

Tp ch Epsilon, S 06, 12/2015

148

S D CA A:ax C Bx
Yimin Ge, Vienna, o
(Ngi dch: Nguyn Tt Thu, ng Nai)

1. M u
Trong thi gian gn y, mt s vn ca l thuyt s tr nn ph bin trong cc k thi. C
th l vn tm s d ca ax C bx theo modulo ca s nguyn dng m. Trong bi vit ny,
chng ti a ra bi ton tng qut cho cc bi ton trn.
Chng ta s bt u vi mt nh l c bn v s tn ti thng d bc hai. Mc d trong bi ton
ny ta khng cn n n, nhng tng chng minh nh l c th p dng trong nhiu vn
v chng ti s s dng n trong sut bi vit ny.
nh l 1. Cho p l s nguyn t l, k l s nguyn dng v a l mt s nguyn khng chia ht
cho p. Khi a l thng d bc hai theo modulo p k khi v ch khi a l thng d bc hai theo
modulo p.
Chng minh. Ta thy nu a l thng d bc hai theo modulo p k th hin nhin a l thng d bc
hai theo modulo p. Do , ta ch cn chng minh a l thng d bc hai theo modulo p k vi a l
thng d bc hai theo modulo p. Ta chng minh vn ny bng cch quy np theo k.
Gi s a l thng d bc hai theo modulo p k , ta chng minh a l thng d bc hai theo modulo
p kC1 . Tht vy:
V a l thng d bc hai theo modulo p k nn tn ti s t nhin x sao cho
x2  a

.mod p k /

hay l tn ti s nguyn l sao cho


x 2 D a C l:p k
vi x khng chia ht cho p.
t x 0 D x C y:p k , vi y l mt s nguyn. Ta chng minh tn ti s nguyn y sao cho
x 02  a

.mod p kC1 /:

Ta c

2
x 02 D x C y:p k D x 2 C2xyp k Cy 2 p 2k D aC.lC2xy/p k Cy 2 p 2k  aC.lC2xy/:p k

.mod p kC1 /:

Ta chng minh tn ti y sao cho


l C 2xy  0

.mod p/:

R rng y l phng trnh ng d tuyn tnh v .2x; p/ D 1 nn phng trnh lun c nghim
nguyn y.
Vy nh l c chng minh.
149

Tp ch Epsilon, S 06, 12/2015


tng quan trng trong chng minh trn l mt k thut rt hu ch: S dng gi thit quy np
theo modulo m, ta xy dng mt nghim mi x 0 theo modulo m0 da vo nghim x theo modulo
m bng cch thm vo bin mi y. Ch rng x 0 vn bt bin theo modulo m khi y thay i. N
cn c dng chng minh cc vn khc khi chn nhng gi tr y thch hp.
Bi ton sau xut hin trong k thi Olympic Brazil nm 2005.
Bi ton 1. (Brazil 2005) Cho cc s nguyn dng a, b v c. Chng minh rng tn ti mt s
nguyn dng x sao cho
ax C x  b .mod c/:
Mt trng hp c bit ca bi ton trn xut hin trong IMO Shortlist nm 2006.
Bi ton 2. (IMO Shortlist 2006) Chng minh rng vi mi s nguyn dng n lun tn ti s
nguyn dng m sao cho n l c ca 2m C m.
Mt bi ton tng t c a ra trong k thi USA TST nm 2007.
Bi ton 3. (USA TST 2007) Tn ti hay khng cc s nguyn dng a, b sao cho a khng l
c ca b n n vi mi s nguyn dng n.
Tt c cc bi ton c c tng qut thnh bi ton sau
nh l 2. Cho A; a; B l cc s nguyn v M l s nguyn dng. Khi , iu kin cn v
vi mi s nguyn C , lun tn ti s nguyn dng x sao cho
A:ax C Bx  C

.mod M /

l gcd.B; M / D 1:
Lu : Ta c th pht biu nh li trn theo mt cch khc l
fA:ax C Bx mod mjx 2 ZC g D f0; 1; : : : ; M

1g khi v ch khi gcd.M; B/ D 1.

Chng minh. Ta chng minh iu kin cn: Gi s .M; B/ > 1, gi p l mt c nguyn t


chung ca M v B. Khi , nu pjAa th ta chn C D 1, ngc li ta chn C D 0. Khi ,
phng trnh A:ax C Bx  C .mod p/ khng c nghim x.
Chng minh iu kin : Ta c th chng minh theo hai cch sau:
Cch 1. Gi s .B; M / D 1. Gi C l mt s nguyn bt k v t M D mn vi m; n 2 ZC
sao cho mi c nguyn t ca n l c ca a v gcd.a; m/ D gcd.n; m/ D 1 (iu ny c
ngha l nu a D p11 :p22 : : : pkk :m0 v M D p11 :p22 : : : pkk :m vi pi 6 j m0 ; m th ta chn
n D p11 :p22 : : : pkk ). Khi njax vi x ln. Ta c
(
A:ax C Bx  C .mod m/
x
A:a C Bx  C .mod M / ,
:
A:ax C Bx  C .mod m/
T .B; n/ D 1, suy ra tn ti s nguyn dng B 0 sao cho BB 0  1 .mod n/. Khi , vi x
ln tha mn x  B 0 C .mod n/ tha mn phng trnh
A:ax C Bx  C
150

.mod n/:

.1/

Tp ch Epsilon, S 06, 12/2015


t x D y n C B 0 C , ta chng minh tn ti s nguyn dng y ln tha mn
A:ax C Bx  C

.mod m/:

.2/

iu ny tng ng vi
0

A:anyCB C C B.ny C B 0 C /  C
0

, A:aB C :any C .Bn/y C .BB 0 C


y

c:e C by C t  0
B0C

.mod m/

C/  0

.mod m/

.mod m/

.3/

vi c D A:a ; e D a ; b D Bn; t D BB C C . R rng gcd.e; m/ D gcd.b; m/ D 1:


t f .y/ D ce y C by C t . By gi ta chng minh bi ton bng phng php quy np theo m.
Vi m D 1 th .3/ hin nhin ng. Gi s m > 1 v bi ton ng vi mi m0 < m. Gi p l c
nguyn t ln nht ca m v gi s m D p k :p1k1 :p2k2 : : : : prkr : t m0 D p k 1 :p1k1 :p2k2 : : : : prkr .
Khi , tn ti s nguyn dng y
.mod m0 /

f .y/  0

hay tn ti s nguyn l sao cho f .y/ D lm0 : Ta chn


0

y D y C z:.p

1/p

k 1

r
Y

1/piki

.pi

i D1

vi z l mt s nguyn dng. Ta chng minh tn ti z sao cho f .y 0 /  0 .mod m/: Ta c


!
r
Q
r
k
Y
.pi 1/pi i
yCz.p 1/p k 1

k
k
1
0
i D1
.pi 1/pi i C t
C b y C z.p 1/p
f y D ce
i D1
y

D ce :e

z.p 1/p k

r
Q
iD1

.pi 1/pi i

C by C t C zb.p

1/p

k 1

r
Y

.pi

1/piki

i D1

 .ce y C by C t/ C zb.p

1/p k

r
Y

.pi

1/piki

i D1

 lm0 C z:nm0 .p

1/

r
Y

.pi

1/ .mod m/

i D1

( trn ta s dng e

z.p 1/p k

r
Q
iD1

.pi 1/pi i

 1 .mod m/ theo nh l Eulers).

Do , ta chng minh
lm0 C zbm0 .p

1/

r
Y

.pi

1/  0

.mod m/

i D1

c nghim nguyn dng z, tc l phng trnh


l C zb.p

1/

r
Y

.pi

i D1

151

1/  0

.mod p/

Tp ch Epsilon, S 06, 12/2015


c nghim nguyn dng z.
iu ny lun ng do y l phng trnh ng d tuyn tnh v gcd.b.p

1/

r
Y

.pi

1/; p/ D

i D1

1. Do vy bc quy np c hon tt.


trn ta chng minh c tn ti s nguyn dng y sao cho f .y/  0 .mod m/ hay tn ti
x tha mn (2). Chng ta cn chng minh c th chn y ln ty tha mn (3). iu ny lun
thc hin c v nu y tha f .y/  0 .mod m/ th y C :m:.m/ vi  2 ZC cng tha mn.
Vy nh l c chng minh.
Cch 2. Gi s gcd.B; M / D 1 v C l mt s nguyn bt k. Ta c dy Aa1 ; Aa2 ; : : : l dy
tun hon xt theo modulo M . Gi T l chu k ca dy, tc l T l s nguyn dng nh nht
sao cho
A:axCkT  A:ax .mod M /
vi mi x ln.
t d D gcd.T; M /. Trc ht ta chng minh d < M vi M > 1.
V d jM nn d  M . Gi s d D M , khi M jT nn M  T . Tuy nhin, T l chu k ca dy
Aai khi xt theo modulo M nn T  M , do T D M . Suy ra tn ti s nguyn dng X sao
cho A:aX  0 .mod M / v nh vy A:ax  0 .mod M / vi mi x  X. Dn n T D 1, nn
M D 1.
Tip theo ta chng minh bi ton bng phng php quy np theo M .
Vi M D 1 l trng hp tm thng. Ta xt M > 1 v gi s bi ton ng vi mi m < M ,
dn n bi ton cng ng vi d . Tc l, tn ti s nguyn dng x sao cho
A:ax C Bx  C

.mod d /

hay A:ax C Bx D C C ld vi l l s nguyn. t x 0 D x C kT vi k l s nguyn dng. Ta c


0

A:ax C Bx 0 D A:axCkT C B.x C kT /


 .A:ax C Bx/ C kBT
D C C ld C kBT .mod M /:
Tip theo ta chng minh phng trnh ng d
ld C kBT  0

.mod M /

c nghim nguyn dng k. Chia hai v ca phng trnh ng d trn cho d ta c phng
trnh
T
M
l C kB  0 .mod
/
d
d


T M
Phng trnh ny lun c nghim nguyn dng k v gcd B ;
D 1:
d d
Vy bi ton c chng minh.

152

BI TON HAY LI GII P


Trn Nam Dng
(Trng i hc Khoa hc T nhin, HQG - TP.HCM)
LI GII THIU
Chuyn mc ny c ly cm hng t bi vit ca thy Nguyn Duy Lin
Epsilon s 3 v bi ton s 6 trong k thi IMO 2001 vi 5 cch gii khc nhau. Mc
ny s dnh vit v nhng bi ton hay, li gii p v nhng cu chuyn th v
xung quan nhng bi ton v li gii .
Tn ca chuyn mc c mn t tn ca mt nhm nhng ngi yu ton trn
Facebook do anh Nguyn Vn Li sng lp Bi ton hay Li gii p am m
ton hc. Chuyn mc ghi nhn cc c ca bn c v s chn ng mi k 1; 2
bi ton.
S ny chng ti s gii thiu vi bn c v bi ton s 1 trong thi ton quc
t nm 1993.
Bi ton 1. (IMO 1993, bi ton 1) Cho f .x/ D x n C 5x n 1 C 3 vi n > 1: Chng minh rng
f .x/ khng th phn tch thnh tch ca hai a thc khc hng vi h s nguyn.
Bi ton ny tuy c t v tr s 1; tc l c coi l bi d ca ngy th nht (v cng l ca
c k thi) nhng thc t th im trung bnh ca bi ny thp hn nhiu so vi cc bi s 4; s 5
v xp x vi bi 2 v bi 6: Gn mt na th sinh khng lm c g b khong 20% th sinh
khc ch c 1 im (chc l do tng phn chng). Chnh tnh hung bi 1 kh l gii v
sao im chun nm 1993 rt thp .11 cho huy chng ng, 20 im cho huy chng bc v 30
im cho huy chng vng).
Trong chng trnh chuyn ton ph thng, khng c nhiu cng c chng minh tnh bt kh
quy ca mt a thc vi h s nguyn. Thc t chng ta ch c mt tiu chun bt kh quy duy
nht (iu kin ) l tiu chun Eisentein:
Tiu chun Eisenstein. Cho a thc P .x/ D an x n C an 1 xn 1 C    C a1 x C a0 2 Zx: Gi
s tn ti s nguyn t p sao cho cc iu kin sau y ng thi xy ra:
i) an khng chia ht cho p:
ii) a0 ; a1 ; : : : ; an

chia ht cho p:

iii) a0 khng chia ht cho p 2 :


Khi P .x/ bt kh quy.
Ngoi ra, cn c mt cch tip cn dng n nghim ca a thc, vi v d kinh in sau:
Bi ton 2. Chng minh rng a thc x n C 4 kh quy trn Zx khi v ch khi n chia ht cho 4:
153

Tp ch Epsilon, S 06, 12/2015


Li gii ca bi ton IMO 1993; rt th v, da trn vic phn tch k cch chng minh tiu
chun Eisenstein v li gii bi ton 2 ni trn. V c tiu chun Eisenstein v bi ton 2 u rt
ni ting, nn chng ti nhc li chng minh v li gii m chuyn sang lun phn li gii cho
bi ton 1:
Cch 1. Ta khng th p dng trc tip tiu chun Eisenstein cho p D 3 v an 1 D 5 khng
chia ht cho 3 (v nu p dng c nh vy th khng phi l thi ton quc t). Tuy nhin,
ta c th dng t tng trong chng minh tiu chun Eisenstein gii quyt bi ton.
C th, lp li php chng minh tiu chun Eisenstein, ta c th chng minh c tiu chun
Eisentein m rng nh sau:
Cho a thc P .x/ D an x n C an 1 xn 1 C    C a1 x C a0 2 Zx: Gi s tn ti s nguyn t p
v s nguyn dng k tho mn ng thi cc iu kin sau:
1) an khng chia ht cho p:
2) a0 chia ht cho p nhng khng chia ht cho p 2 :
3) a1 ; a2 ; : : : ; an

chia ht cho p:

Khi , nu P .x/ D Q.x/    S.x/ vi Q.x/; S.x/ l cc a thc vi h s nguyn th mt trong


hai a thc Q.x/; S.x/ c bc nh hn k:
y theo iu kin bi th k D 2: Do nu f .x/ kh quy v f .x/ D Q.x/    S.x/ th p
dng tiu chun trn, ta suy ra mt trong hai a thc Q.x/ v S.x/ c bc 1: T suy ra f .x/
c nghim hu t. Th nhng ta bit mi nghim hu t ca f .x/ D x n C 5x n 1 C 3 u phi
l s nguyn (xem bi tp 2 mc 1/; m khi x nguyn th x n C 5x n 1 C 3 lun l s l, khng
th bng 0: Mu thun. Vy iu gi s l sai, suy ra f .x/ bt kh quy.
Cch 2. Gi s x1 ; : : : ; xn l cc nghim phc ca a thc x n C 5x n 1 C 3: Khi ta c
P .x/ D x n C 5x n 1 C 3 D .x x1 /.x x2 /    .x xn /: Ta c vi mi i D 1; 2; : : : ; n
xin 1 .xi C 5/ D

3; suy ra jxi jn 1 jxi C 5j D 3:

.1/

Gi s P .x/ D Q.x/  S.x/ vi Q.x/; S.x/ l cc a thc vi h s nguyn th khng mt tnh


tng qut, ta c th gi s
Q.x/ D .x

x1 /    .x

xk /; S.x/ D .x

xkC1 /    .x

xn /:

Ta c x1 x2    xk v xkC1    xn l cc s nguyn c tch bng 3: Khng mt tnh tng qut, gi


s
jx1    xk j D 3 jxkC1    xk j D 1:
.2/:
Trong .1/ cho i chy t 1 n k ri nhn li, ch n .2/ ta c
3n 1 j.x1 C 5/    .xk C 5/j D 3k :
Biu thc trong du j j bng jQ. 5/j do l s nguyn dng, suy ra k > n 1: Nh vy S.x/
l a thc bc nht. V n y ta li p dng l lun nh phn cui ca cc gii 1:
154

Tp ch Epsilon, S 06, 12/2015


Vi cch gii p dng cch 2, ta c th lm mnh kt qu sau, l bi ton xut hin trong
k thi VMO 2014 (v cng gy kh d cho nhiu th sinh).
Bi ton 3. (VMO 2014) Cho a thc P .x/ D .x 2 7x C 6/2n C 13; trong n l mt s
nguyn dng. Chng minh rng P .x/ khng th biu din di dng tch ca n C 1 a thc
khc hng c h s nguyn.
Li gii chnh thc ca bi ton da vo cc sau:
1) P .x/ khng c nghim thc, do cc tha s ca P .x/ c bc chn.
2) P .x/ c bc 4n; do nu P .x/ phn tch thnh tch ca n C 1 a thc khc hng vi h
s nguyn th mt trong cc a thc phi c bc 2:
Tip theo dng n vic xr cc tnh cht s hc ca cc h s v suy ra mu thun.
Tuy nhin, nu xem xt k cch chng minh v dng tng ca cch gii th hai th ta c th
d dng chng minh c kt qu mnh hn nh sau.
Bi ton 4. Cho a thc P .x/ D .x 2 7x C 6/2n C 13; trong n l mt s nguyn dng.
Chng minh rng P .x/ khng th biu din di dng tch ca 2 a thc khc hng c h s
nguyn.
Bi tp ny chng ti xin dnh cho bn c. Hy c k li cch 2 ca li gii trn v ch thm
mt chi tit l nu Q.x/ l mt a thc vi h s nguyn th Q.1/ Q.6/ chia ht cho 5:
Nh vy bi ton 1 ca IMO 1993 pht trin mt ti kinh in v t ra mt hng pht
trin th v. Cc vn v a thc bt kh quy lun th v v kh. V khi chng ta vt qua c
mt kh khn, kho tng phng php ca chng ta li c trang b mt v kh mi. C l l
iu khin cc thi IMO lun c ch i, n nhn v bnh phm mt cch nhit tnh.

155

Tp ch Epsilon, S 06, 12/2015

156

CC VN C IN V HIN I
Trn Nam Dng
(Trng i hc Khoa hc T nhin, HQG - TP.HCM)
LI GII THIU
Chuyn mc ny dnh cho cc vn c in v hin i c trnh by di
dng cc bi ton xu chui. c th l chui cc bi gii bi ton ng chu,
1
2
1
chng minh ng thc Euler k diu 1 C 2 C 2 C    D
; mt chui bi ton
2
3
6
vn tr ... Cch trnh by xut pht t nhng vn n gin, d hiu, nhng khi
nim mi s c nh ngha lun trong bi c th c tng i c lp. V mi
mt chui bi s nu ra nhng vn nht nh, c th l gii quyt mt bi ton
kinh in hay nu ra nhng gi thuyt mi, nhng vn mi. Li gii v tho lun
v cc bi ton s c ng s N C 3:
Trong s ny, chng ti tip tc ng phn cui tm tt li gii cc bi ton
ng s 2 v bt u ng li gii tm tt cc bi ton ng s 3:

Bt ng thc Schapiro (tip theo k trc)


Ta ang ni v bt ng thc sau vi x1 ; x2 ; : : : ; xn l cc s thc dng
x1
x2
xn 1
xn
n
C
C  C
C
 :
x2 C x3
x3 C x4
xn C x1
x1 C x2
2

.1/

Khi cuc thi din ra, cc th sinh gii quyt kh tt cc bi ton ngh mc 1; v Ban t
chc quyt nh b sung hai bi ton sau, thc cht l chng minh mt kt qu rt mnh ca
V.Drinfeld v bt ng thc Schapiro (kt qu mnh nht cho cc nh gi dng 1:9/:
1.10. a) Vimi s nguyn
dng n tn ti s qn > 1; sao cho vi mi s thc x1 ; x2 ; : : : ; xn

1
thuc on
; qn bt ng thc .1/ ng.
qn


1
b)* Tn ti hay khng s q > 1; sao cho vi mi s nguyn dng n v vi mi xi 2
; q bt
q
ng thc .1/ ng.
Chng minh. a) Kt qu ny thuc v V.Cirtoaje [9]. cho gn, ta t yk D xk C xkC1 : Khi
bt ng thc c vit li thnh
x2
xn
n
x1
C
C  C
 :
y2
y3
y1
2
Ta bin i n mt cht v dng
n
X
2q 2 xk
n

kD1

ykC1

ykC1
157

 n.qn2

1/;

Tp ch Epsilon, S 06, 12/2015


Trong qn l tham s s c chng ta chn sau, sao cho tt c cc bt ng thc m ta xt
ng. V
2qn2 xk ykC1 D .qn2 xk xkC1 / C .qn2 xk xkC2 /  0;
nn theo bt ng thc Cauchy-Schwarz, ta c
n
P


n
X
2q 2 xk

ykC1

ykC1

kD1

2

2qn2 xk

ykC1

kD1
n
P

.2qn2 xk

ykC1 /ykC1

kD1

Nh vy, ta ch cn chng minh rng


" n
#2
X
2
2
A D
.2qn xk ykC1 /  n.qn2

1/

kD1

n
X
kD1

yk D 2

n
X

.2qn2 xk

ykC1 /ykC1 D n.qn2

1/B:

kD1

n
X

xk ; ta c ng thc

kD1

AD

.qn2

1/

n
X

yk ;

kD1

BD

2qn2

n
X

n
X

xk ykC1

kD1

yk2

D2qn2

kD1

n
X

.qn2

yk ykC1

C 1/

kD1

yk2 :

kD1

T ta ch cn cn chng minh rng


!2
"
n
n
X
X
2
2
.qn 1/
yk  n 2qn
yk ykC1
kD1

n
X

.qn2

C 1/

kD1

n
X

#
yk2

.2/

kD1

Bin i v tri s dng ng thc


n
X

!2
Dn

yk

kD1

n
X

yk2

kD1

yk /2 ;

.yi

i <k

bt ng thc .2/ c th vit di dng


n

n
X

.yk


ykC1 /  1
2

kD1


1 X
.yi
qn2

yk /2 :

i <k

Theo bt ng thc Cauchy-Schwarz


n
X

.yk

ykC1 /2 

k 1
X

.yj

yj C1 /2 

j Di

kD1

1
k

.yi

yk /2 
158

1
k

j
1

2
k 1
X
4
.yj
j Di

.yi

yk /2 :

3
yj C1 /5

Tp ch Epsilon, S 06, 12/2015


Suy ra
n

n.n

1/ X
2

ykC1 /2 

.yk

kD1

1
n

X
1

i <k

1
2
n 1
D
;
tc
l
q
D
p
n
qn2
.n 1/2
n2 2n

v ta c th chn 1

yk /2 ;

.yi

> 1:

Lu . Khi n tng th cc gi tr qn tm c dn n 1:
b) Hin cha c cu tr li.
1.11. Gi S D f .x1 ; x2 ; : : : ; xn / l v tri ca bt ng thc Schapiro. K hiu a1 ; a2 ; : : : ; an
x2 x3
xn x1
l cc s ;
; :::;
;
xp theo th t tng dn.
x1 x2
xn 1 xn
a) Chng minh rng
1
1
1
C
C  C
:
a1 .1 C an / a2 .1 C an 1 /
an .1 C a1 /

S

b) t bk D

<

1
ak anC1
ak anC1

nu ak anC1

1

nu ak anC1

<1

2
p
ak anC1
k C

; chng minh rng


k

2S > b1 C b2 C    C bn :
c) Gi g l hm li ln nht khng vt qu cc hm e

v 2.e x C e 2 / 1 : Chng minh rng

2S > gln.a1 an / C gln.a2 an 1 // C    C gln.an a1 / > n  g.0/:


d) Chng minh rng vi mi  >
sao cho S 6 n:
Chng minh. a) t ki D

g.0/
tn ti s nguyn dng n v cc s dng x1 ; x2 ; : : : ; xn
2

xiC1
; khi
xi
1

SD

C  C

k1 .k2 C 1/ k2 .k3 C 1/
kn .k1 C 1/
1
1
1

C
C  C
:
a1 .an C 1/ a2 .an 1 C 1/
an .a1 C 1/

b) Bt ng thc ng bi v
a a

1
ai .anC1

C 1/

anC1

nC1 i
1 C .1Caii /.1Ca
1
nC1
D
ai anC1 i
i .ai C 1/

trong bt ng thc cui cng ng do .1 C ai /.1 C anC1 i /  .1 C

i/

 bi ;

c) Bt ng thc
2S > gln.a1 an / C gln.a2 an 1 // C    C gln.an a1 /;
159

ai anC1 i / :

Tp ch Epsilon, S 06, 12/2015


x

ng v g.x/ nh hn c e

v 2.e x C e 2 / 1 :

Cn bt ng thc
gln.a1 an / C gln.a2 an 1 // C    C gln.an a1 / > n  g.0/;
ng theo bt ng thc Jensen, v g l hm li.
d) Xem [2].

3.3. Cc bt ng thc hu dng lin quan


Chng minh cc bt ng thc di y vi iu kin tt c cc s xk dng. Hy kim tra rng
cc hng s c in m khng th thay th bng cc s ln hn (vi mi n/:
2.1. Bt ng thc Mordell.
a) Vi mi s khng m x1 ; x2 ; : : : ; xn ta c bt ng thc
!2
n
n
nn o X
X
xk  min ; 3 
xk .xkC1 C xkC2 /:
2
kD1

kD1

b) Hy xc nh vi nhng gi tr khng m no ca x1 ; x2 ; : : : ; xn bt ng thc Mordell


tr thnh ng thc.
Chng minh. Bt ng thc ny c chng minh trong [20].
a) Vi n D 3 v n D 5; sau khi b du ngoc ta thu c bt ng thc
X
.n 1/.a1 C a2 C    C an /2  2n
ai ak :

.3/

i <k

Bt ng thc ny d dng chng minh da vo bt ng thc Cauchy-Schwarz cho cc s


.a1 ; a2 ; : : : ; an / v .1; 1; : : : ; 1/
n.a12 C a22 C    C an2 /  .a1 C a2 C    C an /2 :
Tip theo ta ch rng
n.a1 C a2 C    C an /2 D n.a12 C a22 C    C an2 / C 2n

ai ak

i <k
2

 .a1 C a2 C    C an / C 2n

ai ak ;

i <k

t suy ra .3/: Vi n D 4 ta cn kim tra bt ng thc


.x1 C x2 C x3 C x4 /2  2x1 x2 C 2x2 x3 C 2x3 x4 C 2x4 x1 C 4x1 x3 C 4x2 x4 :
Khai trin v rt gn cc s hng ng dng, ta thu c bt ng thc hin nhin
x12 C x22 C x32 C x42  2x1 x3 C 2x2 x4 :
160

Tp ch Epsilon, S 06, 12/2015


By gi ta chuyn sang trng hp n > 6: Hon v vng quanh cc s nu cn, ta c th gi s
rng x3 > x1 v x3 > x2 v d lm cho x3 ln nht. Vi r D 1; 2 hoc 3 ta k hiu ar l tng
tt c cc s xk sao cho k  r .mod 3/; k 6 n: Khi x1 C x2 C    C xn D a1 C a2 C a3 v
theo bt ng thc .3/ ta suy ra
.x1 C x2 C    C xn /2 D .a1 C a2 C a3 /2  3.a1 a2 C a2 a3 C a3 a1 /
X
D3
xi xk :
i k0

t A D

xi xk v B D

i k0 .mod 3/

n
X

.mod 3/

xk .xkC1 C xkC2 / v kim tra rng A > B:

kD1

Tht vy, vi n  0 .mod 3/ tt c cc s hng ca tng B c trong tng A: Vi n  0 .mod 3/


trong tng A so vi tng B ch thiu s hng xn x1 : Nhng s hng ny li khng vt qu xn x3
l s hng c trong A nhng khng c trong B; v cui cng, vi n  2 .mod 3/ th trong tng
A so vi tng B ch thiu cc s hng xn 1 x1 v xn x2 : Nhng nhng s hng ny tng ng li
nh hn cc s hng xn 1 x3 v xn x3 :
Nh vy trong mi trng hp A > B: Do
2

.x1 C x2 C    C xn / D .a1 C a2 C a3 /  3A  3B D 3

n
X

xk .xkC1 C xkC2 /:

kD1

Tnh tt nht ca hng s min

nn

o
; 3 l hin nhin. Vi n  6 ta c th ly tt c cc bin bng 1

2
cn vi n  6; ta ly x1 D x2 D x3 D 1 v x4 D x5 D    D xn D 0:

b) Trng hp n < 6 hin nhin. Vi n D 6 du bng xy ra khi x1 C x4 D x2 C x5 D x3 C x6 :


Vi n > 6 du bng xy ra trn tp hp c dng .t; 1; 1; 1 t; 0; 0; : : : ; 0/ trong t 2 0; 1
v cc hon v vng quanh ca chng.
2.2. Vi mi s khng m x1 ; x2 ; : : : ; xn hy chng minh bt ng thc
!2

 n
n
X
n 8 X

xk  min ;
xk .xkC1 C xkC2 C xkC3 /:
3 3
kD1

kD1

Chng minh. Bt ng thc ny c chng minh trong bi bo [20]. Ta bt u t n 6 8:


Vi n D 4 v n D 7 y l trng hp ring ca bt ng thc .3/:
Vi n D 5 bt ng thc tng ng vi bt ng thc
X
.xk 2xkC2 C xkC4 /2  0:
Vi n D 6 sau khi b du ngoc v rt gn ta thu c bt ng thc hin nhin
x12 C x22 C    C x62  2x1 x4 C 2x2 x5 C 2x3 x6 :
chng minh bt ng thc trong trng hp n D 8; ta ph ngoc trong h qu ca bt ng
thc Cauchy-Schwarz
4.x12 C x22 C x32 C x42 /  .x1 C x2 C x3 C x4 /2 ;
161

Tp ch Epsilon, S 06, 12/2015


c
3.x12 C x22 C x32 C x42 /  2.x1 x2 C x1 x3 C x1 x4 C x2 x3 C x2 x4 C x3 x4 /:
Suy ra
3.x1 C x2 C x3 C x4 /2  8.x1 x2 C x1 x3 C x1 x4 C x2 x3 C x2 x4 C x3 x4 /;

.4/

chnh l bt ng thc cn chng minh vi n D 8: chng minh trng hp n > 8; ta cng


gi s x4 ln nht ri t ar ; r D 0; 1; 2; 3 l tng cc xk vi k  r .mod 4/; k 6 n: Khi
x1 C x2 C    C xn D a1 C a2 C a3 C a4 v theo .3/ ta c
3.x1 C x2 C    x4 /2 D .a1 C a2 C a3 C a4 /2  8.a1 a2 C a1 a3 C a1 a4 C a2 a3 C a2 a4 C a3 a4 /
X
8
xi xk :
i k0

By gi t A D

.mod 4/

xi xk v B D

i k0 .mod 4/

n
X

xk .xkC1 C xkC2 C xkC3 /; bng l lun hon

kD1

ton tng t nh trong bi 2:1 ta suy ra A > B v t suy ra iu phi chng minh.
2.3. a) Vi n 6 8 hy chng minh bt ng thc
x1
x2
xn 1
xn
n
C
C  C
C
 :
x2 C x3 C x4
x3 C x4 C x5
xn C x1 C x2
x1 C x2 C x3
3
b)* Bt ng thc trn cn ng vi gi tr no khc ca n hay khng ?
Chng minh. a) Bt ng thc ny c chng minh trong bi bo [11], s dng php bin i
Fourier. Ta a ra cch chng minh s cp. Theo bt ng thc Cauchy-Schwarz ta c
 n
2
P
xk
n
X
xk
n
kD1
 ;
 n
P
xkC1 C xkC2 C xkC3
3
kD1
xk .xkC1 C xkC2 C xkC3 /
kD1

trong bt ng thc cui cng suy ra t 2:2:


b) Cha r cu tr li.
2.4. Chng minh rng vi mi n > 4; th
.x1 C x2 C    C xn /2 > 4.x1 x2 C x2 x3 C    C xn 1 xn C xn x1 /:
Chng minh. Bi ton ca B.Ginzburg [1, bi ton 187], c xut cho thi ton Lin X
nm 1972: Bng cch hon v vng quanh cc s, c th gi s x1 6 x2 : t
S D x1 C x2 C    C xn ; S1 D x1 C x3 C    ; S2 D x2 C x4 C   
Khi

S12

S22

.S1 C S2 /2
S2

D
; suy ra
2
2
S2
 S2
2

S12

S22 D 2

X
i k0

162

.mod 2/

xi xk :

.5/

Tp ch Epsilon, S 06, 12/2015


Nu n chn th tng cui cng cha tt c cc s hng dng xk xkC1 ; cn nu n l th tng ny
thiu s hng xn x1 nhng li c s hng xn x2 : Nh vy
S2
 2.x1 x2 C x2 x3 C    C xn x1 /:
2
Ta c iu phi chng minh.
2.5. Chng minh rng
n
X
kD1

X xkC1
xk

:
xkC1 C xkC2
xk C xkC1
kD1

Chng minh. Xem li gii bi ton 1:3:


2.6. Vi mi n  4; th
x1
x2
xn 1
xn
C
C  C
C
 2:
xn C x2
x1 C x3
xn 2 C xn
xn 1 C x1
Chng minh. y l bi ton ca A.Prokopiev, cuc thi gia cc thnh ph 1981
cng b trong tp ch Kvant, 1982; s 6; bi ton M 749:

1982; 4;

Ta gi v tri ca bt ng thc l Ln : Vi n D 4; ta c
L4 D

x1 C x3
x2 C x4
1
C
D a C  2:
x2 C x4
x1 C x3
a

Vi n > 4 ta l lun bng quy np. V bt ng thc l hon v vng quanh nn ta c th gi s


xnC1 nh nht trong tt c cc s. Khi b i s hng cui cng ca LnC1 ; sau gim hai s
hng khc, ta c
LnC1 

x1
C  C
xnC1 C x2
xn

xn
x1
xn

C  C
D Ln :
xn C x2
xn 1 C x1
1 C xnC1

chng minh hng s v phi l tt nht, ta chn x1 D x2 D 1; x3 D t; x4 D t 2 ; : : : ;


xn D t n 2 ri cho t ! 0:
S dng bt ng thc Cauchy-Schwarz nh trong li gii bi ton di y, bn c s c th
tm ra mt li gii khc bng cch a v bt ng thc 2:4:
2.7. Chng minh rng vi mi n  4; th
x1 C x2
x2 C x3
xn
C
C  C
x1 C x3
x2 C x4
xn

C xn
xn C x1
C
 4:
xn C x2
1 C x1

Chng minh. Chng ti ly bi ny t bi bo [10].t S D x1 C x2 C    C xn : p dng


xk C xkC1
bt ng thc Cauchy-Schwarz cho b s
v f.xk C xkC1 /.xk C xkC2 /g ta c
xk C xkC2
x1 C x2
x2 C x3
xn
C
C  C
x1 C x3
x2 C x4
xn

C xn
xn C x1
4.x1 C x2 C    C xn /2
C
 n
:
P
xn C x2
1 C x1
.xk C xkC1 /.xk C xkC2 /

kD1

163

Tp ch Epsilon, S 06, 12/2015


Nh vy, chng minh bt ng thc cho, ta ch cn xc lp bt ng thc
2

S 

n
X

.xk C xkC1 /.xk C xkC2 / D

kD1

n
X
kD1

xk2

C2

n
X

xk xkC1 C

kD1

n
X

xk xkC2 ;

kD1

Bt ng thc ny ng bng cch ph ngoc v phi, v vi n > 4 th cc s hng xk xkC1 v


xk xkC2 vi k D 1; 2; : : : ; n l khc nhau.
Ta s chng t hng s 4 khng th thay th bng s ln hn. Tht vy, t xk D ak 1 vi
k D 1; 2; : : : ; n 1 v xn D an 2 : Khi a ! 1 th n 3 s hng u dn n 0; cn cc s
hng cui dn n 1; 2; 1:
2.8. Chng minh rng vi mi n > 6; th
x1
x2
xn 1
xn
C
C  C
C
 3:
xn C x3
x1 C x4
xn 2 C x1
xn 1 C x2
Chng minh. Chng ti ly bi ton ny t bi bo [6]. Bi ny cng chng minh bng cch
p dng bt ng thc Cauchy -Schwarz tng t bi 2:7; sau p dng bt ng thc Mordell
(bi ton 2:1/:
Ta chng t rng hng s 3 khng th thay th bng s ln hn. Tht vy, t xk D ak 1 vi
k D 1; 2; : : : ; n 2 v xn 1 D xn D 1: Khi a ! 0 th s hng u v s hng cui dn n 1;
cn cc s hng khc dn n 0:
2.9. Chng minh rng nu n  6; th
x2 C x3
x3 C x4
xn C x1
x1 C x2
C
C  C
C
 6:
x1 C x4
x2 C x5
xn 1 C x2
xn C x3
Chng minh. Chng ti ly bi ton ny t bi bo [5]. Ta ch cn cng hai bt ng thc
2:8 li (bt ng thc 2:8 v bt ng thc cho b s o ngc).
Ta chng t rng hng s 6 khng th thay th bng s ln hn. Tht vy, t xk D ak 1 vi
k D 1; 2; : : : ; n 2 v xn 1 D xn D 1: Khi a ! 0 th bn s hng cui cng bng 1; 2; 2; 1
tng ng, cn cc s hng khc dn n 0:
2.10. Chng minh rng
x1 C x2
x2 C x3
x2004 C x1
C
C  C
 6:
x1 C x4
x2 C x5
x2004 C x3
Chng minh. Trong bi bo [19], khng nh ny c nu ra di dng gi thuyt. Chng minh
di y thuc v cc th sinh ca cuc thi P.Milosevic v M.Bukic.
Bt ng thc cn chng minh l tng ca hai bt ng thc vi n D 2004 - bt ng thc t bi
2:8 v bt ng thc
x1
x2
xn
C
C  C
 3:
x1 C x4
x2 C x5
xn C x3
Ta s chng minh bt ng thc ny. Vi n D 3m n l tng ca ba bt ng thc
x1
x4
xn 2
C
C  C
 1;
x1 C x4
x4 C x7
xn 2 C x1
164

Tp ch Epsilon, S 06, 12/2015


x2
x5
xn 1
C
C  C
 1;
x2 C x5
x5 C x8
xn 1 C x2
x3
x6
xn
C
C  C
 1:
x3 C x6
x6 C x9
xn C x3
Mi bt ng thc ny u c dng
1
1
1
C
C  C
 1;
1 C a1
1 C a2
1 C am
trong vi a1 a2    am D 1:
Bt ng thc ny c chng minh bng quy np theo m: C s quy np m D 2; th
1
1
1
1
D 1:
C
D
C
1 C a1
1 C a2
1 C a1
1 C a11
thc hin bc quy np, ta ch rng
1
1
1
C

:
1Cb
1Cc
1 C bc
Ta chng t rng hng s 6 khng th thay c bng mt hng s ln hn. Ly cc s
x1 D x2 D x3 D 1; xk D an kC1 vi k D 4; 5; : : : ; n th khi a ! 0; s hng th nht v th
hai dn n 2; s hng th ba v s hng cui cng dn n 1; cc s hng cn li dn n 0:
2.11. Chng minh rng
x1
x2
xn 1
xn
C
C  C
C
 4;
xn C x4
x1 C x5
xn 2 C x2
xn 1 C x3
trong n l s chn ln hn 7:
Chng minh. y l chng minh ca A.Khrabrov. t S D x1 C x2 C    C
 xn v T D
X
xk
v fxk .xk 1 C xkC3 /g
xi xk : Theo bt ng thc Cauchy-Schwarz cho cc b
xk 1 C xkC3
i k0 .mod 2/

ta thy v tri ca bt ng thc trn s khng nh hn


.x1 C x2 C    C xn /2
:
.x1 x2 C x2 x3 C    C xn x1 / C .x1 x4 C x2 x5 C    C xn x3 /
Nh vy ch cn chng minh
S 2  4.x1 x2 C x2 x3 C    C xn x1 / C 4.x1 x4 C x2 x5 C    C xn x3 /:
Trong li gii bi 2:4 ta chng minh c S 2 > 4T: Nh vy, ch cn chng minh rng
T  .x1 x2 C x2 x3 C    C xn x1 / C .x1 x4 C x2 x5 C    C xn x3 /:

.6/

V n chn nn mi s hng tng bn phi u cha trong tng bn tri.


Ta chng t hng s 4 khng th thay bng hng s ln hn. Ta xt b s xk D ak 1 vi
k D 1; 2; : : : ; n 3 v xn 2 D xn 1 D xn D 1: Khi a ! 0 th s hng u tin v 3 s hng
cui cng dn n 1; cc s hng cn li dn n 0:
165

Tp ch Epsilon, S 06, 12/2015


2.12. Chng minh rng
n
X
kD1

2
xkC1



xk2
nC1

:
2
2
xkC1 xkC2 C xkC2

Chng minh. Chng ti ly bi ny t bi bo [14].


Ch rng a2

ab C b 2 6 maxfa; bg2 :

Gi s xi1 l s ln nht trong cc s x1 ; x2 ; : : : ; xn ; xi2 l s ln nht trong hai s tip sau


xi1 ; xi3 l s ln nht trong hai s tip xi2 ; : : : Ta c xy dng dy s ny cho n khi gp
n
c ch s k sao cho s ln nht trong hai s tip sau xik l xi1 : R rng k  v v vy
2


nC1
k
:
2
n
X
kD1

2
xkC1

k
X
xi2j
xk2

 k:
2
2
xkC1 xkC2 C xkC2
x
i
j C1
j D1

nh gi cui cng ta dng bt ng thc AM-GM.




nC1
Hng s
khng th thay bng hng s ln hn v nu ta thay xk D 1 vi k l v xk D 0
2


nC1
vi k chn th v tri s bng
:
2

Pht trin t duy t hp qua cc bi ton m


m s on thng, s tam gic, s hnh vung khng phi cho vui m thc ra hnh thnh cc
t duy t hp rt quan trng: Bit chia trng hp, bit phn loi, bit d on quy lut, bit tng
qut ha, bit loi tr ... Di y l chui cc bi ton m dnh cho cc hc sinh lp ln ca
tiu hc v cc lp Trung hc c s. Vi nhng t duy t hp c hnh thnh mc trc
gic, khi c trang b cc cng c t hp, hc sinh s d dng hiu nhanh, hiu su cc khi
nim v c kh nng p dng.
1. C 5 im nm trn mt ng thng. Hi c bao nhiu im c to thnh?

p s. (10 im) Ta c th m theo kiu lit k, hoc dng tng hp chp 2 ca 5 phn t.
2. C 5 im nm trn mt ng thng v 1 im nm ngoi ng thng . Hi c bao nhiu
tam gic c to thnh?
166

Tp ch Epsilon, S 06, 12/2015

p s. (10 tam gic) Mt tam gic gm 1 nh v 1 cnh y. Lin h vi bi ton 1:


3. C 4 im trn 1 ng thng v 5 im trn mt ng thng khc. Hi c bao nhiu tam
gic c nh ti cc im ?

p s. (70 tam gic) C 2 loi tam gic: Loi 1 l nh trn, y di v loi 2 l y


di, nh trn. Vi loi 1 ta c 4 cch chn nh v 10 cch chn y (lin h bi 2/: Vi loi
2 ta c 5 cch chn nh v 6 cch chn y. Vy c 4  10 C 5  6 D 70 tam gic. Trong bi
ton ny ta lm quen vi 2 quy tc m quan trng l quy tc cng (phn trng hp) v quy
tc nhn (phn cng on).
4. Trn 3 cnh ca mt tam gic c tng ng 3; 4; 5 im. Hi c bao nhiu tam gic c nh
ti cc im ?

167

Tp ch Epsilon, S 06, 12/2015


p s. .114 tam gic) C 4 loi tam gic: Loi 1; 2; 3 l c cnh nm trn 1 trong 3 cnh ca
tam gic v nh nm trn 1 trong 2 cnh cn li. Loi 4 l loi c 3 nh nm trn 3 cnh. Ta
tnh th loi 1 l loi c cnh nm trn cnh bn bn tri. C 6 cch chn cnh ny v 6 cch
chn nh cn li, suy ra c 6  6 D 36 tam gic loi 1: L lun tng t, vi tam gic loi 2 c
cnh nm trn cnh bn bn phi, s c 3 cch chn cnh v 7 cch chn nh v loi 3 cng th.
Cui cng, c 4  3  3 D 36 cch chn tam gic loi 4: Vy p s l
6  6 C 3  7 C 3  7 C 4  3  3 D 114:
3
Ta cng th m bng cch ly C10
(l s b 3 nh ly t 10 nh) tr i s cc tam gic suy
bin (l 4 C 1 C 1 D 6/ ta c kt qu l 120 6 D 114:

5. C 9 im to thnh 1 li vung 3  3: Hi c bao nhiu tam gic c nh ti cc im


cho?

p s. (76 tam gic)


Cch 1. Tng t nh bi 4; ta chia cc tam gic lm 2 loi. Loi 1 l tam gic c 2 nh nm trn
1 ng ngang v nh kia nm mt trong hai ng ngang cn li. Loi 2 l loi c 3 nh nm
trn 3 ng ngang. C 3 cch chn ng ngang cha 2 nh. Vi mi ng ngang chn, c
3 cch chn 2 nh. C 6 cch chn nh cn li. Vy s tam gic loi 1 bng 3  3  6 D 54:
Vi loi 2; r rng s cch chn ra mi hng ngang 1 im bng 3  3  3 D 27: Tuy nhin,
trong s nhng cch chn ny c nhng cch chn to ra tam gic suy bin. D thy c 5 tam
gic suy bin nh vy. Vy s tam gic loi 2 l 27 5 D 22: V tng s tam gic cn tm bng
54 D 22 D 76:
Cch 2. n gin l C93

8 D 84

8 D 76 (bn c hy th gii thch li gii ny).

6. C bao nhiu tam gic trong hnh sau?


168

Tp ch Epsilon, S 06, 12/2015

p s. (90 tam gic) K hiu hai nh y l B; C.B bn tri, C bn phi). Ta thy 1 tam
gic bt k phi nhn B hoc C lm nh.
Ta m s tam gic nhn B lm nh. Khi hai nh cn li phi nm trn 1 trong 4 ng
thng k t C: C 4 cch chn ng thng . Trn mi ng thng c 6 nh, do c 15
cch chn 2 nh. Suy ra s tam gic nhn B lm nh bng 4  15 D 60:
Tng t, s tam gic c C lm nh bng 5  10 D 50:
Trong s cc tam gic ny, c mt s tam gic c m 2 ln, l nhng tam gic nhn c B
v C lm nh. C 5  4 D 20 tam gic nh vy (nh c li l giao ca 5 ng thng k t B
v 4 ng thng k t C /: Suy ra c 60 C 50 20 D 90 tam gic.
7. Trong hnh sau c bao nhiu hnh vung?

p s. (30 hnh vung) C 4 loi hnh vung: 1  1; 2  2; 3  3 v 4  4: Do p s l


16 C 9 C 4 C 1 D 30:
8. Trong cc hnh sau c tng ng bao nhiu hnh ch nht?

169

Tp ch Epsilon, S 06, 12/2015


p s. 1; 9; 36 hnh vung.
9. Th d on xem trong hnh bi 7 c bao nhiu hnh ch nht? Kim tra d on ca bn
bng cch m trc tip.
p s. (100 hnh).
10. C bao nhiu hnh bnh hnh trong hnh sau?

p s. (45 hnh)
11. V 9 im trn mt phng sao cho c ng 74 tam gic c nh ti cc im ny.
Cch gii. Hy v lm sao c 10 tam gic suy bin!

B cc tng i
K.Kokhas ngh cho Hi ngh ma h, cuc thi ton gia cc thnh ph 2002:
Gi s ta c b cc s thc A D fa1 ; a2 ; : : : ; an g (trong cc s ai c th c nhng s bng
nhau). B cc tng i l b cc s c dng ai C aj vi i < j: Ta k hiu b ny l A.2/: V d
vi A D f1; 1; 2; 7g th A.2/ D f2; 3; 3; 8; 8; 9g:
1. T cc s a1 ; a2 ; a3 ; a4 ; a5 ta c th to thnh 10 tng i, k hiu chng l x1 ; x2 ; : : : ; x10 :
Chng minh rng nu bit cc s x1 ; x2 ; : : : ; x10 (nhng, d nhin, l khng bit cc s l
tng ca nhng s no), ta c th xc nh c cc s a1 ; a2 ; a3 ; a4 ; a 5:
2. Bit rng b s A c b cc tng i l f2; 2; 2; 3; 3; 3; 3; 4; 4; 4; 4; 5; 5; 5; 6g: Hy tm
tng bnh phng cc s ca b A:
3. a) Nu v d hai b, mi b c 4 s (trong mi b cho php c nhng s bng nhau) c b cc
tng i trng nhau.
b) Gi s b1 < b2 < b3 < b4 < b5 < b6 l b cc tng i ca mt b 4 s. Chng minh rng
c mt b bn s na cng c cng b cc tng i.
170

Tp ch Epsilon, S 06, 12/2015


4. Chng minh rng vi n D 2k; k D 1; 2; : : : tn ti hai b n s thc c cng b cc tng i.
5. Chng minh rng vi mi n 2k; b gm n s hon ton c xc nh bi b cc tng i
ca n.
6. a) Ch ra v d ba b s (c nhiu hn 2 s) c cng b cc tng i.
b) Chng minh rng khng tn ti ba b 4 s c cng b cc tng i.
c) Chng minh rng khng tn ti bn b 8 s c cng b cc tng i.
7. Hc tr km Sasha ngh ra 3 s thc v ghi ln bng 3 tng i, 3 tng i vi du tr v tt
c cc hiu i mt, tng cng l 12 s. Hi hc tr gii Nadia c th bit c tr tuyt i ca
cc s m Sasha ngh khng?
8. Chng minh rng t b fa1 ; a2 ; : : : ; an g cc s thc phn bit, s cc tng phn bit dng
n.n 1/
:
ai C aj c th l mi s nm gia 2n 1 v
2

Ti liu tham kho


[1] . ., . . . .:
, 1988.
[2] . . // . . 1971. . 9.
2. . 113119.
[3] . ., . // . 1991.
4. . 1418.
[4] . . .
.: , 2009.
[5] . // . 1999.
. 22. ( . .).
[6] ., ., .
// , . 2000. . 7 (168). .
7984.
[7] Bushell P. J. Shapiros Cyclic Sum // Bull. London Math. Soc. 1994: Vol. 26: No 6: P.
564574
[8] Bushell P. J., McLeod J. B. Shapiros cyclic inequality for even n // J. Inequal. & Appl.,
2002: Vol. 7.3/: P. 331348
[9] Cirtoaje V. Crux Mathematicorum. 2006: Vol. 32: No. 8: Problem 3195:
[10] Daykin D. E. Inequalities for certain cyclic sums // Proc. Edinburgh Math. Soc.
.2/ 1970=71: Vol. 17: P. 257262:
[11] Diananda P. H. Extensions of an inequality of H. S. Shapiro // Amer. Math. Monthly 1959:
Vol. 66: P. 489491:
171

Tp ch Epsilon, S 06, 12/2015


[12] Diananda P. H. On a conjecture of L. J. Mordell regarding an inequality involoving quadratic
forms // J. London Math. Soc. 1961: Vol. 36: P. 185192:
[13] Diananda P. H. Inequalities for a class of cyclic and other sums // J. London Math. Soc.
1962: Vol. 37: P. 424431:
[14] [14] Diananda P. H. Some cyclic and other inequalities // Proc. Cambridge Philos. Soc.
1962: Vol. 58: P. 425427:
[15] Diananda P. H. Some cyclic and other inequalities, II // Proc. Cambridge Philos. Soc. 1962:
Vol. 58: P. 703705:
[16] Diananda P. H. On a cyclic sum // Proc. Glasgow Math. Assoc. 1963: Vol. 6: P. 1113:
[17] Elbert A. On a cyclic inequality // Period. Math. Hungarica. 1973. Vol. 4. No 23. P.
163168.
[18] Malcolm M. A. A note on a conjecture of L. J. Mordell // Math. Comp. 1971: Vol. 25: P.
375377:
[19] Mitrinovic D. S., Pecaric J. E., Fink A. M. Classical and new inequalities in analysis,
Kluwer Academic Publishers Group, Dordrecht, 1993: (Mathematics and its Applications
(East European Series), Vol. 61/:
n
X

xr
n
 and some others // Abh. Math. Sem.
x
C xrC2
2
rD1 rC1
Univ. Hamburg. 1958: Vol. 22: P. 229240:

[20] Mordell L. J. On the inequality

[21] Nowosad P. Isoperimetric eigenvalue problems in algebras // Comm. Pure Appl. Math.
1968: Vol. 21: P. 401465:
[22] Shapiro H. S., Northover F. H. Amer. Math. Monthly. 1956. Vol. 63. No 3. P. 191192.
[23] Tanahashi K., Tomiyama J. Indecomposable positive maps in matrix algebras // Canad.
Math. Bull. 1988. Vol. 31. No 3. P. 308317.
[24] Troesch B. A. Full solution of Shapiros cyclic inequality // Notices Amer. Math. Soc. 1985.
Vol. 39. No 4. P. 318.
[25] Vukmirovic J. A note on an inequality for the cyclic sums introduced by D. E. Daykin //
Math. Balk. 1978: Vol. 8: P. 293297:
[26] Yamagami S. Cyclic inequalities // Proc. Amer. Math. Soc. 1993. Vol. 118. No 2. P.
521527.
[27] Zulauf A. Note on a conjecture of L. J. Mordell // Abh. Math. Sem. Univ. Hamburg. 1958:
Vol. 22: P. 240241:
[28] Zulauf A. Note on an Inequality // Math. Gazette. 1962. Vol. 46. No 355. P. 4142.

172

K THI TON QUC T FORMULA OF UNITY


THE THIRD MILLENIUM
Ban Bin tp Epsilon

K t s tp ch th 5, Ban bin tp s dnh mt chuyn mc gii thiu v cc


k thi trong nc v th gii. S th 5 va ri im qua mt s thi, li gii v
bnh lun cho chn i tuyn ca mt s trng chuyn ln trong c nc, chun
b cho k thi Hc sinh gii ton quc sp ti. Trong s ln ny, chng ti s tip
tc gii thiu v Formula of Unity - The Third Millennium, mt k thi trong khun
kh International Townament of the Towns (ITOT), k thi Ton gia cc thnh ph,
c t chc vo gia thng 10 ti H Ni va qua. V thi chnh thc cng nh
li gii ca ITOT, chng ti s gii thiu trong mt s bo khc.
y cng l bi cui cng ca Epsilon s 6 k ny, hn gp li c gi Epsilon
s 7!

1. thi dnh cho Khi lp R5


Bi 1. Peter, Basil v Anatoly gp tin tit kim mua mt qu bng gi 9 la. Bit rng, s
tin gp ca mi ngi khng nhiu hn mt na tng s tin gp ca hai ngi cn li. Hi
Peter ng gp bao nhiu tin?
Bi 2. Pauline vit hai s A v B ln bng. Victoria xa hai s A; B v vit tng C v tch D
ca chng. Sau , Pauline li xo hai s C v D; thay bi tng E v tch F ca chng. Bit
rng mt trong hai s E v F l s l. Hi l s no?
Bi 3. Ta ni rng hc sinh A hc tt hn hc sinh B nu im ca A cao hn im ca B trong
phn ln cc bi kim tra. Sau 3 bi kim tra, thy gio nhn xt rng hc sinh A hc tt hn hc
sinh B, hc sinh B hc tt hn hc sinh C v hc sinh C li hc tt hn hc sinh A: Liu iu
ny c th xy ra khng?
Bi 4. Nu Leon b im km trng, cu ta s dnh c bui ti ni di m. Cn nu ngc
li, cu ta s lun ni tht. Leon c mt c em gi lun c m cho ko nu nh hm c b
khng b im km. Mt bui ti, Leon ni vi m: "Hm nay con b nhiu im km hn em".
Hi c em gi ca Leon c c m cho ko hay khng?
Bi 5. Mt t lch ma thut ch ng ngy tt c cc ngy chn ca thng v sai ngy vo tt c
cc ngy l. Hi s ln nht cc ngy lin tip nhau c ghi cng ngy trn lch l bao nhiu?
V ngy c th l ngy no trong thng?
Bi 6. C bao nhiu s c biu din thp phn gm 10 ch s khc nhau v c cha on 0123?
Bi 7. Hnh vung 8  8 c v trn t giy k vung dc theo cc ng k. Alex ct hnh
vung 8  8 theo cc ng k thnh 7 phn vi chu vi bng nhau. Hy ch ra mt cch ct ca
Alex.
173

Tp ch Epsilon, S 06, 12/2015

2. thi dnh cho Khi lp R6


Bi 1. C 14 ngi ngi quanh mt vng trn. Peter, Victoria, Anatoly v Genghis ang ngi
cnh nhau theo th t v cc bn c cc ng xu mnh gi 1; 2; 5 v 10 rp. Mt ngi bt k c
th a mt ng xu ca mnh cho ngi bn tri hoc bn phi ca mnh nu c ng 3 ngi
ngi gia h. Sau mt lc chuyn nh vy, cc bn Peter, Victoria, Anatoly v Gengis li nhn
c cc ng xu. Hi lc ny, bn no ang cm ng xu no? Hy ch ra tt c cc kh nng
v chng minh khng cn trng hp no khc.
Bi 2. Pauline vit hai s A v B ln bng. Victoria xa hai s A; B v vit tng C v tch D
ca chng. Sau , Pauline li xo hai s C v D; thay bi tng E v tch F ca chng. Bit
rng mt trong hai s E v F l s l. Hi l s no?
Bi 3. Ta ni rng hc sinh A hc tt hn hc sinh B nu im ca A cao hn im ca B trong
phn ln cc bi kim tra. Sau 3 bi kim tra, thy gio nhn xt rng hc sinh A hc tt hn hc
sinh B, hc sinh B hc tt hn hc sinh C v hc sinh C li hc tt hn hc sinh A: Liu iu
ny c th xy ra khng?
Bi 4. Nu Leon b im km trng, cu ta s dnh c bui ti ni di m. Cn nu ngc
li, cu ta s lun ni tht. Leon c mt c em gi lun c m cho ko nu nh hm c b
khng b im km. Mt bui ti, Leon ni vi m: "Hm nay con b nhiu im km hn em".
Hi c em gi ca Leon c c m cho ko hay khng?
Bi 5. Mt t lch ma thut ch ng ngy tt c cc ngy chn ca thng v sai ngy vo tt c
cc ngy l. Hi s ln nht cc ngy lin tip nhau c ghi cng ngy trn lch l bao nhiu?
V ngy c th l ngy no trong thng?
Bi 6. C bao nhiu s c biu din thp phn gm 10 ch s khc nhau v c cha on 0123
hoc on 3210?
Bi 7. Hnh vung 8  8 c v trn t giy k vung dc theo cc ng k. Alex ct hnh
vung 8  8 theo cc ng k thnh 7 phn vi chu vi bng nhau. Hy ch ra mt cch ct ca
Alex.

3. thi dnh cho Khi lp R7


Bi 1. Mt t lch ma thut ch ng ngy tt c cc ngy chn ca thng v sai ngy vo tt c
cc ngy l. Hi s ln nht cc ngy lin tip nhau c ghi cng ngy trn lch l bao nhiu?
V ngy c th l nhng ngy no?
Bi 2. Hy in vo cc ca bng vung 5  5 cc s nguyn dng phn bit sao cho tng
cc s trn mi hng, mi ct l bng nhau v nh nht c th. Bit rng, cc s 1; 2; 3; 4; 2015
c in trc trn mt ng cho.
Bi 3. Hnh vung 8  8 c v trn t giy k vung dc theo cc ng k. Alex ct hnh
vung 8  8 theo cc ng k thnh 7 phn vi chu vi bng nhau. Hy ch ra mt cch ct ca
Alex.
174

Tp ch Epsilon, S 06, 12/2015


Bi 4. C 27 con gin tham gia mt cuc chy ua. Trong mi vng s c ba con gin chy. Mi
con gin chy vi tc c nh, khng i gia cc vng ua, v tc ca cc con gin l i
mt khc nhau. Sau mi vng, ngi ta ghi li th t v ch ca cc con gin tham gia vng
ua . Hi 14 vng ua c xc nh chnh xc theo th t hai con gin chy nhanh nht
khng?
Bi 5. Ta ni rng hc sinh A hc tt hn hc sinh B nu im ca A cao hn im ca B trong
phn ln cc bi kim tra. Sau 3 bi kim tra, thy gio nhn xt rng hc sinh A hc tt hn hc
sinh B; hc sinh B hc tt hn hc sinh C v hc sinh C li hc tt hn hc sinh A: Liu iu
ny c th xy ra khng?
Bi 6. Ta ni mt s nguyn dng l p nu n l tch cc giai tha ca cc s nguyn t
(khng nht thit phi phn bit). Ta gi mt s hu t dng l tt nu n l t s gia hai s
nguyn dng p. Chng minh rng tt c cc s hu t dng u tt.
Bi 7. Ta gi mt s nguyn dng l p nu dy cc ch s ca n tng thc s, v d 1589 l
s tng cn 447 th khng. Hy tm s nh nht cc s nguyn dng p vi tng l 2015:

4. thi dnh cho Khi lp R8


Bi 1. Hy in vo cc ca bng vung 5  5 cc s nguyn dng phn bit sao cho tng
cc s trn mi hng, mi ct l bng nhau v nh nht c th. Bit rng, cc s 1; 2; 3; 4; 2015
c in trc trn mt ng cho.
Bi 2. C 27 con gin tham gia mt cuc chy ua. Trong mi vng s c ba con gin chy. Mi
con gin chy vi tc c nh, khng i gia cc vng ua, v tc ca cc con gin l i
mt khc nhau. Sau mi vng, ngi ta ghi li th t v ch ca cc con gin tham gia vng
ua . Hi 14 vng ua c xc nh chnh xc theo th t hai con gin chy nhanh nht
khng?
Bi 3. Hy tm mt s nguyn dng sao cho tch cc c t nhin ca n l 1090:
Bi 4. John c 12 que g vi di mi que l mt s nguyn dng khng vt qu 56: Chng
minh rng John c 3 que c th to thnh mt tam gic.
Bi 5. Ta ni mt s nguyn dng l p nu n l tch cc giai tha ca cc s nguyn t
(khng nht thit phi phn bit). Ta gi mt s hu t dng l tt nu n l t s gia hai s
nguyn dng p. Chng minh rng tt c cc s hu t dng u tt.
Bi 6. Cho tam gic ABC vi B D 30 ; C D 105 v D l trung im on thng BC:
Tm gc BAD?
Bi 7. Ta ni rng hc sinh A hc tt hn hc sinh B nu im ca A cao hn im ca B trong
phn ln cc bi kim tra. Sau 3 bi kim tra, thy gio nhn xt rng hc sinh A hc tt hn hc
sinh B; hc sinh B hc tt hn hc sinh C v hc sinh C li hc tt hn hc sinh A: Liu iu
ny c th xy ra khng?
175

Tp ch Epsilon, S 06, 12/2015

5. thi dnh cho Khi lp R9


Bi 1. Cc nh ca mt a gic u 12 cnh c t mu xanh v . Bit rng trong 3 nh bt
k to thnh mt tam gic u, c t nht 2 nh mu . Chng minh rng ta c th chn 4 nh
to thnh mt hnh vung vi t nht 3 nh .
Bi 2. Ta ni mt s nguyn dng l p nu n l tch cc giai tha ca cc s nguyn t
(khng nht thit phi phn bit). Ta gi mt s hu t dng l tt nu n l t s gia hai s
nguyn dng p. Chng minh rng tt c cc s hu t dng u tt.
Bi 3. C 27 con gin tham gia mt cuc chy ua. Trong mi vng s c ba con gin chy. Mi
con gin chy vi tc c nh, khng i gia cc vng ua, v tc ca cc con gin l i
mt khc nhau. Sau mi vng, ngi ta ghi li th t v ch ca cc con gin tham gia vng
ua . Hi 14 vng ua c xc nh chnh xc theo th t hai con gin chy nhanh nht
khng?
Bi 4. Cho tam gic ABC vi B D 30 ; C D 105 v D l trung im on thng BC:
Tm gc BAD?
Bi 5. John c 12 que g vi di mi que l mt s nguyn dng khng vt qu 56: Chng
minh rng John c 3 que c th to thnh mt tam gic.
Bi 6. Hy tm mt s nguyn dng sao cho tch cc c t nhin ca n l 1090:
Bi 7. Tt c chng ta u bit 32 C 42 D 52 . Bn cnh , khng phi ai cng bit rng
102 C 112 C 122 D 132 C 142 : Liu c tn ti hay khng 2015 s nguyn dng lin tip sao
cho tng bnh phng ca 1008 s u tin bng tng bnh phng ca 1007 s sau ?

6. thi dnh cho Khi lp R10


Bi 1. Hai ch th Bugs v Roger c cc xem ai nhanh hn. xc nh ngi chin thng, hai
bn quyt nh t chc mt cuc thi. Mi bn th s nhy 50 mt theo mt hng v sau quay
li nhy ngc li. Bit rng, di mi bc nhy ca Bugs l 50 cm v ca Roger l 60cm,
nhng th Bugs nhy c 6 bc trong khi Roger ch nhy c 5 bc. Hi ai s ginh c
chin thng?
Bi 2. Vi nhng gi tr no ca n th ta c th chia mt hnh vung thnh n hnh ch nht ng
dng sao cho c t nht hai trong s chng l khng bng nhau?
Bi 3. C tn ti hay khng cc s nguyn dng a v b sao cho
lcm.a; b/ D lcm.a C 2015; b C 2016/
y, lcm.a; b/ c k hiu cho bi chung nh nht ca hai s a v b:
Bi 4. Cho tam gic ABC vi B D 30 ; C D 105 v D l trung im on thng BC:
Tm gc BAD?
Bi 5. Hy in vo cc ca bng vung 1010 cc s nguyn dng phn bit sao cho tng cc
s trn mi hng, mi ct l bng nhau v nh nht c th. Bit rng, cc s 1; 2; 3; 4; 5; 6; 7; 8; 9
v 2015 c in trc trn mt ng cho.
176

Tp ch Epsilon, S 06, 12/2015


Bi 6. ng trn ni tip tam gic ABC tip xc vi cc cnh AB; BC v AC ti cc im
C1 ; A1 v B1 tng ng. Chng minh rng
AC
CB
BA
C
C
> 4:
AB1
CA1
BC1
Bi 7. Tt c chng ta u bit 32 C 42 D 52 . Bn cnh , khng phi ai cng bit rng
102 C 112 C 122 D 132 C 142 : Khng nh sau ng hay sai: Vi mi s nguyn dng k; c
2k C 1 s nguyn dng lin tip sao cho tng bnh phng ca k C 1 s u tin bng tng
bnh phng ca k s cn li?

7. thi dnh cho Khi lp R11


Bi 1. Hai ch th Bugs v Roger c cc xem ai nhanh hn. xc nh ngi chin thng, hai
bn quyt nh t chc mt cuc thi. Mi bn th s nhy 50 mt theo mt hng v sau quay
li nhy ngc li. Bit rng, di mi bc nhy ca Bugs l 50 cm v ca Roger l 60cm,
nhng th Bugs nhy c 6 bc trong khi Roger ch nhy c 5 bc. Hi ai s ginh c
chin thng?
Bi 2. Vi nhng gi tr no ca n th ta c th chia mt hnh vung thnh n hnh ch nht ng
dng sao cho c t nht hai trong s chng l khng bng nhau?
Bi 3. C tn ti hay khng cc s nguyn dng a v b sao cho
lcm.a; b/ D lcm.a C 2015; b C 2016/
y, lcm.a; b/ c k hiu cho bi chung nh nht ca hai s a v b:
Bi 4. Cho tam gic ABC vi B D 30 ; C D 105 v D l trung im on thng BC:
Tm gc BAD?
Bi 5. Ti mi im c ta nguyn trn mt phng ta trng mt cy vi ng knh 10 6 .
Mt bc tiu phu n cy ti gc ta .0; 0/ v ng trn gc cy. Hi phn mt phng m anh
ta nhn thy c b gii hn hay khng? y, cc cy c coi nh l mt ct hnh tr v hn vi
cc trc cha cc im nguyn ca mt phng ta .
Bi 6. Hy ch ra mt b 4 s dng khng th l bn knh ca bn hnh cu i mt tip xc
nhau.
Bi 7. Tt c chng ta u bit 32 C 42 D 52 . Bn cnh , khng phi ai cng bit rng
102 C 112 C 122 D 132 C 142 : Khng nh sau ng hay sai: Vi mi s nguyn dng k; c
2k C 1 s nguyn dng lin tip sao cho tng bnh phng ca k C 1 s u tin bng tng
bnh phng ca k s cn li?

177

You might also like